Vous êtes sur la page 1sur 250
THE NEW YO CONTES T PROBLEM BOOK Problems and Solutions from the New York City Interscholastic Mathematics League 1975-1984 Grades 10-12 Mark E. Saul Gilbert W. Kessler SEL Gilley PETS erat) ii } PT Nee) aoe Be) aor ile ai THE NEW YORK CITY CONTEST PROBLEM BOOK Problems and Solutions from the New York City Interscholastic Mathematics League 1975-1984 Mark E. Saul Bronx High School of Science/Bronxville Schools Gilbert W. Kessler Canarsie High Schoo! Sheila Krilov Hunter College Campus School Lawrence Zimmerman Brooklyn Technical High School DALE SEYMOUR PUBLICATIONS Copyright © 1986 by Dale Seymour Publications. All rights reserved. Printed in the United States of America, Published simultaneously in Canada. Limited reproduction permission, The publisher grants permission to reproduce up to 100 copies of any part of this book for noncommercial classroom or individual use. Any further duplication is prohibited. ISBN 0-86651-307-8 Order Number DS01614 DALE SEYMOUR BUBLICATIONS. P.O BOX 10888 PALOALTO Canas03 3456789 ML 9897969594 SPRING 1975 FALL 1975 SPRING 1976 FALL 1976 SPRING 1977 FALL 1977 SPRING 1978 FALL 1978 SPRING 1979 FALL 1979 SPRING 1980 FALL 1980 SPRING 1981 FALL 1981 SPRING 1982 FALL 1982 SPRING 1983 FALL 1983 SPRING 1984 FALL 1984 APPENDIX A APPENDIX B: ANNOTATED BIBLIOGRAPHY. « INDEX... ..233 Selected Theorems. . CONTENTS Problems....+ 1 Problems..... 3 Problems 6 Problems.....10 Problems....+13 Problems.....15 Problems..../18 Problems.....20 23 Problens.. Problems... .26 Problens.....28 Problems. ....31 Problems.....34 Problems.....36 Problems.....39 Problems.....41 Problems, ....44 Problems.....46 Problems.....49 Problems, ,...52 Mass Points...,.220 4.224 Answers.....55 Answers..,..55 Answers. ....56 Answers... .56 Answers. ....57 Answers. .-,.57 Answers.....58 Answers. ..+.58 Answers....59 Answers...+.59 Answers...+.60 Answers... .60 Answers... +61 Answers. ..++61 Answers... +462 Answers... ..62 Answers. ....63 Answers.....63 Answers. ....64 Answers.....64 Solutions..... 65 Solutions..... 74 Solutions..... 82 Solutions..... 90 Solutions..... 97 Solutions... .106 Solutions.....113 Solutions,....121 Solutions.....130 Solutions.....138 Solutions. ....245 Solutions.....151 Solutions.....156 Solutions.....162 Solutions.....168 Solutions.....176 Solutions... .182 Solutions.....188 Solutions.....195 Solutions.....203 PREFACE The New York City Interscholastic Mathematics League is the oldest and largest local organization of its type in the United States. Records exist of competitions given by the League since the early 1920s, and there are indications that these contests were the continuation of an even earlier tradition. The contests have contributed to the unique position that New York City holds in mathematics education today. In national and inter- national contests, New York City high school students have shown a high degree of talent, interest, and achievement. Many of these students have gone on to make their careers in mathematics, and some of them have contributed problems that appear in the present book. Thus the New York City IML contests have been instrumental in developing a mathematical problem-solving community in New York City, which in turn has made its contribution to the field on a national level. The questions in this book are from the Senior A division contests. This division is open to teams of students at or below their senior year in high school. A series of five contests is given each term. Each contest consists of three pairs of questions, with a time limit of 9-12 minutes for each pair. A Senior B division is organized similarly, with somewhat easier questions. The junior division is open to teams of students at or below their junior year in high school; three contests are offered each term. The IML also runs eighth and ninth year contests and a series of elementary school competitions. ‘The IML has always been an organization of classroom teachers. Math teams are often coached by teachers on their own time, using their own talents and interests in developing materials for tean practice. For this reason, it is a particularly difficult task to acknowledge all the people whose work appears in this book. Many teachers and former students have contributed one or two problems, or one or two ideas that later became problems. In every case, the problems and solutions were subject to much editing, so that the original problem is often unrecognizable in its final form. Nonetheless, it seems appropriate to mention a few individuals who have been instrumental in creating the contests of the last ten years. Steven R, Conrad, formerly of Benjamin Cardozo High School and President of the League from 1968 to 1978, is responsible in large measure for the scope and format of the contests as they are held today. Irwin Kaufman, Director of Technology for the New York City Board of Education, Joel Arougheti of the High School of Art and Design, and Richard Geller of Stuyvesant High School, also contributed much time and effort to the organization of the League. Of the many contributors of problems, Harold Shapiro and Peter Ungar of the Courant Institute of Mathenatical Sciences, New York University, deserve special mention, as does Murray Klamkin of the University of Alberta. In the final preparation of the contest, Robert Saenger, Sherrill Mirsky, Sohn Reutershan, and Werner Weingartner of the Bronx High School of Science, and Harry Ruderman of Hunter College High School devoted much time and effort. Maria De Salvio of the Bureau of Curriculum and Instruction and Miriam Refkin, now retired from New York City School District 15, helped enormously with the myriad organizational details. Finally, the late Harry Sitomer was responsible for many original problems and solutions. Perhaps the most important, though least tangible, contribution has been made by the math team coaches of the New York City high schools and by their students. Without the teachers’ work and the students' interest, the contests would have been meaningless. As graduates, the following students later made direct contributions to the contests: David Karr, Brian Sheppard, and Benji Fisher of the Bronx High School of Science; David Wolland of Hunter College High School; Ashfaq Munshi and David Zagorski of Stuyvesant Wigh School; Alan Edelman of Canarsie High School; Mark Prysant of South Shore High School; and Nicomedes Alonso of Long Islané City High School. All of these individuals, and many others, have made the New York City Interscholastic Mathematics League a major force in the field of mathematics education in New York City and throughout the country. Mark E. Saul President, NYCIML SUGGESTIONS FOR USING THIS BOOK The problems in this book are presented, for the most part, exactly as they appeared in the original contests. In a very few cases, there were errors in the statement of the problem, and these have been corrected. The percentage of contestants who correctly solved the problem is indicated below the problem number for many of the contests. This information may help in anticipating the level of difficulty of a given problem (a task that is surprisingly difficult, even after much experience). References to items in Appendix A (Selected Theorems) are given as decimal numbers, while references to the bibliography are made by giving the author‘s name (and the book's title, if appropriate). ‘The two appendices and the index are provided to help organize the material contained in the problems. Appendix A is cross-referenced. A search for related problems can begin with a topic in the appendix, by following the references given to problems using that topic. Or it can begin with a problen, by following the reference given in the problem to the appendix. ‘The index groups problems in the same general area. We have tried to list problems under more than one category. The process of classifying a problem is one step toward solving it, and a hasty classification may be a step in the wrong direction. Some of the most interesting problems resist classification. We have grouped these under the heading "Difficult to Classify." ‘The annotated bibliography is intended to help bridge the gap between short- answer contest problems and mathenatics in its more natural setting. A contest problem at its best will stimulate the problem solver to do further investigation. The bibliography provides a set of starting points for this process. Neither the appendices, nor the index, nor the bibliography are meant as complete surveys of their domains. Many interesting theorems and formuias are not mentioned in the appendix, simply because they are not used in the problems. Similarly, the bibliography is by no means exhaustive. It offers only a small part of the rich literature that has developed in problem solving and recreational mathematics. It is intended to furnish an introduction to the tradition on which the New York City contests are based. Problems—Spring 1975 PROBLEMS Problems—Spring 1975 1. cass) 2. (ass) 3. C38) 4. (23%) 5. C98) 6. (58%) 2 (18) 8. (318) 9. (208) 10. (478) The positive integer n, when divided by 3, 4, 5, 6, and 7, leaves remainders of 2, 3, 4, 5, and 6, respectively. Find the least possible value of n. Find all positive numbers = that satisfy (2 + Logie)? + (-1 + logiax)? = (1 + logion”)*. ‘Triangles I and II are coplanar, and each point of Triangle II is interior to and 2 units from Triangle I. [f the side-lengths of ‘Triangle I are 13, 14, and 15, find the area of the region that is the intersection of the interior of Triangle I and the exterior of Triangle II. There are two times between 5 A.M. and 6 A.M. when the hands of an accurate clock are perpendicular. Exactly how many minutes must elapse between these two times? Quadrilateral ABCD has right angles at B and D. If ABCD is kite shaped with 48 = AD = 20 and BC = CD = 15, find the length of a radius of the circle inscribe? in ABCD. How many real values of x satisfy ve? — 4 ~ 2/e?7 >I = 2? The two-digit base ten numeral 4B(A # 0) represents the positive integer x. Find every integer « for which the sum of the digits in the product kx is equal to A + B for every integral value of k from 1 to 9 inclusive. Find the coordinates of that point on the circle with equation (x - 6)? + (Yy - 5)? = 25 that is nearest the point (-2,11). Each of a set of four congruent circles is internally tangent to a fifth circle and externally tangent to exactly two other circles in the set. If the radius-length of each of the congruent circles is m(¥2 - 1), find the radius-length of the fifth circle in terms of m. If a 0 and if (@ + 1)(@ + 2)(@ + 3)(@ + 4) +1 = (42? + Be + 0)? find the ordered triple (4,2,C). Circle 0 has radius-length 5, and point A, coplanar with the circle, is 5 units fron the center 0. One tangent from 4 to the circle intersects the circle at point 7. If X is a point on A? such that XO intersects the circle at a point Y with XA = XY, find the length of XA. lb + b¥a | aVB - bia yo avb - bva avd + bYa equation expressing a explicitly in terms of b. Ifa>0,b>0,a#b, and write an The ratio of the width of a rectangle to its length equals the ratio of its length to half its perimeter. If the area of the rectangle is 50(v5 - 1) and the width of the rectangle is k(V5 - 1), compute the value of k. Let q, », and s be, three distinct real numbers such that \ (2 - Qe - ») = a” + an + be and (@ - q)(@- 8) = 2° + be + ac. Tf a, B, and @ are real numbers such that abo(a -b) #0, write an equation expressing ¢ explicitly in terms of a and b. Find all real values of x that satisfy Jz] + 3 - |2 + 3/ = Find all positive real values of © that satisfy ate 2- fe <1, 2. (358) 22. (78) 23. (39%) 24. (20%) 2s. (18) 26. (25%) 27. (188) 28. (18%) 29. (26%) 30. (aoa) Prob! 1. (9%) Problems—Fall 1975 The length of JB is 12, and ¥ is a variable point on AB._ Squares AXCD and XBEF are drawn in a plane on the same side of AB. The centers of these squares are Y and Z, respectively, and W is the midpoint of YZ. As X moves from 4 to B, find the length of the path traced out by point W. Find all real values of = that satisfy 3c? - 20? +241 _ 30? - 20? Se - 13 Bel Wet ae Be ae Ses In MABC, perpendiculars from A to the bisectors of angle B and angle C meet the bisectors in D and #, respectively. The line through D and F intersects AC at X and 4B at Y. What fractional part of the area of MABC is the area of repion XYBC? Find all real values of « that satisfy 2" - dx? + Se? - 42 41 If 4 and B are digits and the base ten numeral 30455 can be expressed as the product 225m, find all positive integral values of n. If a = 716, b = VOSS, o = VOGT, and d = (.2), arrange a, b, ec, and d in increasing order. In AABC, C' is on AB such that_AC':C'B = 1:2, and Bt is on AC such, that AB':B'C = 3:4. If BB’ n Cct = (P} and if (a'} = AP n BC, find AP:PA". ‘The equation of line m is 3c + 4y = 12 in a rectangular coordinate system. If the line whose equation is 3x + dy = k is 2 units from m, find the two possible values of k. In isosceles right triangle ABC, M is the midpoint of hypotenuse 4B. An equilateral triangle has one vertex on AC, one on BC, and one at M. If AZ = 24 and if the side-length of the equilateral triangle is k(/3 - 1), find k. Ifat sy? +s? =a? anda ty sazearsyrs c? equation expressing «yz explicitly in terms of a. a, write an ms—Fall 1975 Let [x] denote the greatest integer n such that n 3, a= 3,55, adifa,sa, +4,,- 4, forall n> 3 Find the sum of the first thirty terms of the sequence, 12. (45%) 13. (68) 14. C98) 15. (36%) 16, (16%) 17. (80%) 1s. (258) 19. (208) 20. (39%) 21, ( 58) Problems—Fall 1975 In AABC, D is on AB and £ is on BC. Let CD n Aw = {X} and let Bk n WC = (r}, If AK:KE 2 and BK:KF = 4:1, find CK:xD. The sun of the ages of a father, a mother, and a daughter is 73. When the father will be twice as old 2s the daughter, the sum of their two ages will be 152, Write an equation expressing the mother’s present age, m, in terms of the daughter's present age, d. The bases AB and CD of isosceles trapezoid ABCD are 12 units apart, 4B = 10, and CD = 8. Point R is on the axis of symmetry of the trapezoid so that angle CRB measures 90°, Find the possible distances from F to AB. The distances from a point on the circumcircle of an equilateral triangle to the two nearest vertices are 3 and 6, Find the distance from the point to the furthest vertex. If ay = X, ap = 17, and, in general, a, = Y"-1, calculate aye ai,avo,e0y correct to the nearest thousendeh when x = (2)*7", On hypotenuse AB of right triangle ABC, D is the point for which CB = ED. tf angle 5 measures 40°, find the measure of angle ACD. A two-person game is played in which each player, in turn, removes 1, 2, 3, 4, 5, 6, or 7 toothpicks from a conmon pile, until the pile is exhausted. The player taking the last toothpick LOSES. If the starting pile contains 1000 toothpicks, how many toothpicks must the first player take on his first turn to guarantee a win with perfect subsequent play? 12(?_- de + 3) wider es 5 has a Find all integral values of « for which positive integral value. A piece of paper in the shape of an equilateral triangle ABC has AB 8, When A is folded over to any point on BC, a crease of length x is formed which joins some point of AC to sone point of AB. As A is moved along EC from B to C, « varies. If the greatest value of x is M and the least value of x is m, find the ordered pair (sm). ‘The coordinates of the vertices of pentagon ABCDE are A(0,0), B(11,0), C(11,2), D(6,2) and (0,8). The line whose equation is a = k partitions this pentagonal region into two regions of equal area. If k = a + bv6, find the ordered pair of integers (a,b), Problems—Spring 1976 22. (098) 23. (308) 24. (228) 25. (a8) 30. (59%) Two men, 4 and B, start at the same point, walk in opposite directions, and reach their respective destinations, X and Y, each in one hour. Had they exchanged destinations from the start, A would have asrived at Y 35 minutes after B would have arrived at X. Find the ratio of 4's speed to B's speed. Find the integer @ for which 2° = 656,356,768. The length of each side of MEC is 6. If X is the_trisection point of Ci nearer C and if median Alf intersects BY at J, then Mi = v3. Find k. A unit fraction is a fraction of the form +, where » is an integer greater than 1. Find the two largest unit fractions, each of which is the square of a rational number, whose sum is also the square of a rational number. Ifa #0 and if 2 + y =a and x? + y? = b, write an equation expressing a” + y? explicitly in terms of a and b. Find all ordered pairs of real nunbers (a,b) for which 3v=- 2 10 and x = ay + by A piece of paper in the shape of an equilateral triangle 4BC has 4B = 15. When 4 is folded over to the point D on BC for which__ BD = 3, a crease is formed along a line that joins a point on AB to a point on AC. The length of the crease is 3¥n. Find x. The length of dianeter ZB of a circle is 12, and its trisection points are C and D. Semicircles are drawn on AC and AD as diameters on the same side of A®, and on 2) and BC as diameters, on the other side of AP. These four semicircles partition the circle into three regions. If the area of the middle region is kn, Find k. In the five-digit base ten numeral 4BCDE (4 # 0), different letters do not necessarily represent different digits. If this numeral is the fourth power of an integer and if 4+ (+= 8+ 0D, find the digit C. Problems—Spring 1976 1 Find the sum of the infinite series whose nth term is In A4BC, angle C measures 60° and angle A > angle B. The bisector of angle C intersects 4B at E. If CE is a mean proportional between AF and EB, find the value of AE/AC. (S78) (s%) a7) 10. cass) lu. (288) 12. (478) 13. (868) 14. (44%) Problems—Spring 1976 In Moc, 8 on HE so that 40:08 = 1:2, and G 48 on ZD ao that CasGD ="3:2., If BE intersects AC at P, find BO:AR. In base fifty, the integer x is represented by the numeral CC and «* is represented by the numeral AHBA, If C > 0, express all possible values of B in base ten. In MBC, angle A measures 120°, AB + BC = 21, and AC + BC = Find BC. One of the roots of esi, _2 (2 , 1+k r xe] 10 S , where k is a negative integer. Find k. If 100 more than the sum of n consecutive integers is equal to the sum of the next n consecutive integers, find 1. External squares are drawn on each side of a rhombus, and the centers of these squares are joined to form convex quadrilateral Q. Tf the length of each side of the rhombus is 6 and if one of its angles has a degree-measure of 30, find the area of quadri- lateral Q. A lattice point in a rectangular coordinate plane is a point both of Whose teerdtnates aro integers. How many’ lattice points are there on the circle that is the graph of the equation x? + y? = 6287 A proper divisor of a positive integer is a positive integral divisor Of the integer, other than the integer itself. Find the largest proper divisor of 1,030,301. The points of intersection of the graphs of 20 and x* + y® = 41 are joined to form a convex quadrilateral. Find the area of the quadrilateral. A merchant bought some oranges at the rate of 3 for 16 cents. He bought twice as many oranges at the rate of 4 for 21 cents. To make a profit of 20%, based on his investment, he sold them all at the rate of 3 for k cents. Find k. A club found that it could achieve a membership ratio of 2 adults for each minor either by inducting 24 adults or by expelling « minors. Find x. The length of each side of MBC is 10, and a circle of radius- length 3 is tangent to AB_and to AC, If the distance from the center of the circle to BC is av3 - b, find the ordered pair of rational nunbers (a,b). Problems—Spring 1976 1s. (55%) 16. (298) i (35%) 18. (138) 19 (78%) 20. (328) 21. (288) 22. (328) 23. (348) 24, 8%) In MBC, D is on BG so that B ABLED ='536. IF B :DC = 3:2, and £ is on AD so that imtersects AC at P, find EE:EF. Find the sum of the infinite series a) + az +a3 +... +4, + where, for ma positive integer, = dm - 3 or n= 4m - 2 = dm = Lor n= 4m. Let a, b, and ¢ be positiye integers such that a is the cube of an integer, c= b+}, anda’ +b’ =’. Find the least possible value of ¢. In rectangle ABCD, AB = 6 and BC = 8. Fquilateral triangles ADE and DCF are drawn on the exterior of the rectangle. If the area of ABEF is a/3 +b, find the ordered pair of rational numbers {a.b). An integer is represented by a two-digit base ten numeral. If three times the sum of its digits is added to the integer, the result is the original integer with digits reversed. Find all such positive integers. In a triangle, the lengths of the three medians are 9, 12, and 15. Find the length of the side to which the longest median is drawn. ‘The points A(-1,4) and B(2,-3) are in a rectangular coordinate systen. Point C is on AB with AC:CB = 3:4, Find the coordinates of point Cc. Find all ordered triples of real numbers (2,y,2) that satisfy | rye = ve- ye ve, g+y+2= 8, and e-ysne4 Let A be a leg of the right triangle of least perimeter whose sides have integral lengths, whose hypotenuse is one unit longer than AB, and in which AB > 100. Find AB. In an arithmetic progression whose n°? term is as ai = 6 a2 = 8 and as = 10. A second sequence, whose n°” term is Bs has by and, form > 1,b,= 3, b, explicitly in terms of n. +a,_,- Write an equation expressing 25. 26. 28. 29. 30. 31. (238) 32. (28%) 33. (37%) Problems—Spring 1976 the expression Vio +410 + Ji0 * + , where the dots indicate an infinite repetition of the indicated pattern of operations, a+b can be expressed in the form >» where a, b, and ¢ are integers, no two of which have a common prime factor. Find the ordered triple (a,b,c). In a rectangular coordinate system, find the area of the region bounded by the graph of |x| + |y - 1] ‘The altitude and the median from vertex 4 of MBC are 4 and 5 units long, respectively. The angle determined by AF and the median is bisected by the altitude. Find AC. The squares of three positive integers are in arithmetic progression, and the third integer is 12 more than the first. Find the three integers. A rectangular sheet of paper, ABCD, is folded and creased so that AD lies along DC. The crease formed determines a point F on AB, and it is found the F is the trisection point of 4B nearer 2. Find EC:BC. Find all real values of « that satisfy (16x* - 9)? + (9x? - 16)? (25 - 25)*. (Due to unusual circumstances, the six problems that follow served as replacements for the six problems immediately preceding.) 1 ; nee ney. If sin z + cos x= -Z and if t 1 <2 <1, find the value of cos 2x. Find all ordered triples of real numbers («,y,#) that satisfy ys +a = 13, ma + zy = 25, my + ys = 20. In honor of the economic freeze in the state of New York, governnent officials are rumored to have put forth a plan to institute one of two new types of thermometers. On these new scales, °F represents degrees Ford and °C represents degrees Carey. It is known that 40°F = 25°C, that 280°F = 125°C, and that degrees Ford is a linear function of degrees Carey. Write an equation expressing °F explicitly in terns of °C. Problems—Fall 1976 34. Im MBC, AB = 5, BC = 6, and AC = 7. Points P on HB, 0 on BC, (138) and R on AC are located so that AP = 2, 89 = 2, and CR= 3. Tf the area of ABC is x, then the area of APOR is kx/3S. Find the nunerical value of k. 35. In the sequence 6, «, y» 16, the first three terms form an (418) arithmetic sequence and the last three terns form a geometric sequence. Find all possible ordered pairs (¢,y). 36. ‘The parabola that is the graph of the equation y = ax* + be +e (388) passes through the points (-1,-11), (1,1), and (2,4). Find the coordinates of the vertex of this parabola. Problems—Fall 1976 1. For all nonzero real nunbers x, the function is defined by (359) pq + 2)]'/* =k. If y is a nonzero real number, find the value of [f+ oi in terms of k. 2. In the complex domain, find all solutions of (x + 1)° = 2° +1. (33%) a : 3. rf tan(}0 +2) = A sec 2x +B tan 2x is an identity for all real Q7%) — yatues of x for which both sides of the equation are defined, find the ordered pair of rational numbers (4,8). 4 In acute triangte ABC, AC = 14 and the length of the altitude from (19%) Bis 12. A square has two of its vertices on AC and its other two vertices on AB and HC, respectively. Find the length of a side of this square. 5. In AABC, D is the point on BC for which AD bisects median BE. (528) Find apipc. 6. Find all ordered triples of real nunbers (=,y,2) that setisfy (ASS) RoTy alot oM GET ena (6Nenf etl oa i-Tes on Find the maximum area of a triangle in which the length of one (55%) side is 6 and the sum of the lengths of the other two sides is 10. 8. Of all ordered triples of positive integers (e,y,2) that satisfy (27) 4° . 2"¥ = 25%, find the ordered triple with the smallest value of 2. 9. Find the positive integer whose cube exceeds its square by 4624. (738) -10- 10. (21%) ll. (45%) 12. (ass) 13. (49%) 14. (19%) 15. (44s) 16. (53%) 17. (63%) 18. (488) 19. (278) 20. (478) Problems—Fall 1976 ‘The length of a radius of a circle is 3 + 2V3. Three congruent circles are drawn in the interior of the original circle, each internally tangent to the original circle and externally tangent to the others. Find the length of a radius of one of the three congruent circles. If log(logflog(1og «)]) = 0, where the base of each logarithm is 10, then © = 10%, Find the positive integer k. cot?x - tan’x T+ cote + tne has a minimum value of m and a maximum value of M, for 2 a real number. Find the ordered pair (m0). 1f « # SL, were & is an integer, then +2 sinte In ABC, D is on AB such that ADjDB = 3:2 and & is on BC such that BE:EC = 3:2. If DE intersects AC at F, find DE:EF. Find the value of tan x if sine + cos @=Zand3n< 2 + aw 17s ieee + 9° a7 n=0 ‘Two jugs each have a capacity of x gallons. One is filled with wine and the other is filled with water. Two gallons are taken from each jug and are then transferred into the other, after which each jug has its contents thoroughly mixed. If two gallons were now taken from each jug and then transferred into the other, the amount of wine in each jug would be the same. Find x. Bases AB and CD of trapezoid ABCD are each perpendicular to leg AD. If CD = 6, AB = 8, and AD = 10, find the average of the areas of all noncongruent triangles two of whose vertices are A and D and whose third vertex is a point on BC. Find all solutions of x(x? - 1) - 6(2? +241) In base x, Zand .T7 are numerals for the same number. Find =. Half the perimeter of a rectangle exceeds the length of one of its diagonals by one-third the length of the longer side. Find the tangent of the angle formed by its longer side and one of its diagonals. -u- Problems—Fall 1976 2. (37%) 22. (208) 23. (46%) 24. (21s) 25. (398) 26. (128) 27. (50%) 28. (32%) 29. (338) 30. (1st) While B is riding a bicycle from Here to There, C is driving a car from There to Here, each at a steady rate along the same road. They start at the same time and, after passing each other, 3 takes 25 times as long to complete the journey as C. Find the ratio of the speed of the bicycle to the speed of the car. If a square and a regular hexagon have equal areas, then the ratio of their respective perimeters is Vk:1. Find k. A pair of twin primes is a pair of primes that are consecutive odé integers. “Find the largest integer that is a divisor of the sum of the two elements in every pair of twin primes, if these two elements are each primes greater than 3. If the length of a circumradius of regular octagon, 4BCDEFGH is 4, the area of the quadrilateral determined by lines 45, CD, HF, and Gi is k(2 + V2). Find k. Tsed the value of cos (Arssin f+ arctan were Are denotes principal value. Aman lives in a building located five blocks south and five blocks west of the building in which his girlfriend lives. Thus, in driving to his girlfriend's home each evening, he drives ten blocks. All the streets are in a rectangular pattern and all are available to him for driving. In how many different ways can he drive from his home to his girlfriend's hone, driving only ten blocks? Ife >a>Oandifa-bee an? + be +e = 0. 0, find the larger root of A box, in the shape of a rectangular solid, has a length of 12, a width of 8, and a height of 8. What is the length, along the outside surface of the box, of the shortest path that can te— drawn between two Of the vertices of the box which do not lie in the same face of the box? The ancient Greeks used Z as an approximation of /2. The ratio of the absolute value of the approximation error to vZ 1s Find k. The sides of a right triangle all have integral lengths, and the sum of the lengths of one of the legs and the hypotenuse is 49. Find all possible lengths for the other leg. -12- Problems—Spring 1977 Problems—Spring 1977 1. In a triangle, segments are drawn from one vertex to the trisection (18%) points of the opposite side. A median drawn from a second vertex is divided, by these segments, into the continued ratio x:y: If 2 >y>s, find xyz. a An integer is chosen at random from {x|100 < 2 < 300}. Find the (52%) probability that this integer is divisible by 7 or 11. 3. For all real numbers 2 and y, the function f satisfies f(zy) (28%) = f(z) + Fly) and f(0) 4 0. Find the numerical value of /(1977). 4, If 0 <@ Sing, find all C3) possible values of tan 2. 5. Find all ordered pairs of real numbers (x,y) that satisfy (258) e+y+vesy= 56 ande-y + ve-y = 30. 6. In the binomial expansion of (x + y)°, the sum of the two middle (36%) terms is equal to the sum of the first and last terms. If x+y #0, find the numerical value of the ratio (@ - y)*:ay. 1 Find the ordered pair of positive numbers (2,y) that satisfies (2%) we ye yea? -y?, 8. Tf sin®o + cos = 2, find all possible values of sin 20. (15%) 9. For all « > 0 and all y > 9, the function f satisfies f(y) = f(e) (688) + fly). TF f(2) = a and f(3) = b, find the value of f(72) explicitly in terms of a and b. 10. Find the ordered pair of rational numbers («,y) that satisfies OM) +33 = 04 vy. i. ‘The three-digit base numeral 7y3 is twice the three-digit base (35%) numeral 3y7. If x is a positive integer, express the sum = + yas a base ten numeral. 12. In an infinite sequence of regular hexagons, each hexagon except (338) the first is formed by connecting the midpoints of the sides of the previous hexagon. If the perimeter of the first hexagon is 12, find the sum of the perimeters of all the hexagons. 13. If Arc denotes principal value, find all real values of © that (ss) satisfy arctan 2e + arctan 3 = 2, -13- Problems—Spring 1977 14. (s18) 15. (808) 16. (28%) 17. (46%) 18. (658) 19. (62%) 20. ais) ai. (35%) 22. (25%) 23. (45%) 24. (578) 25. (548) ‘Two different-sized vertical poles are staked into level ground. ‘The two laser beams from the top of each pole to the ground-level base of the other pole cross at a point 24 meters above ground. If the top of the shorter pole is 40 meters above ground and the top of the taller pole is « meters above ground, find x. Let f and g be functions defined by f(x) = 9x + 1 and g(x) = 27. Find all values of « that satisfy f{g(e)] = glf(@)]. Every expression of the form a*b* + b’c? + od? + d’a’® can be expressed as the sum of two squares in at least two different ways. Find any one of the three possible ordered pairs of, positive integers (sy), with « > y, that satisfies x? + y? = 44? + 10? + 10? + 337 + 337 + 5? 4 5? + 4g, If xy # 0, express in simplest form Sicbaluelod 2 (20 B)' + Cea)" + (ara) — (2 YD When the system 3¢ - 4y < 6 and 6x + 4y = 15 is solved, all solutions (2,3) have the forn(2 Find the ratio vis. Find the value of = that satisfies 1og,25 - 1og,4 = log,v/. Find all ordered pairs of positive integers (x,y) that satisfy wit 292 y Find all ordered pairs of real numbers (x,y) that satisfy a? - y? = 19 and x*y - ay? = 6. A circle is inscribed in a 3-4-5 triangle. A segment is drawn from the smaller acute angle to the point of tangency on the opposite side. This segment is divided in the ratio p:q by the segnent drawn fron the larger acute angle to the point of tangency on its opposite side. If p > q, find p:q. If Are denotes principal value and if 2 = Aresin $+ avccos #8, find the value of sin 2. If © > 1, the equation {x + v2e-1-Vx - ir - 1 = VE is an identity. Find, in simplest form, the value of k. Let be the base « numeral representing the number where x is an integer greater than 1. Find W. -14- Problems—Fall 1977 26. A grain of sand, when at the point (,y) of a coordinate plane, is (22%) allowed to be moved to the point (x + 1,y) or to the point (e,y +1), but not both. If the grain starts at (0,0) and moves to (4,4), find the probability that it passes through (2,2). 27. ‘Traveling at respective rates of a and b kilometers per hour, (75%) A and B head directly towards each other across a distance of 120 kilometers. If both start at 10:30 A.M., they will meet at noon. If 4 starts at 10 A.M. and B starts at 11 A.M., they will also meet at noon. Find the ordered pair (a,b). 28. Tn AABC, the lines containing the bisector of angle # and the (38%) bisector of an exterior angle_at C intersect_at Q. A line, through @, drawn parallel to BC, intersects AC at F and AB at D. If BD = 8 and EC = 6, find ED. 29. Let © be a nonintegral positive number and let [x] denote the (68%) greatest integer n such that 1, find the two smallest integral values of n for which x? +241 is a factor of (x + 1)" - 2” polynomials with integral coefficients = 1, over the set of The sum of two of the three roots of x’ + ax* + be + ¢ = 0, where a, b, and ¢ are real constants, is zero. Write an equation expressing ¢ explicitly in terns of a and b. There are two circles that pass through (1,9) and (8,8) and are tangent to the z-axis. Find the lengths of their radii. In an isosceles triangle, the lengths of the base and the median to one leg are equal. Find the cosine of the vertex angle. A function f is defined on the positive integers by f(1) = 0 and F@) = 3f( - 1) +1 fer all n> 1. Express the numerical value of (1003) - f(1001) in the form (p")(q3), where p and q are positive primes and » and @ are integers. The lengths of the sides of a triangle are 13, 37, and 40, Find the length of the altitude to the longest side. Ifa+b+c=0 anda’ +b* +c? = 216, find the value of abe. A detachment of s marchers was arranged in a square formation in which there were as many marchers in each row as there were rows. At the last minute, 64 marchers were removed from the detachment, and those who remained could again be arranged into a square formation. If z > 64, find all possible values of 2. If sin®15° is one root of «* + br + ¢ of rational numbers (b,c). 0, find the ordered pair With seamstress X making x dresses per day and seamstress Y making y dresses per day, X requires § more days than does ¥ to make 60 dresses. If each seamstress were to increase her daily output by 1 dress, X would need only 3 days more than ¥ to make 60 dresses. Find the ordered pair («,y). -17- Problems—Spring 1978 30. Suppose f and g are functions respectively defined by f(z) = 2 +2 and g(a) =<. Find all 2 > -2 for which gC) L0H) L peey, Problems—Spring 1978 1 10. ‘Three different nunbers are randomly selected from among the first ten positive integers. Find the probability that the sum of these three numbers is 15. A circle is inscribed in a 3-4-5 triangle. A second circle, interior to the triangle, is tangent to the first circle and to both sides of the larger acute angle of the triangle. If the length of a radius of the second circle is r, find the numerical ler value of =. The lengths of the sides of a triangle are 4, 13, and 15. Find the sine of the angle opposite the side whose length is 13. In base eight, the four-digit numeral BRCC is the square of the two-digit numeral 44, Find the ordered triple of digits (4,B,C). Find the length of the line segments whose endpoints have polar coordinates (7,40°) and (15,100°). Find the first of 100 consecutive odd integers whose sum is 1001°°, Find the real value of © that satisfies logst + logst + loge:x = In arranging the ordered pairs of positive integers thusly: (1,1), (2), (2,0, (1,3), (2.2), (3.1, (4), ..-, if two ordered pairs have a different element-sum, the one with the smaller element-sum comes first. If they have the same element-sum, the one with the smaller first element comes first. In this arrangenent, find the 1978th ordered pair. Find all ordered pairs of rational numbers (x,y) that satisfy 6e(2e + 1) +1 = y3. (You may want to use this special case of Fermat's Last Theorem: If a? + b? = c? has a solution in integers, then abe = 0.) In a right triangle whose legs have integral lengths @ and b and whose hypotenuse has integral length c, if a = 3 and ¢ = S, then a@+e¢-= 8, a perfect cube. Find the least value of ¢ greater than 5 for which a and c are relatively prime odd integers whose sum is the cube of an integer. -18- ue 12. 13. 14. 1s. 16. 17. 18. 19. 20. a. 22. 23. Problems—Spring 1978 ‘The longer base of an isosceles trapezoid is a chord of a circle, and the shorter base is tangent to the circle. If the length of one leg is 5 and the lengths of the bases are 24 and 18, find the area of the circle. Find the simplified numerical value of tan 20° + tan 40° + v3 tan 20° tan 40°. 24 3g find the value of sin‘x + cos*x. If sin 2e = For all complex numbers x, the polynomial function P is defined by P(e + 2) - P(x) = 6 and P(O) = 2. Write an equation expressing P(e} explicitly in terms of 2. All the circles in an infinite chain of successively tangent circles of decreasing size are tangent to both sides of a 60° angle. If the length of a radius of the largest of the circles in the chain is 105, find the sum of the lengths of the radii of all the circles in the chain. The sun of the squares of the first and fourth terms of an arithmetic progression is 260, while the sum of the squares of the second and third terms is 136. Find the product of these four terms. If a, b, and ¢ are positive numbers, find the numerical value of the product 1og,? * log.c* + log.a’. Find the numerical value of b for which the length of the path from A(0,2) to B(b,0) to C(e,10) to D(5,9) will be a mininun. In an isosceles triangle with a 30° vertex angle, the perpendicular bisector of one leg divides the other leg into the ratio k:2. If k> 2, find k. The function f is defined by f(1) = 1 and f(n) = f(% - 1) + 2n for every positive integer n > 2. Write an equation expressing f(r) explicitly in terms of n. After witnessing a four-car "Demolition Derby," a young man constructed the alphametic (4) (NEW) = DENT, in which NEW is odd and 7 = 4, If WEW and DENT respectively represent three-digit and four-digit base ten numerals (with digits ", E, W and D, E, , 7, respectively), find + £ + W. Find the fundamental period of the function defined by f(«) = sin‘e + cos"x, where x is a real number. The value of ¢ $2 bills and f $5 bills is $87. If all ordered pairs (#,f) are equally likely, find the probability that both + and f are prime numbers. -19- Problems—Fall 1978 24. 25. 26. 27. 28. 29. 30. 7 ‘The decimal expansion of n-1 10°" product of the next four digits of this expansion. begins .02040816. Find the In angle 48C, angle A = 45 and angle ¢ = 30, If altitude Bi intersects median Al at P, then AP:PM = 1:k. Find k, Find all ordered pairs of positive integers (#,y) that satisfy a sy? - ay = 49. ‘Two students toss a fair coin. Whenever it lands heads up, the first student wins one dollar. If not, (s)he loses one dollar. Find the probability that the first student's net winnings after five tosses is exactly one dollar. In terms of », the sum of the squares of the first n positive integers is + "+2. re the sum of the squares of the first n positive odd integers is an? + bn? + en + d, find the ordered 4-tuple (a,,2,d). If # is a real number, find the minimum value of (@ + 8)(2 - 8)( + 6)(e - 6). A circle is inscribed in a right triangle the lengths of whose legs are 30 and 40. Find the length of the line segment whose endpoints are the vertex of the right angle and the point of tangency, on the hypotenuse, of the inscribed circle. Problems—Fall 1978 et (49%) 2. (47%) 3. (33%) In the isosceles triangle, the length of the base is 30 and the length of an altitude to one of the legs is 24, Find the length of one of the legs. Find all integral values of « that satisfy 2 . 2 a+vl- a 2-VE- a When 16 is subtracted from a three-digit nunber (greater than 100) and the resulting difference is divided by 2, the result is a three-digit number (greater than 100) whose digits are those of the original number, but in reverse order. If the sum of the three digits is 20, find the original number. -20- 13. (46%) 14. (298) 1s. (8933 16. (818) Problems—Fall 1978 The degree-measure of angle ABC is 120, and AB = 1. Lying on BC are the points By, Bz, By, ..., 8, such that the distance from 4 to B, is vk, If the distance from B, to 8B is 2, find k. x x £0 Psy Which of the folowing integers is the largest? se, 27, 3° A fair coin is tossed 17 times. Determine the probability of obtaining at least 11 consecutive heads. A middle-aged man, noting that his present age was a prime number, observed that the next occurrence of this kind for him was exactly as far away as was the most recent one of this kind. The last time he could have made such an observation was when he was five years old, How old is the man (in years)? In a sequence whose first term is 1, the n'™ term, a,, is recursively defined by a, = 34, #1, "> 1, Find ais7e- Find a11 values of = that satisfy oa +2 2 1 - 35(2 +5 +50 Find the numerical value of sin 40° sin 80° + sin 80° sin 160° + sin 160° sin 320°. If logio3 = a and logio7 = b, find the value of 1og9 in terms of a and b. Point C lies on a circle one of whose diameters is AB. The bisector of angle CAB intersects BC at D and intersects the circle at EB. If BD = 25 and CD = 7, find BF. Find the least positive integral value of 2? for which there exist different positive integers 2, y, and 2 that satisfy 2? + y? = 2°. Find the value of x that satisfies logsw = (-2 + log2100) (log3v2). For all real numbers # and y, the function f has the property that fle+y) =f) +f). If fC) = 3, find the value of f(10). In a triangle, the degree-measure of one angle is 60 more than that of another. ‘The ratio of the lengths of the sides opposite these two angles is 2:1, Find the degree-measure of the largest angle of this triangle. -21- Problems—Fall 1978 17. (29%) 18. (28%) 19. (398) 20. (13%) a. (cass) 22. (558) 23. (59%) 24, (43%) 2s. (538) 26. (308) 27. (238) Mr. Smith left on a trip very early one morning. Not wishing to wake Mrs. Smith, Mr. Smith packed in the dark. He had socks that were alike except for color, and his socks came in six different colors. Find the least nunber of socks he would have had to pack to be Sure of getting at least four matching pairs. Consider eight points that are equally spaced on a circle in which the length of a radius is 1. Find the product of the distances from one of the points to each of the other seven points. ‘The points with rectangular coordinates (1,1), (3.5), and (-1,4) are three vertices of a parallelogram, Find all possible ordered pairs that could represent the coordinates of the fourth vertex of this parallelogran. In calculating his income tax, a teacher found that he could calculate his federel tax as 20% of his income after state taxes were deducted, and he could calculate his state tax as 10% of his income after federal taxes were deducted. If the teacher's income before taxes was $9800, how much federal tax did he pay (in dollars)? Using only multiplication, find the least nunber of multiplications required to calculate «*', given the real number 2. For any fixed real numbers a, b, and c, find the maximum number of different real solutions for « in the equation Ve ~a+¥e-Bb = Ve-s. A lattice point in a rectangular coordinate plane is defined as a point both of whose coordinates are integers. Find the number of lattice points in the region defined by |a| + |y| < 5. Find all values of a for which the set of simultaneous equations ntytara cay sgt sa, eisyi te sa, at yt + 2! is consistent (i.e., has 4 common solution). If [k] denotes the greatest integer < k, then the set of all real numbers # that satisfy [2? - (@ - 1)?] * 10 is (ela <2 T - va? + Ze +1) a Cl - QtTT - et Fae TH -22- 28. (65%) 29. (978) 30. (458) Problems—Spring 1979 Im convex quadrilateral ABCD, the area of triangle DAB is 1, that of triangle ABC is 6, and that of triangle CDA is 2. Find the area of triangle BCD. Find three distinct positive integers whose sum equals the product of the largest two of them. If (1 + cos 60° + ¢ sin 60°)'? = a + bi where a and b are real numbers and i is the imaginary unit, find the value of b. Problems—Spring 1979 1. 7 Find the numerical value of (1og23) (loge4), expressing your answer in simplest form. The lengths of the four sides of a trapezoid are 2, 3, /7, and 2v7, with the last two being the lengths of the parallel sides. om_a point P within or on the trapezoid, the four perpendiculars PA, PB, PC, and PD are drawn to the respective sides of the trapezoid (where one or more of these perpendiculars may have a length of 0). Determine the maximum value of PA + PB + PC + PD. On a trip to the supermarket, a shopper bought 25 items, none of which cost more than $1.10.’ The sales tax on the total purchase was 65¢. The bill was incorrectly calculated to be $28.97. If the cashier's error on any one item was less than 10¢, at least how many errors were made by the cashier? ~ Find the degree-measure of the least positive angle that satisfies sin 20° + sin 40° = sin 9, In the cube determined by 1 << 2, 1y >a, then +y + 2 - max(x,y) ~ max(x,2) - max(y,2) + max(z,y,3) always equals one of z, y, or 3. Determine which one. In a circle, two chords of lengths 4 and 11 respectively subtend central angles whose degree-measures are in the ratio 1 to 3 respectively. Determine the length of a radius of the circle. -25- Problems—Fall 1979 29. 30. Consider all positive integers between 3000 and 4000 which, when divided by any of the first three primes, leave a remainder of 1. Find the sum of all these integers. Suppose the probability that the Yankees will beat the Mets in a game of baseball is 2/5. If these teams were to meet in a World Series, what would be the probability that the Mets would win by four games to two? (For those who may not know, there are no ties in the game of baseball, and a World Series ends when one team has won four games). Problems—Fall 1979 1. a7) 2. (42%) 3. (288) 4. (168) (26%) a) (678) (gt) (G28) The three positive integers 3, 36, and « are such that their arithmetic mean exceeds their geometric mean by 13. Find the value of =. The number ¥, represented by the decimal numeral ABC, is divided by the decimal numeral AC. The quotient is 9 and the remainder is 0. What is the maximum value possible for 17? T£ log 80 = a and log 45 = b, find log 36 in terms of @ and b. In equilateral triangle ABC, points D, £, and F are on and CA, respectively, with AD = BE = CF = 1 and DB = EC = FA = V3. ‘The area of triangle DEF is a + b/3. Find the ordered pair of rational numbers (a,b). 24 Ae oe 4 < 180° in4 If sin A = 5 and 90° < 4 < 180°, find sin 5 Find all real numbers x that satisfy the equation (Vie +1 + /2- =) + (Viel + oa)? + E+ Ta)? = ‘The sum of the first six terms of an arithmetic progression whose first term is 1 is equal to the sum of the first six terms of the geometric progression beginning 1, 2, 4, .... Find the sixth term of the arithmetic progression. If a permutation of the digits 1, 2, 3, 4, 5 is written at random, what is the probability that exactly two of the digits will occur in their correct places? (The digit » is in its correct place if it occupies the nth position in the permutation.) n 99 = 1 2 or + + + log > If S = log $+ log 2+ tog $+... + tog ET log ay? where the base of the logarithms is 10, find the integral value of 5 in simplest form. -26- 10. (32%) 1. (33%) 12. (318) 13. (27%) 4. (87%) 15. (478) 16. (13%) 17. (13%) 18. (208) 19. (29%) 20. (348) Problems—Fall 1979 The slopes of lines £1 and Lz are 4 and 1. Find the slope of the line that bisects the acute angle formed by lines Ly and L2- If A +B = 225%, and (cot A)(cot B) # 0, find the numerical value of Lt cota, 1+ cot B cot A cot B Write an equation expressing o explicitly in terms of a and b if asaty,b=asy?, andosary The largest power of 7 that divides 343! is 7”. Find the value of x. (For positive integral n, ! = n(n = 1)(n - 2) ... (3)(2)().) How many real values of # satisfy the equation tan{Arctan 2 + Arctan(»?)] = 2? ‘The two legs of an isosceles triangle have length y. What is the maximum area that the triangle can have? Express your answer in terms of y. Find all values of « for which there is no ordered pair (x,y) satisfying the following simultaneous equations: 2 2 a2 = Day - 3y Boye 79 a2 = day + Sy? es ay? 7 O° The dimensions of a rectangle are x and y, where « and y are integers. Its area is divided into unit squares by Zines parallel to the sides of the rectangle. The umber of unit squares touching the sides of the rectangle is equal to the nunber of unit squares not touching the sides. Find all possible values of the product «ty. When a certain polynomial is divided by « - 2, the remainder is 2. When the polynomial is divided by x + 2, the remainder is -2. What is the remainder when the polynomial is divided by 2° - 4? In how many zeroes does the quantity 127! end? Find all values of x that satisfy the equation: aft a ve+fh fh Ve fhe svt -27- Problems—Spring 1980 21. Find m if n> 2 and (nt)?(n? = 2)1 = 3e*)1C@m = 2)? (268) 22. Le 0° < @ < 90° and tan®w - 4 tan +1 = 0, find the numerical (52%) value of sin 2x in simplest form. 23. Find all ordered triples (+,y,2) of real numbers that satisfy (378) nya? = 24, ay%z = 54, and z'yz = 6 simultaneously. 24. In regular hexagon 4BCDEF, the bisector of angle AFB meets AB at (17%) G, and # is a point on the bisector of angle BFE such that FH = FG. Find the numerical value of the ratio of the area of triangle FOE to the area of the hexagon. 25. What is the base of the numeral system in which = .333 ... (where (32%) the digit 3 repeats forever)? 26. Find the ordered pair of real numbers (2,y) that_satisfies (438) simultaneously + y + V+ y= 12 andz-y + B-y = 6. 27. Four students are about to take a test containing 100 questions of (22%) equal difficulty. The probabilities are that the first student will answer 60 questions correctly, the second will answer 50 correctly, and the others will answer 40 and 25 correctly, respectively. What is the probability that at least one student will correctly answer the first question? i t,t, 1 , 1. 28.1, Given that > 1 and soesy * Toga * Toga * Tops * Tapia” Tout ° find the value of the integer «. 29. Points A, 8, C, and D lie on the graph of y? + ey + 2? + 3m + 4y {és 52" ofand esch’has an absclasa of" oy “1. OF thee points, tet 4 and C be the furthest apart. If 4c =¥p + qV3, where p and q are integers, find p + q- 30. The area of a circle circumscribed about an isosceles triangle whose (15%) legs are 1 unit long and whose base is # units long is given by the formula 4 = avRt » where @ and b are real numbers. Find the ordered pair (a,b)- Problems—Spring 1980 1. Five math league coaches, one each from the Bronx, Nassau, Westchester, Canton, and Hudson, are to be seated in a row at the speakers’ table et a convention. How many possible seating arrangements are there if the Bronx and Canton coaches cannot be seated next to one another? -28- 3. 10. ld. 12. 13. 14. Problems—Spring 1980 Express 2? + 2 + 2c + 4x? + 2 as the product of two polynomials of degree two. Find all integers = such that (VE - 1)/E(E +1) = 6. 1 1 1 Find the sum of the (finite) series ———+ =F V2+vVI VB 4 v2 VE + VB + ss + = whose n®™ term is ——+—.. VB + Vm wave Solve for all real values of x, 0° < x < 90°: sin 40° + cos 40° vI+ sine. In triangle ABC, AC = 3, BC = 4, and AB_= 5. A semicircle with its center on segment AC is tangent to AB and BC. Find the radius of the semicircle. After graduation exercises at North Beach High School, each senior gave a snapshot of himself (herself) to every other senior and received a snapshot in return. If 870 snapshots were exchanged, how many seniors were in the graduating class? The length = of a side of an equilateral triangle is also a root of the equation «® - 2x‘ - 120 = 0. Find the area of this triangle. Find the largest three-digit nunber (written in decimal notation) that is divisible by 22 and such that the sum of the units digit and the tens digit is 11. The polynomial ax* + br? + cx? + de + e, where a > 0, has integral coefficients whose greatest common divisor is 1. Find the poly- nomial if its value is zero when x = /-7 and when x = Find the ordered pair of real numbers (x,y) that satisfies = geigr led gi Three circles with centers 01, 02, and 03 are externally tangent in pairs at points 4, B, and C, Find the area of the triangular region ABC if the radii of the circles are 1, 2, and 3. 4 ‘ive for fa: + Solve for al1 real values of 2: logst + 7557 If the domain for x is all complex numbers, find the solution set of 4c* + 1 = 0. Express each element of the solution set in the form a + bi, where a and b are rational numbers. -29- Problems—Spring 1980 1s. 16. 17. 18. 19. 20. 21. 22. 23. 24. 25. 26. There are two circles that have their centers on the x-axis, pass through the origin, and are tangent to the circle (a - 7)? + (y - 6)? = 25, Find the radius of the smaller of these two circles. Urn A contains 2 white balls, and urn 8 contains 1 white ball and 2 black balls. Two balls are drawn at random from Urn 3 and, without their color being noted, are placed in urn A. Then one ball is drawn from urn A. What is the probability that this ball is white? Find the degree-measure of all angles 4 for which -180° < A < 180° and cos*4 + 2 sind = 1, The lengths of the sides of a triangle are in arithmetic progres- sion. The area of the triangle is 126 square centimeters, and the radius of the inscribed circle is 3 centimeters. Find the length, in centimeters, of the shortest side of the triangle. Find all positive integers m for which »* - 1 is prime. A polynomial f(x), with real coefficients, is of degree 3. If f(4) = f(-4) = -4 and f(0) = 1, how many real solutions are there to the equation f(a) = 0? The arithmetic mean of two positive numbers exceeds their geometric mean by 50. By how much does the square root of the larger of the two numbers exceed the square root of the smaller? P(#) and Q(«) are polynomials with positive integral coefficients such that (Q(x? = (P(x) + 2x7 + 1, Find Q(x). The equation 4x? + ar + b = 0 is such that one of its roots is the sum, and the other the product, of the roots of 2x7 + 2° - 3 = 0. Find the ordered pair (a,b). An infinite sequence of real numbers a1, az» as, «++ has,the property that for all n>, a +az +a; +e. +a,= 7a, and and a) = 5. Find the numerical value of a1 + az + a3 +... + asp. The integers from 1 through 10 are written on index cards. It is desired to color each of these cards with one of three colors (xed, white, or blue), in such a way that any two cards with integers differing by less than three are different colors. In how many ways can this be done? Find all ordered pairs of positive integers (i,y) that satisfy gi -y? 721. -30- 27, 28. 29. 30. Problems—Fall 1980 Solve for 2 (log, (logs (logse))) = Chord AP of a circle is extended through 8 to an exterior point D, and a Line is drawn from D tangent to the circle at C. If 4B = CD and triangle BCD has area 25 square centimeters, then the area of triangle ABC can be expressed as a + by'S square centimeters. Find the ordered pair (a,b). One vertex of a triangle is the point (0,6), and one side lies along the x-axis. ‘The area of this triangle is divided into two equal parts by the line y = kK, Find k, ‘A man has forgotten how many children he has. He does, however, know that if he adds the number of children to its square, squares the result, and then subtracts the fourth power of the original nunber, he obtains the same result as the one obtained if he adds the original nunber to its square, subtracts 3, and then multiplies by 12. How many children docs he have? Problems—Fall 1980 rb (838) 2. (57%) 3. (87%) 4 (548) Ss. (478) 6. (52%) 7. (358) A telegram costs m cents for 12 words and s cents for each additional word. If k > 12, express in terms of m, k, and the cost of sending a telegram of words. Tf P(e + 3) = 2% + Te + 4 and P(e) = ax® triple (a,b,c). + be +e, find the ordered In a basketball tournament, each of six teams played five game: one gane against each of the other five teams. No game ended in a tie, and, at the conclusion of the tournament, no two teams had won the same number of games. How many games were won by the team that won the tournament? In parallelogram ABCD, AC and BD are diagonals. If BC = 7, 4B = 8, and angle = 60°, find the value of AC? - 2D?, It is known that any odd number greater than 7 can be expressed as the sum of three odd primes, not necessarily distinct. Find the smallest odd number that can be expressed as the sum of three odd primes in three different ways (1 is not considered a prime). Find the mimerical value of log, ,,,,25 VIS. When the polynomial f(a) is divided by « - a, the quotient is @(2) and the remainder is R. When f(2) is divided by 3(e - a), the quotient is g + @(x) and the remainder is h + R, where g and h (as Well as a and R) are constants. Find the ordered pair (g,h). -31- Problems—Fall 1980 lo. (59%) 1. (30%) 12. (20%) 13, (378) 14. (428) 15. (35%) 16. (42% 17. (66%) 18. (1st) 19. (408) In isosceles triangle ABC, E is the midpoint of base BC. Points P and G are on line segments AB and AC, respectively, and BFG is an equilateral triangle, If BC = 6 and AB = 5, then FG = (-24/11) (a + By3). Fina the ordered pair (a,b) of rational numbers. The roots of the equation x? - pz +q= 0 area and b. The roots of the equation «* - mz + n= 0 area + 2/b and b + 2/a. Write an equation expressing n explicitly in terms of q only. The sum of m consecutive positive odd integers, starting with the least one, a, is 105. Find the smallest possible value of a Find the ordered pair of positive integers (x,y) for which log “log(zy)) = log(log x) + log(log y)- (All logarithms have base ten.) A median of a triangle is equal in length to the geometric mean of the lengths of the sides that include it. If these two sides are 7 and 10, find the length of the side of the triangle to which the median is drawn. ‘In triangle ABC, D is the midpoint of BC and £ is the midpoint of AD. Ray BE intersects AC at F. Find the numerical value of FE) AP EB FO * Find the greatest common factor of all numbers of the form 2. 3°" ~ 1, where n is an integer greater than 1. ‘The integer m is greater than 7 and is the first of five consecutive integers that are, respectively, multiples of 3, 4, 5, 6, and 7. Find the smailest such m. In acute triangle ABC, AB = ¢, BC = a, CA = b, and ae = 2. Find the munerical value of £284 + £086 | The angles of a triangle are in the ratio 3:4:5. Find the numerical value of the ratio of the sine of the smallest angle to the sine of the next smallest angle. WB is the diameter of a circle, and ABCD is an inscribed trapezoid. If the areas of triangles ABC and ACD are 150 and 120 square units, respectively, find the length of a leg of the trapezoid. 1 If sine = —1_ tan & » find the numerical value of cos =. 20. (358) 21. (358) 22. (16%) 23. (318) 24, (54%) 25. (25%) 26. ( 98) 27. (428) 28. (59%) 29. (25%) 30. as) Problems—Fall 1980 Runners A and B run a race from point P to point @ and back again. B starts out running at 9 miles per hour and A at 12 miles per hour. As B is running towards point @ he meets A, who is already returning to point P. As soon as they meet, 3 increases his speed, and the race ends in a tie. By how much did 5 increase his speed? Find the largest positive integer g such that g is not the square of an integer and V10 + vq can be expressed as /= + vy, where # and y are positive integers, « # y. In trapezoid ABCD, base AD is twice as long as base BG. F is the midpoint of leg 4B and F is a point on leg CD. Ray EF intersects ray AD in G and AD = DG. Find the ratio CF:FD. If the logarithm is taken to the base ten and © denotes radian measure, for how many positive values of x does logiow = sin x? The four jacks, four queens, and four kings of an ordinary deck of cards are removed and thoroughly shuffled. The first three cards are turned face up one at a time in an experiment to detect extrasensory perception. A subject in another room announces that the first card is a jack, the second is a queen, and the third is aking. Assuming he is guessing, what is the probability that he is correct? In triangle ABC, AB = 15, BC = 14, CA = 14, and DC and EA are the altitudes from C and 4, respectively. Find DE. — The equation «* + ax’ + be + ¢ = 0 has the root *¥v28 + 6 -*y/28 - If a, b, and ¢ are integers, find the ordered triple (a,b,c). If 0 @ < 0/2, find all values of © for which cos 3 * cos x + sin Se + sin @ = cos Urn 4 contains 8 balls, each either black or white. Urn B contains 3 black balls and 2 white balls. Each morning, a ball is drawn at random from urn 4 and placed in urn B. Every evening a ball is chosen from urn B. Then the balls are arranged as before for the next morning's drawing. The evening ball is white 3/8 of the time. How many white balls are in urn 4? If the reciprocal of (z + y) is equal to the sum of the reciprocal of and the reciprocal of y, find all possible numerical values of =. y Find the numerical value of (sin 10°)(sin 50°) (sin 70°). 33+ Problems—Spring 1981 Problems—Spring 1981 ny If the ratio of a to b is r and a # b, express, in terms of r, the (72%) ratio of a+b toa-b. 2. In triangle ABC, points_D and_£ lie on CA and GB, respectively, so (23%) that DE is parallel to BA. DE divides the triangle into two regions such that the ratio of the area of the smaller region to that of the larger equals the ratio of the area of the larger region to that of triangle 4BC. If DEC is the smaller region, find the ratio of the area of triangle DEC to that of triangle ABC. 3 Find all ordered pairs (p,q) of real numbers for which |p| = |q] (75%) 1 and the equation x* + pa + q = 0 has only real roots. 4 A circle of radius 1 is externally tangent to seven smaller (258) congruent circles. Each of these smaller circles is externally tangent to two of the other smaller circles. If a = sin 7/7, express the radius of one of these snaller circles in terms of a. 5. If a+ 3, find the absolute value of a - 2. (46%) 6. A man lost in the woods decides to flip a coin, walking one nile (27%) east each time it appears heads and one mile vest cach tine it appears tails. He flips the coin six times and walks accordingly. Find the probability that he will be at least two miles from his starting point at the end of this procedure. 7 After a hard math test, each of the 35 students in a math class (66%) got a peck at the teacher's grade book. No student sav all of the grades, and no student was able to see his own grade. Each student noticed exactly ten failures among the grades (sjhe saw. What is the smallest number of failures that could have occurred in the class? 8. If the radius of the inscribed circle of a regular heptagon (seven- (47%) sided figure) is 1, find the distance from the center of the inscribed circle to one of the longest diagonals. If b = sin 7/14, express your answer in tens of b. me 9. Let A= »Be and ¢ = “5 , where 7s) ae at? a - 2")@ - 2°) - 2”**) 4 0. Write an equation expressing C explicitly in terms of A and 2. 10. Gilbert is eight years younger than Irwin. If Irwin were one year (52%) older, he would be exactly ” times as old as Gilbert is now, where n is a positive integer. Find all possible integral values of Gilbert's age. =34- uu. (548) 12, (468) 13. (438) 14. (198) 15. (608) 16. (26%) 17. (27%) 18. (13%) 19. (698) 20. (238) al. (578) 22, (41%) Problems—Spring 1982 If (a + bi)? = b + ai, where ¢ = V=1, find the ordered pair of positive numbers (a,b). Four points are coplanar but not collinear. What is the minimum nunber of midpoints of line segments determined by these four points? If0 W are positive integers. ‘The outside surface of the block is painted, and the block is then cut into 6M unit cubes. Exactly half of these unit cubes have no paint at all on them. Find the least possible value of The ratio of John's age to Mary's age is now *. If 1 0, find the ordered pair of real numbers (a,b). Find the radius of the circle in which chord AB, measuring 2 centimeters, and chord 4C, measuring 5V3 centimeters, meet at an angle of 30°. If Arctan denotes principal value, solve for = 3 arctan 3+ arotan 3+ Arotan 2. a The integer 999,999,995,904 may be factored as a’® + b? +e ad + e+ f, where a, b,c, d, e, and f are primes anda 1 by t= Fn(n + 1), How many of the first 100 "triangular" numbers are multiples of 3? For certain integers x, one can find another integer » so that ne =n+10e. Find the largest such integer x- Find the degree-measure of all angles 6 such that 0° < 6 < 90° and sin 6 + sin 26 = cos 6 + cos 26. In acute triangle ABC, AB = S, AC = 7, and angle BAC = 60°. A point X is selected on arc BC of the circle circumscribing ABC, and perpendiculars YM and YX are dropped to lines AB and AC, respectively. Find the largest possible value for the length of WK. Problems—Spring 1982 ‘The units digit of the difference of the squares of two integers (written in base ten notation) is 2. What is the units digit of the sum of these two squares? Find the sum of all natural numbers that divide 3087 without remainder. If John gets 97 on the next math test, his average will be 90. If he gets 73, his average will be 87. How many tests has John already taken? Points X, Y, Z are chosen on sides AB, BC, AC respectively, of triangle ABC. If AY:XB = BY:¥C = CZ:ZA = 1:4, find the ratio of the area of triangle X¥% to that of triangle ABC. A girl bought some 1¢, 2¢, and S¢ stamps, for a total value of $1-00. She bought at least one of each kind, and got ten times as many 1¢ stamps as 2¢ stamps. How many S¢ stamps did she buy? If the ordered pair (#,y) of nonzero real nunbers satisfies simultaneously 2 + 2xy + y? - 2c - 2y = 0 and 2? - y? - 2x + 2y = 0, find the nunerical value of «+. Of five lines, no twa are parallel and no three are coincident. Into how many bounded regions do they divide the plane? -39- Problems—Spring 1982 10. ll. 12. 13. 14. 1s. 16. 7 18. ‘The number 500 = 2? + 5? has no prime factors other than 2 and 5. Some other such numbers are 2, 16, 10, and 1 (this last has no prime factors at all, so certainly none other than 2 or 5). Find the sum of all such numbers less than or equal to 500. Point X is on side CD of parallelogram ABCD. Angle ADC is acute and XC = CB, XB = BA, and XA = AD. Find the measure of angle ADX. The polynomial x" + 64 can be expressed as {P(x)]? - [Q(«)]?, where P(z) and @(z) are polynomials with positive integer coefficients, and the degree of P(x) is larger than that of Q(z), Find P(z). i , cos 2b ef Find the munerical value of =Hy5p(S0tse’ apy for 0< <5, © aed. Points X, ¥, and Z are chosen on sides AB, BC, CA, respectively, of triangle ABC, and AX:XB = BY:YC = (2:24 <1. If the area of triangle XYZ is.7/25 that of triangle ABC, find the ratio AY:4B. ‘The number 120 + 7384 can be expressed as va + vB, where a and b are rational and a 1, with positive integer coefficients, and where Qi) has the larger degree. Find Q(z). Points A, B, and @ are consecutive vertices of a regular polygon whose side_is 1 unit long. Point D is the foot of the perpendicular from C to 4B (extended). If BD = y and AC = a, write an equation expressing y in terms of a. The sequence mj, "2, 73, "ys ... consists of the number 1 and all those natural numbers greater than 1 whose prime factorizations consist of only 2's, 3's, and 5's, Find the infinite sum Pei ,aya my * ip is ye Problems—Fall 1982 19. If the number of permutations of n objects taken r at a time is 6 times the number of combinations of x objects taken » at a time, find the numerical value of r. 20. id the greatest integer less than 2¥5 + /21. 2. A worker received a salary increase of p% one year and a decrease of p% the next year. The net result over the two years was a decrease of xt. Write an equation expressing « in terms of p. 22. The number 27? + 1 has exactly one prime factor greater than 1000. Find it. 23. In isosceles triangle 4BC, angle ACB = 108°, Find in radical form the numerical value of AB:AC. — 24. T€ A and B are positive acute angles, tan A= 2, and sin B = find the degree-measure of A + 2B. vio 25. A square-free number is a natural number that is not divisible by the square of any natural number (other than 1). How many square- free numbers are there less than or equal to 100? 26. 1f 2 = J20 + 14V2 + ¥20 - 1472, find the numerical value of 2° - 6x in simplest form. 27. For how many value(s) of #, 0 <2 < 2n, does sin = + sin 2 + sin 3x = 0? 28. In triangle ABC, AB = BC = 5. Point D is on line segment AC and DC = 4, If BD = 4, find AD. 29. The probability of getting exactly k heads in 35 tosses of a fair coin is the same as the probability of getting exactly k + 1 heads in 36 tosses. Find i 30. In isosceles triangle ABC, CD is the altitude to base AB and AB is the bisector of angle CAB. If AF = 2CD, find the degree-measure of angle ACB. Problems—Fall 1982 a Find all real e for which e+ i= 20. (82%) 2. How many triples (a,b,c) are there of positive integers with the (55%) following two properties: (1) a, b, and ¢ are primes and @Q) a - bP =o? -41- Problems—Fall 1982 lu. (40%) 12. (358) 13. (93%) 14. (528) 15. (488) 16. (47%) If the hundreds digit of a three-digit number is increased by n, and both the tens and units digits are decreased by n, the new nunber formed is n times greater than the original number. Find the original nunber. Express in simplest form the real number (*//iB + 72)? . Two circles of radii 6 and 2 are coplanar, and their centers are 12 units apart. Find the length of their common external tangent. In how many ways can ten different marbles be placed in three different urns, if each urn must have at least one marble in it? ‘The product of two positive integers is 588000. Find the largest possible value for the greatest common divisor of the two integers. e+4 fe-i_s Find all real # such that /Z* 44/24 = 3. For # > 0, find the smallest possible value of 2 + 5. From the corners of square ABCD, equal isosceles triangles are cut off, leaving a regular octagon. Find the ratio 6f the area of the octagon to that of the original square. ‘Two circles of radii 6 and 2 are coplanar, and their centers are 12 units apart. How far from the smaller circle do their common internal tangents meet? If a, b, © are real numbers such that a+b +c = 0, find the largest possible value of (a? +b? + 0%)? a+ bes oF How many of the numbers 2', 27, 2%, ... , 279°, written in base ten notation, end in 6? 7 Find aff real = > 0 such that log,4 + loge = 27 For «> 0, find the minimum value of ‘Two nonintersecting circles have radii of 9 and 16. A third circle is tangent to these two and also to one of their common external tangents. If the centers of all three circles are collinear, find the radius of the third circle. -42- 17. (828) 18. (388) 19. (49%) 20. (718) a. (69%) 22. (428) 23. (128) 24. (61%) 25. (60%) 26. (66%) 27. (63%) 28. (35%) 29. (31%) 30. (01s) Problems—Fall 1982 ‘Twice Jack's age plus three times Jill's age equals 31. Their ages are integers, and the difference in their ages is a multiple of 4. If Jack's age is a and Jill's age is b, find the ordered pair (a,b). Three sides of a triangle are in arithmetic progression. The middle side has length 2, and the angle opposite this side is 60°. Find the area of the triangle. If tan 6 + cot 9 = Pando <0 <2, find all possible values of sec 8. If az + by = 8 passes through (2,-3) and (1,4), it also passes through (11,k). Find k. For 2 > 0, find the smallest possible value of G*2U+=) | Find the largest prime p such that no number of the form 999...9 has a factor of p. ‘Two nonintersecting circles have radii 1 and 9, and one of their common external tangents is 12 units long. A third circle is tangent to the first two and also to one of their common external tangents. Find the radius of the third circie. ‘The three angles A, B, C of triangle ABC satisfy cos 4 cos B + sin A sin B sin C= 1. Find the degree-measure of angle 4. CTR k a fe fa 2)" ps, For all a, b> 0, yt /2 JE =(Z) . Find the rational number k. Find the numerical vaiue of 1 + 3(2) + s(2)° + x(2)' + of) « If 2 can be any real number, find the minimal value of (4 + VIB)™ + (4 - VIB)". How many four-digit numbers, made up only of the digits 1, 2 are multiples of 9? The decimal representation of the integer l consists entirely of 9's, and is a multiple of 17. Find the smallest possible munber of digits in the decimal representation of iV. Find all values of @ such that 0 < @ 5. 26. Find the area of a triangle whose sides have lengths a, b, and ¢ (488) if 4a’b? - (a? + B® - 0”)? = 16. 27. Find all ordered triples of integers (a,b,c) such that 0 5, a, = a, Find ai999- 2. ‘The numbers 1, 2, 3,..., 1000 are written in a row. Sam started at 1 and circled every 24th number in red. Janet started at 1 and circled every 15th number in blue. What is the smallest possible (positive) difference between a red number and a blue number? 3. How many five-digit numbers (in base ten notation, with the left- most digit not equal to zero) are there such that each digit is strictly greater than the sum of the digits to its right (in particular, the tens digit is larger than the units digit)? 4 If0 1, The sequence {a,} is defined as follows: ax a, is the sum of the digits in the decimal representation of @,_,- Find the numerical value of ayo. In triangle ABC, AB = 3, BC = 4, AC = 6. If BC is extended through C to D and BC = CD, find AD. How many ordered pairs (x,y) of positive integers are there such that both # and y are less than 100 and the expression logiot + logicy has an integral value? In triangle ABC, sin?A + sin?B of angle C. Find the degree-measure ‘The number W is represented by the base q numeral 1441. When divided by eleven, leaves a remainder of 1. If 1 1 and for all 2 >0, loge = 3 log.z, write an equation expressing m explicitly in terms of n. In trapezoid ABCD, AB = BC = CD = 6 and AD = 8. Points P and R are chosen on line AD (on opposite sides of point A), so that there exists a point Q on the plane for which B is the midpoint of Pd, C is the midpoint of RQ, and BQ = QC. Find PQ. The sequence {a,} is defined by setting a: = 7 and, for 7 > 1, taking a, to be the sum of the digits in the decimal representation 2 Ry of (a;_,)?- Find aise. Let be the maximum nunber of points that can be arranged in a plane so that of any four of them, there are three that determine an equilateral triangle. Find n. Find all values of a such that the three equations ¢ = + y are satisfied simultaneously by some ordered pair (#,y). Points M and_W are on side AC of triangle ABC, and points P and Q are on side AB. ‘The lines MP, W@, and BC are parallel, and they divide the triangle into three regions of equal area. If N= 4, find BC. In triangle ABC, m angle ACB = 90 and m angle ABC = 45. Points X,Y, Z are on sides AC, CB, FA, respectively, so that AX:xC = cYiYB = B2:24 = 2:1. If 0% intersects XY in P, find the degree- measure of angle (PY. ‘The symbol [2] represents the largest integer not exceeding =. Find all positive integral values of for which the expression (4) represents a prime number (1 is not prime). A sequence of digits has the property that each pair of successive digits, taken in the order written, forms a decimal numeral representing a multiple of either 17 or 23. If the first digit is 9 and the sequence is finite, what is its last (possible) digit? In parallelogram ABCD, AB = 4, AD = 9, and m angle BAD = 30. The circle through points A, B, and D intersects BC at B and X. Find x¢. A right circular cone has a base with radius 12. A plane parallel to the plane of the base cuts the cone into two equal volumes. Find, in radical form, the radius of the circle of intersection of this plane with the cone. -48- 28. 29. 30. Problems—Spring 1984 If [2] denotes the greatest integer not exceeding @ and {2} z- [2], find all ordered triples (z,y,2) of real numbers such thats +[y] + (a) = 11, y+ [2] + le} = 2.2, 2+ [a] + &) 3.3 1£0< =< and 20% cos 2. 8°!" ©, find the numerical value of Acute triangle ABC is inscribed ina circle. Altitudes Wf and Gi are extendet to meet the circle again at P and Q, respectively. Tf PQ:AC = 7:2, find the numerical value of sin B. Problems—Spring 1984 ry Instead of finding twice the square of a number, a student found twice the square root of that number and got 10 as her answer. What is the correct answer? Dianeter 4B of a circle whose_center is 0 is extended past 4 to point P, and tangent segnent PP is drawn. On AO as diameter, a small semicircle is drawn on the same side of 4B as point 7.” If, OF intersects the smaller semicircle at X and angle TPB = 40°, find the degree measure of arc Of. In triangle ABC, AC = 6, BC = 8, and AB = 10. Squares ACKY and BCWZ are drawn exterior to the triangle (the triangle has no interior point in common with either of the squares). Find the distance between the centers of the two squares. Tf a = logia(VIS + V3), express in terms of a the value of Logio (VIS - 73). The nth term, a, of a sequence of nunbers is given by a = 1 and a@=@ +n forn> i. Wri ion expressing a, as 4,74, + 2n for n jrite an equation expressing a, as a polynomial in n. In parallelogram PQRS, point X is on PQ and Px:PQ = 1:3. Point ¥ is on PS and PY:PS = 1:4. The line XY intersects diagonal PR in 2, Find the numerical value of the ratio P2:PR. ‘The number 7 has three digits when written in base ten notation. Its cube root is the sum of its three digits. Find w. A train leaves the Jerome Avenue subway terminal every 12 minutes. ‘These trains travel down the Jerome branch line, then down the Lexington Avenue trunk line. A train leaves the White Plains Road terminal every 6 minutes. These trains travel down the White Plains branch line, then also join the Lexington Avenue trunk line. All the trains are scheduled so that there are constant intervals between them as they run down Lexington Avenue. How many minutes are there in this interval? -49- Problems—Spring 1984 10. lL. 12. 13. 14, 15. 16. 7 18. 19. Find the largest integer smaller than logy9 + logs28. In equilateral triangle ABC, AB = 12. One vertex of a square is at the midpoint of AB, and the two adjacent vertices are on the other two sides of the triangle. The Tength of a side of the square may be expressed as pv? + q/6 where p and q are rational numbers. Find the ordered pair (p,q). 1f0 << Zand cos a+ sing = 3 7 > find the numerical value of cos 2 - sin x. Find the least positive remainder when the integer 2° + 0? + 2! + + 22 4 22 4 2% + 3 + 2h 4 ar + 2199's 100 is divided by 8. ‘The number W is represented by a three-digit base ten nuneral. Wis equal to the cube of its units digit, and is also equal to the square of a two-digit numeral formed by its other two digits. Find 1. In triangle ACB, AC = 7 and BC = 5. Squares ACKY and BCW2 are drawn exterior to the triangle (the triangle has no interior point in common with either square). Find the numerical value of AB? + aw? Find the sum of the seventeenth powers of the 17 roots of the equation «'” - 3r+1=0. A train leaves Main Street for Times Square every 15 minutes, starting at 8 A.M. A train also leaves Times Square for Main Street on the same schedule. All trains run at a constant speed along the same tracks, and the trip either way takes 90 minutes. Including the trains it meets at each terminal, how many trains will the one leaving Main Street at noon encounter on its trip to Times Square? Altitude GD to hypotenuse AB of right_triangle AB@_is a diameter of circle 0. This circle intersects AC in £ and BO in P. If AC = 9 and BC = 12, find BF. Find the degree-measure of all angles © such that 0° < x < 180° and costs - sinter + (Sin"2s) (cos 28) 0. AB and CA are decimal numerals, and 4, 2, C are distinct digits. Tf four times 4B equals CA, find the ordered triple (4,3,0). -50- 20. 2. 22. 23. 24, 25. 26. 27. 28. 29. Problems—Spring 1984 ‘Two cardboard rectangles each have dimensions 2 and 8. They are placed on a table so that one pair of diagonally opposite corners coincides, but the other pair of diagonally opposite corners does not coincide. Find the area of the region in which the rectangles overlap. The numbers @ and b are both perfect squares, and are both represented by four-digit decimal numerals. The digits of these two numerals are the same but in the reverse order. If the square root of a divides the square root of b, find a. In trapezoid ABCD, the ratio of base AB to base CD is 2:3. Diagonals AC and BD intersect in x, and the line through X parallel to AB intersects 4D at P, Find the ratio of the area of triangle PAX to the area of triangle ABD. From a point P on the hyperbola with equation x” - y? = 9 - dy, tangents are drawn to the circle centered at the origin with radius 1. What is the minimal possible length of these tangents? Walking along a bus line, a student found that a bus caught up with her (going in the same direction) every 12 minutes and a bus passed her (going in the opposite direction) every 4 minutes. The student travels at a constant rate, and all buses travel at the same constant rate as one another. The buses leave their terminals at equally spaced intervals of time. How long, in minutes, is each such interval? A boy has as many brothers as he has sisters. His sister has twice as many brothers as she has sisters. How many children are in this family? Radii GA and OB_of circle 0 meet at an angle of 120°. A square has one vertex on Gd, another on OB, and two more on minor arc AB. If OA = 13, the area of the square can be written as p + qV3, where p and q are rational. Find the ordered pair (p,q). Tf ry, 2, «++, My are the roots of 2x7 - 2° - Sx° + 172" - 41927 = 3722 - 10 = 0, find (1 + )(1 + %2)-+-(1 + 17). Find all ordered pairs (2,3) of positive integers satisfying o-2 eed One hundred pennies are arranged in seven stacks, of which no two stacks contain the same number of pennies. A student counts the number of pennies in each stack and takes 50 pennies in such a way as to disturb the fewest number of stacks. He ends up taking pennies from stacks. For all such arrangements of pennies, what is the largest possible value of i that will be necessary? -51- Problems—Fall 1984 30. A right angle has its vertex at the centroid of an equilateral triangle of side 1 unit. Find the maximum possible area interior to both the angle and the triangle. Problems—Fall 1984 7 In square ABCD, points P, @, R, S are chosen on sides #8, BC, TD, (alt) DA, respectively, so that AP:PB = BQ:@C = CR:RD = DS:SA = 1:3. Find the ratio of the area of PORS to that of ABCD. 2. A student guesses at random at three true-false questions. What is (628) the probability that she gets at least two correct answers? 3. In rectangle ABCD, AB = 6 and diagonal BD = 10. Circle 0 (with (518) center 0) is inscribed in triangle CBD, and circle P (with center P) is inscribed in triangle BAD. Find OP. 4. The cube of a certain integer has a decimal representation (05%) consisting of ten digits, of which the two leftmost, as well as the rightmost, are the digit 7. Find the integer whose cube has this form. 5. Find all ordered pairs (x,y) of real numbers such that 3° “**Y = 1 (248) and 2 logsz = loga(y + 3). 6. Find the numerical value of cos 15°(sin 75° + cos 45°) (25%) + sin 15°(cos 75° - sin 45°). 7 For all real nonzero numbers, f(x) = 1 - 4 and g(z) = 1 - (898) = If h(e) = flg(#)] for what value of x does h(x) = 8? cos 15° + sin 15° 8. nd the numerical value of S95 13+ Sin 13) tiay) cos 15° ~ sin 5 9. Side BC of triangle ABC is extended through C to X so that BC = CX. (28%) Similarly, side CH is extended through 4 to Y so that CA = AY, and side AB is extended through B to Z so that AB = BZ. Find the ratio of the area of triangle XYZ to that of triangle ABC. 10. Find the value of ¢ for which the roots of z* - 6x? - 24m +e = 0 (21%) form an arithmetic progression. ue Three ferryboats start at a terminal at noon and go to different (43%) destinations. Ferryboat 4 reaches its destination after 20 minutes, boat B after 15 minutes, and boat ¢ after 32 minutes. Upon reaching their destinations, the boats return to the terminal, then make another trip, and so on. The trip back to the terminal in each case is the same length, and takes the same time, as the trip out. What is the least number of hours after which the three ferries will again dock at the terminal simultaneously? -52- 12. (228) 13. (568) 16. (09%) 17. (438) 18. (02%) 19. a7) 20. (358) au. a7) 22. (148) 23. (27%) Problems-Fall 1984 In right triangle ABC, leg AC = sin @ and leg BC = cos 8. Find the length of the longer leg if the length of the median to the hypotenuse AB is tan 0. Points M, N, and P are the respective midpoints of sides 7B, 30, and CA of triangle ABC. A point X is chosen outside of the plane of triangle ABC. Points D, E, are chosen such that , l, and P are respective midpoints of XD, X£, and XF. Find the ratio of the area of triangle DEF to that of triangle ABC. For all real numbers =, the function f(a) satisfies 2f(#) + f(1 - 2) =a*, Find f(5). In triangle ABC, AB = 5 and AC = 8. Point P is on BC and BP:PC = 3:5. Find the ratio of the radius of the circle through 4, B and P to the radius of the circle through A, C, and P. If @ and y are real numbers, with ¢ > y and sy = 1, find the ninimun possible value for =i. If [2] denotes the "greatest integer" function, find the largest prime number p such that (S]- p for sone integer n. Square ABCD has area 1 square unit. Point P is $ units from its center. Set § is the set of points that can be obtained by rotating point P 90° counterclockwise about some point on or inside the square. Find the area of set S. Circle 0 passes through vertex D of square ABCD, and is tangent to sides AB and BC. If AB = 1, the radius of circle 0 can be expressed as p + q/2. Find the ordered pair of rational numbers a). Find all ordered pairs (x,y) of real numbers for which a+ cy +a = 14 and y? + ay +y = 28. In a rectangular coordinate system, a tangent from the point (24,7) to the circle whose equation is z’ + y* = 400 has point of tangency (a,b) where b> 0. Find a. Angle ABC is a right angle and CB = 1._,D is a point on ray BC such that DB = 3 and E is the point on ray BA such that m angle DEC is maximum. Find the distance BE An ordinary pack of playing cards is shuffled, and two cards are dealt face up. Find the probability that at least one of these is a spade. Problems—Fall 1984 24. ( 58) 2s. (62%) 26. (65%) 27. (76%) 28. (448) 29. (268) 30. (ass) In convex quadrilateral PQRS, diagonals PR and @5 intersect at 7, with PP:?R = $:4 and QP:75 = 2:5. Point ¥ is chosen between 7 and S so that QP = TX, and RY is extended its own length to ¥. If point ¥ is outside the quadrilateral, find the ratio of the area of triangle PSY to that of triangle QRT. Point P is chosen along leg BC of right triangle ABC so that BP = PA. If leg BC = 10 and leg AC = 4, find BP. Five identical black socks and five identical brown socks are in a drawer. Two socks are picked at random. Find the probability that the two socks picked will match. The roots of f(z) = 0 are 2, 3, 7, 5, and 9. The roots of g(x) = 0 are 3, 5, 7, 8, and Find all solutions of the equation fe A set of distinct, nonzero real numbers is placed along the circumference of a circle. Each of the numbers is equal to the product of the two numbers adjacent to it. What is the least possible nunber of numbers in the set. Im equilateral triangle ABC of edge length 1, D is on BC so that angle DAC = 45°, Find the area of triangle DAC. A regular 11-gon is inscribed in a circle. How many triangles are there whose three vertices are all vertices of the 11-gon and whose interiors contain the center of the circle? 54 Answers ANSWERS Spring 1975 ag Semon 6. 0 18, 45, 90, 99 8. (2,8) 9. om 10. (3,7) n. 28 RR. -14% 1B. -1 uu. as. & 6. a=b+4 17. 5 1B. 19. {ele < -3) 20. {ele>3or0 2. Rearranging terms in the original equation, we have: Vet =H - @ = WEST. Squaring both sides, 2x? - 4 - 20VEE—H = 40? - 4 so -2/e?—4 = x. Squaring again, x” - 4n* = x", so 42? = 0 and 2 = 0. But, since |z| > 2, = 0 is an extraneous root, so there are no real solutions. 1975 2 The sum of the digits in any multiple of 9 is itself a multiple of 9. Hence, A + B must be a multiple of 9. The possibilities for AB are thus 18, 27, 36, 45, 54, 63, 72, 81, 90, 99. By trial and error, the only solutions are 18, 45, 90, 99. 8. The closest point, M, is on line OP. Since OP = VG- C2)e + (S11)? = VB + GF = 10, and y= 5, OM = d0P, so Mis the midpoint of OP and M is the point (2,8). 9. The quadrifateral formed by connecting the centers is a square. It follows that av2 + a =r, so a(v2 + 1) = », r(Z- 1 = mV2- 0, and 10. (10 + Vea = Va + VB, where a and_b are positive integers. ‘Square both sides to get 10 + 2/21 = a+b + 2¥ab. Equate yational and irrational parts to get_a + b = 10 and ab = 21, Solve to obtain (a,b) = (7,3) or (3,7). Since a < by (a,b) = (3,7). For more such problems, see Chrystal (pp. 207-210), or Hall and Knight, Elementary Algebra (pp. 281-284) . 67- Sol ul. 12. 13. ions-—Spring 1975 Method 1: PORS is a square. (This is easy to prove, for example, using Similar triangles.) By the Pythagorean theorem in triangle DEA, DE = Vig + 144 = 4V10. In similar triangles PAS and ADE, the ratio of similitude is 2-4-1. merefore, aio 0 1 22 6 . 6 PE 4y= 2/70, PA = S10, sp = pa = S10, ana vio $ 5 5 PS = DE - PE - Sp = avI0 - 2/10 - $15 = *2ViD. Hence, = (22 y7o)p = M4 aa) = 288 Areapoas = (32 V0)? = Hci) = 288, B4F Pie > Method 2 Method 2: Area, = 4 boxes. The total unshaded area is 12 boxes, 'PQRS so the unshaded area in ABCD is 6 boxes, and the total area of ABCD : 4 = A ergay = 288 $10 boxes. Thus, Angas = io(Agscp) = pyClde) = 788 - Let 2 = 2 ‘Then, assuming convergence, we have 2 = yorz? +2 =1 1 Tee Tests 22-140, 22-1472, Since s> 0, = -14+ 2 or /Z- 1, For a discussion of convergence of this type of continued fraction, see Niven and Zuckerman (Chapter 7) or Khinchin or Olds. oe = a? +1_2°(@-1)-@-D go oetea 1 esis @- 1 ~@-N@-)_ @-'@+ Ly Se ees . The only value of « that makes the numerator, but not the denominator, 0 is # = -1. es We have: ~68- 14. 15. 16. 7 Solutions—Spring 1975 Regrouping, we have: (@@+D@+DweNw@+ ayer =(@+ Det N+ Ves} +1 = [ (a? + Sz + 6) (a? + 5x4 4)] + 1 (y+ 9) + 4) +1, where y= 2? + Sx = yh + loy + 25 y+ 5) (a? + Sx + 5)*, so that (4,8,0) = (1,5,5) Note that the product of any four consecutive integers, when increased by one, results in a perfect square. Drawing radius Of, OFLAT, so AT = 4. Using the Pythagorean theorem in triangle OP%, we have 9+ 4- a)? = (3+2)%, and 2 = Since @ > 0 and b > 0, we know that /@5 # 0 and a/b = JavaB and bia = ¥5/ab. The given equation is thus equivalent to yab(Ja + JB) _ yab(va - VB) op vab(va - vB) vab(Va + vB) or Ya+ JB Ja- We. (a+ vB)? - Va- Vo)? We oe va-vb va+vb a-b a-b ; 4 aE ivi s 5 re, Since va # 0, dividing by /aB produces "= 1. Therefor 4sa-banda=be4, Be Ey. sor? = wr + Ww. therefore, £? - WE - W* = 0 and + JRE WE +) be EEF nee > 0, son = EAD, ye are given that 5905 - 1) W IW = 50(V5 - 1), sol = . Equating these two WUS + 1) . SOS - 1) 7 w . expressions for L yields Therefore, ~69- a7. 18. 19. 20. tions—Spring 1975 (Continued) We ee Rationalizing the denominator, W* = 25(/5 - 1)?, 5+ so W = 5(V5 - 1) and k= S. This sort of rectangle is a "golden" rectangle. For more on these, see Coxeter, Posanentier, Excursions (Chapter 9), or Huntley. First note that a #0, b # 0, anda #b. By the. factor theorem (1.311), ¢ is a root of both equations, so: (*) q? + aq + be = q? + bq + ae = 0, aq - bq = ae - be, (a -b)q = (a- b)e. Since a#b,q =e, Substitution in (*) yields c? + ae + be = 0 or e(e +a +b) = 0, whereupon ¢ = 0 or c= -a-b. Since c # 0, This problem can be done purely algebraically, but it is easier to remenber that absolute value may denote distance on the number line. ‘Thus, the equation |z| - |e + 3| = 3 means that the distance from # to 0 is 3 more than the distance from x to -3. This happens whenever « is to the "left" of -3 on the number line, or # < -3. Note that # > 0 for VE to be meaningful and # 0,1 for the denominators to be nonzero. Rationalizing denominators and adding 2) or ‘ing fracti the resulting fractions yields aeons x Since t #0, 27 < 1. Ife>1,2<2-lore>3. If0<2<1,2>2-lorz<3,s00 3}. Method Use coordinate geometry, let 4(0,0), 3(12,0), & zie 4% -2 X(@,0). Tt follows that 15 + G) ana 2(6 +5, 6 3) since ¥ is the midpoint of AG and Z is the midpoint of JF. The coordinates of the midpoint of YZ are then w(s +3, 3) that the y-coordinate of W is always 3. Since 0 < = < 12, W moves Note along a horizontal line from (3,3) to (9,3); this is a path of length 6. yk a) rb flag Law A eay RBM Method 1 Method 2 -70- 2. 22. 23. 24. Solutions—Spring 1975 (Continued) Method 2: Since triangles AYX, X2B are right-angled, we can extend YZ = XT, and W is their common midpoint. Thus the locus of W is the locus of midpoints of the segments 1X as X varies from A to B. This is easily seen to be a line segment parallel to 4B and equal to 445. One way to prove this final result is by using honothecy. See, for example, Coxeter and Greitzer, or Altshiller-Court. a+b ate This equation has the form where a = 30° - 2x7, ‘ a-b a-c aa bex+, and c= Se - 13. By 1.12 it is equivalent to $= 2. Hence a= 0 or b= c, Thus 30? - 22? = 2*(3¢ - 2) = 0 or e+1= 52-13 Considering both cases we obtain x = 0, 2, Ea Z. Method 1: Consider the special case where triangle ABC is equi- Tateral, so D coincides with x and # with Y. Then triangle AXY ~ triangle ABC and XY = 3BC, Area, ‘triangle AXY and Areayy,, = 5 Area, = thre, wiangle ABC yee ~ 7 ATC yi angie ABC TS EIN Bs a a » 8 Method 1 Method 2 Method 2: (This solution is general, and points D and H need not be interior to ABC.) Extend AE and AD to meet BC in F and S, respectively. Then triangle AEC ® triangle REC (by A.S.A.),_ so 4B = ER, $jmilarly, fyiangle ADB = triangle SDB, so AD = DS. Hence DE||RS, making Dé bisect 4B and AC. Therefore, Aree rangle axy = APES + 80 ATCA yy 3 = F Area, triangle ABC’ triangle ABC’ Method 1: Adding 2? to both sides, we get e* - dz? + 62% - de + 1 = a’, or ( - 1)" = «*, so (e - 1)? = +z, Solving these two quadratic equations for real # yields only two solutions, nanely, 345 -n- Solutions—Spring 1975 24, 2s, 26. 27. (Continued) Method 2 a we can divide through by a7, to got 2? - dees -$ 6 0. Ifwe lety=ae ye -2-ays5Say? - 4 - 250% +3, and we find that +320. Thus y= Sory-1. Substituting and solving for x, we find, as before, the real solutions 34% The substitutions worked because the coefficients of the given equation were symmetric (from the left and right terms in towards the middle). Hence if r is a root, + is also a root. The substitution y = 2 ++ "ties" the roots together in pairs, halving the degree of the equation. For more of this sort of technique, see Larsen (problem 1.3.2). Method 1: Let 304B5 = 225n = 3? *5* +m, Since 30ABS is a multiple of 9, the sum of the digits, 8 + A+B, is a multiple of 9. Since each digit is at most 9, it follows that 8 +4 +B = 9 or 18, sod +B = Lor 10. Since 30485 is also an odd multiple of 25, B must be either 2 or 7. Therefore, A + 8 = 10 and either A = 8 (when B = 2) 30825 _ - - 30375 Soe = 137, or A = 3 (when B= 7) son = S57 = p55, making n Method 2: Since 30,000 < 225n < 31,000, it follows that 134 <7 < 137. Since 30485 is odd, » must be odd, so the only possible values for n are 135 or 137. a= S16 = 14; b = V.0639 < V.064 = .4, so b is slightly less than +43 ¢ = V00aT = V4.1 x 10°* = Y4100 x 20°° = VaT00 x 107 > VG096 x 10°’ = 4x 10°? = 0.4, soe is slightly greater than .4. (Note: The suspicion that 4100 was close to 4° motivated computing 4® = (4°)* = 4096.) Lastly, d= (.2)? = .04. Thus, in increasing order, we have d, b, a, c. Using mass points, assign weights of 3 to @ and 4 to A. Then there must be a weight of 2 at B, and hence a weight of 5 at 4". Therefore, iy = 5:4 where « = AP and y = PA". (See Appendix B.) g® -72- 28. 29. Solutions—Spring 1975 Method 1: Let £ be the line perpendicular to m: 3x + 4y = 12 at Q(0,3). The slope of 2 is 2, so its equation is y fe +3. We are looking for points P(a, $ a+) that are on & such that PQ = 2. By the distance formula, (79)? = a +( $a)! = 2%, so 8 =&, of&, 23) is on line m: 30 + k a $+ mena = $, AS, B) line my: 30 + 4y = ki, 6 : so ky = 22. When a ») is on ma: Se + ay = key s0 ke = 2. y NN N SS Xe Method 1 * Method 2 ~\€ Nethod 2: Draw line & perpendicular to line m and passing through 0, Triangle OAC ~ triangle BAO, which is a 3-4-5 triangle. AB 35 2 ‘therefore 24 BB or =F, so0c= 2. We are given that cot = 2, sooct = 242-22. the triangles 0410", Osc are OA". OA OA". 3 oy og : sintiar, so B= SF. Hence Bt = 5 and 04 . Thus, 3S at (0. AL) ties onm: 324 4y = ky so ky = 22. Similarly, in 2 2 oaner, om = 22 2 = Zand oA" = 3. Thus, A"(0,3) Lies on ma: 30+ dy = ke, so ky = 2. Using the special relationships in 30° - 60° - 90° triangle MEP and in isosceles right triangles CPE and CMB, we have cM = + of nin = 12, so 2(1 + V3) = 24, and 2 = 24. Rationalizing the VBed denominator, 2 = 12(V3 - 1) so k = 12. -73- Solutions—Fall 1975 29. 30. (Continued) Let «, y, and z be the roots of the equation g? - kig? + kag - ks = 0. Then x? - ky? + koe - %y = 0, y® - kiy? + key - ks = 0, 2? - kiz? + kez - ky = 0. Adding these equations gives a - kya? + koa - 3k3 = 0. By the conditions of the problem, and 1.4, ki = a, and we need to find ks. Hence we need only express ke in terms of a, To do this, note that a® = (x + y + 2)” sats y? + a? 4 Day + ys + ma) = a? + 2ke. Hence ke = 0 and ks = 24. the trick of forming an equation with roots 2, y, and 2 was motivated by noting that the functions given in the problem are all symmetric in z, y, and s. For many more such problems, see Krechmar (section 5, problems 16-39}. Solutions—Fall 1975 iy roca <12.5,[ = 0, so s(e) = 2, tea >i2s, [722] - 7, 50 r@ = (rés]- Then, if m is a positive integer and 12,5 < # < 12.5(n + 1), fe When 87.5 < # < 100, f(@) = -7, so the range of f contains 8 values, namely, 0, -1, -2, ..., -7. (See 5.21 and 5.22.) Note that all digits are less than 7. We have 814 + 93 + ¢ = 49C + 7B + A, so 804 + 2B - 48C = 0 and B = 8(30 - 54). Since B is a multiple of 8 and B <7, B= 0. Therefore, 30 = SA. Since C <7, C = 5 and A= 3, The number is 3053 = 5037 = 248 in base ten notation. -74- Solutions—Fall 1975 A careful graph shows that the six points (-1,-4), (0,-3), (1,-2), (24-1) 4 §3,0), (451) on the line y = 2 - 3 are also inside the circle 2* + y* = 25. ‘The points (-2,-5) and (5,2) are outside the circle. To check this algebraically, we can write «* + (@ - 3)? 2c? - 6x + 9 < 25, or a - Sr - 8 <0. Since the coefficient of x? is positive, the expression takes on negative values between the two roots (which are real). These roots are $+ 41 op approximately 4.7 and -1.7. ‘The possible integral values for « are thus -1, 0, 1, 2, 3, 45 and the y-values can be found by substitution. Yay Cr aae 2 x Ne have: | number of cost per "people ‘person total cost originally |» ® o 10 drop out | m= 10 ee. @-1@+n 15 more . . . drop out n- 25 | e+ (m - 25) (x + 3) of m - 10)(@ + 1) = me The total cost is always the same, sof (7 De) | 3) 2 ne 10 8 n= low ‘ ‘ or ie: 2 REE JE. Solving simultaneously (by eliminating 2), we find that » = 100. By the angle bisector theoren (3.31), ¢= so=3and8-@=5. 6 = 48 - jo’ 10x = 48 - 6x, rea, u arorrcre aoe triangle ABD ee iene Note that if we use the fact that the ratio of the areas of triangles with the same altitude is equal to the ratio of their bases, we need not even solve explicitly for DB: . ei s 3 Area, _ DB ‘triangle ABD ~ cB” “triangle ACB -75- Solutions—Fall 1975 5 (Continued) C XD ax 8 We have x? + 2px* - px + 10 = 0. By the rational root theorem (1.7), the only possible rational roots are -10, -5, -2, -1, 1, 2, 5, 10. The only sequences in arithmetic progression are -5, -2, land -1, 2, 5. Since the product of the roots is -10 (see 1.4), the roots must be -1, 2, 5. ‘The sum of the roots is -2p = 6, sop = -3. a ‘The train travels 300 + © meters in 20 seconds. Since the light is above the train for 10 seconds, the train travels # meters in 10 seconds, so it would travel 2x meters in 20 seconds. Thus, 300 + @ = 2x and x = 300, 8. If a, b, ¢ are the three roots of «’ + 3m + 14 = 0, then (see 1.311), 2° + 30 + 14 = (@ - a)(e - B)(@- e) = 2 - (a tb + oa? + (ab + ae + be)e - abe, This a+b+e= 0, ab + ac+ be = 3, , L,l,llbevac+ad 3 | 3 abe = -14 (See 1.4.) Hence t+ P+ 2 = 230 3-74. 9 Since the trapezoid is a cyclic quadrilateral, it must be isosceles (see 3.92), Method 1: “By Ptoleny's theorem (3-91) 200 + 89 = 2, soz = 17. A B A 8 NOONE [NO YON 7 Se Method 1 Method 2 twice, i + 228 = 2, 1? + 25 = 89. Subtracting, 200 = «* - 89, sox = 17, ~76- 10. i. 12. 13. Solutions—Fall 1975 ste sfesDeat ed, so ste ses 110, 2-32-1000. ‘The rational root theorem (1.7) suggests possible rational roots #1, £2, #5, #10, +11, #22, #55, +110, Substitution shows that 5 is a root, so z - 5 is a factor (see 1.311). (2 - 5) (2? + Sz + 22) St 3i/7 3 WT BESET | 50 (a,b) = 5.0), (-$. 2) Using the recursive formula and the given initial values, we find a, = 7 and as = 9, and can conjecture that a, = 2n- 1. ‘This is easily proved by induction: we've already established the first few cases. If the conjecture is true for all integers less than n, ata +4 -a in Snr * Sper ~ Sana = (2m = 3) + (2n- 5) - (2H - 7) =m-1, Thus, we are looking for the sum of the first 30 odd integers, which is 30? = 900, (Or, use formula 1.512.) We use mass points (see Appendix B), placing weights of 3 at F, 2at A, 1 at B, and 4 at F. For balance, there must be weights of 2 at Cand 3 at D. Therefore, a, @p L>—~ \ o F@ c® When the sum of the ages of father and daughter is 132, the daughter will be +$2 = 44 and the father will be 88. Thus we will always have f= d+ 44, Since f+d+m= 73, d+ 44+d+m= 73, som = 29 - 2d, (Equation required) Us -77- Solutions—Fall 1975 vas. Mu. We use the Pythagorean theorem, In triangle pd, AD = BC In triangle REC, 16 + (12 - 2)? = a, In triangle RFB, 25 + 2* = 145 - a®. Adding these equations, 2x? - 24m + 195 = 145, a? - 12 + 20 = 0 (@ - 2)(# - 10) = 0 soa = 2, 10. pala o¢ ates = § 1s. By Ptolemy's theorem (3.91), 3a + 6a = at, 9a = ar, soz More generally, if equilateral triangle ABC is inscribed in circle 0 and P'is any point on minor arc AB, PC = PA + PB. > 16. Consider the limit ,: = a, where the dots indicate endless x 1 : ‘ ‘ a a exponentiation. This is equivalent to 2% = a, whence = = a”, 4 3 4, 3.4 -(4yt In particular, when a= 3, 2° =3, soa = (3) =X, Therefore, the Limit of the given sequence is a= 4, s0 a1,oe0,s40 46 very close to 1.333. For a formal treatment of such “exponential towers" and the validity of the above sort of argument see Knoebel. -78- 17. 18. 19. 20. a. Solutions—Fall 1975 In isosceles triangle BCD, the measure of angle BCD = the measure of angle BDC = 70, so the measure of angle ACD = 90 - 70 = 20. Eventually you want to leave your opponent with 1 toothpick. Your strategy is to leave him with 8 + 1 toothpicks at each turn. (Practically speaking, if he takes « toothpicks, you take 8 - = toothpicks.) Initially, you take 7 toothpicks, leaving him with 1000 - 7 = 8(125) - 7 = 8(124) +1 toothpicks. For a fuller discussion, see Kasner and Newman (p. 172). a? - 30? - 2+ 3 = e%(@- 3) - (@ - 3) = @ - 3)@"* - 1), 50 12(e? - de +3) 2 - N@-3) 22 Boge aes @- Det De-) FT ife# 1,3. 1234, 0, 2, 5, 11 The expression =**— is a positive integer if «+ 1 6, 12orz=0, 1, 2, 3,5, 1. Bute #1, 3, sow are the only possibilities. The minimum (m) occurs when A is folded to land at the midpoint of BC, som= + 8= 4. The maximum (M) occurs when A is folded to coincide with B (or C), so M = 4¥3 and (Mm) = (473,4). ‘The area of pentagon ABCDE is $+ 2+ 3+ 6(8 + 2) = 40. The equation of Dé is y = 8 - x, so the coordinates of P are (k, 8 - K). Then the area of trapezoid AQPE is 3X(8 + 8 - k) = 20, so K ~ 16k + 40 = 0 and k = 8 + 2¥6, Since kK < 8, k = 8 - 276 and (a,b) = (8,-2). Solutions—Fall 1975 22. 23. 24. 25. Method 1: Let a, b be the rates of 4, B, respectively. Distance ‘equals rate multiplied by time, so PX = a miles and PY = b miles. Had they exchanged destinations, we would have (since ? 3) . R bla, 38 1,42 24 pe bag. 8, Revesby and 12 + 7 = 12 ; a Lett ae etting r= 2, = (Gr #4) (ar - 3) = 0. Since ¥ > 0, we have » = 3 Method 2: Let rb, b be the rates of A, 3, respectively. The distances are now PX = rb, PY = b, so, when they exchange destinations, the tine equation becones = r+ i. As above, : c aac It is not hard to see that, in decimal notation, the integers a and «° have the same last digit. Here, if 2° = 656356768, the last digit of x must be 8. Next observe that 10° < 2° <'107°, so 10 << 100. Furthermore, x° = 6563.56768 x 10°, 5° = 3125, and 6° = 7776, 30 50 < x < 60 and x = 58. To see that x* and have the same units digit, we can use trial and error. Since we need to keep track only of the units digits, there are only ten cases. Ait = 33. Using mass points (Appendix 8), put weights of 1 at A, 2 at C, and 2 at B. This gives a weight of 4 at M, so AU:UM = 4:1, 1 1 3 ~3 MU = = (a) = 3(3V3) = 2 V3, and k= =. 2 yp Tf the unit fractions are Zp and py , then their sum is SFP ‘The denominator of the sum is always a square. We need the numerator to be a square as well; that is, we need a® + b* = o for some integer c. But this is just the requirement that (a,b,c) be a Pythagorean triple (see 2.1). The largest possible fractions will result from the smallest possible (a,b), which is (3,4). zi 1 1 The fractions are and ye. -80- 26. 27. 28. Solutions—Fali1975 Method 1: (w+ y)9 =a? + y+ 3y(e +), soa? = b + 3axy and ry = Sb Now a? + y? = (x+y)? - 2xy, 2 _ 23 - b) 2 2. a+ 2b so ee or at ey - SR, Method 2: Let d= a? + y?, Factoring, 6 = 2? + y? = @ tye - ay ty) sad - ay), sobed- ay, somea-2, Also, a? = (2 + y)? = d+ 2ey, soa? d+ 2(4 - 2 saat se made ey ef required) 7 a 110. 5,2 Letting n = JE Jy (n> 0), we haven+i=W=5+h, son=dorn=3,. therefore, « - 2y= Zora - 2y = 95 1 ; 1 sons 2y+horz= 2y +9 and (a,b) = (2, $) or (2,9). Method 1: When A is folded to D, triangle AXY lands in position Triangle DxY, so triangle DXY = triangle AXY. We use the law of cosines three times: In triangle CYD, a” = 12? + (15 - a)* = 2+ 12018 - a3) soa = 44. Im triangle Bx, b? = 3? + (15 - b)? - 2+ 3(15 - b)(4) sob = 7, Then, in triangle AXY, a? = q? + b? - 2ab(3) = (BY + (Bin conte r(Se1-$ ze Mw = ivh, and n = (49)(7) = 343. Method 1 Method 2 ~81- Solutions—Spring 1976 28. (Continued) Method 2: As before, triangle DXY = triangle AXY. Analyzing b the angles as shown, triangle BXD ~ triangie CDY, so at ~ ab = cna - * = yg and 18a - ab = 12b. Also Z= 7 S—, so 1Sb - ab = 3a. (+) Subtracting, 15a - 15b = 12b - 3a, so 18a = 27b and a = stituting in (* - 3p =2b, so 32? Substituting in (*), 18b - 3 b* = 3b, sod b40,b=7anda Again, using the law of cosines in 2 triangle AXY, 2 = sa 4) 80m = 343. 29. The shaded area is 2(AreA.omicizele on diameter AD” © *senicircle on diameter AC) = 2(3 + lem - 3+ 4m) = 12m, sok = 12. 30. 10" = 10,000 and 20" = 160,000, so ABCDE is the fourth power of a two-digit number whose first digit is 1. Since4+C+B=B+D is a sufficient condition for divisibility by 11, aBCDE = 11° = 14641, so C= 6. Note that 11" is easily calculated using the binomial theorem (1.21) on (10 + 1)*. Solutions—Spring 1976 ny If the nth term is denoted by a,, we have = 7+ Since this ratio is constant, the series is geometric, and the sum to infinity is fof st 1 Z: (see 1.523.) -82- Solutions—Spring 1976 We apply two angle bisector theorens: 2 =P and t? = ab - ec go Band t* = ab - pq, (See 3.51, 3.32.) Let AB = k, AC = 1, BC = x. Then BE = ke and CE = kVE. Now the second theorem gives kx = 2 - kz, so 2k? = 1 and k= 2. Note that the measures of angles C, A, and B are unimportant. c a * B k¢€ kK Using mass points (see Appendix B), we assign weights 2, 1, 2 to A, B, Cy respectively. ‘Then 24 + 15 = 3D, 24 + 2C = 4F, and 1B + 4P = 2¢ + 30 = 5G, Hence 22 = 4 A+ 50° +B + 507 +B + S0+A= 27 = (S00 +0)? = c? + 50? + 30° + 50° + 307 + 50 +07. Now C? < 50 (otherwise the right side would be a five-digit numeral). Thus A = C?. Then B + SO? + B+ 50 = 3c? + 50? + 3c® + 50 and 3c? < 50, Then C = 1 or 2 and B= 30? = 3 or 24. Using the law of cosines, (20 - b)? = (6 +1)? +b? + b(b+ 1, or 2b? + 43b - 399 = 0, or (2b + 57)(b - 7) = 0, and BC = 13. -ftl 2228 55 1082 - 204 + Let A = = » Then 4 Z° 70° 0 104’ 204 + 10 = (24+ 5)(54 +2) = 0, and a= Zor. rfa-2 Ste ash S=2+2, thene = 20rd 2 wi Ze2ed, tn Zor}. 1f4=2, « will be complex, since for real x, |e + LAT and k 226.1). In fact, 2 +4=2 reads to -83- Solutions—Spring 1976 10. Method 1: We have $fa + (a + - 1)} + 100 = Hla +n) + (@+n+n- 1)] (see 1.512), which simplifies to n® = 100, and n = 10, Method 2: Since each integer in the second sum is more than the Corresponding integer in the first sum, and since there are 7 integers in each sum, Sum + 100 = Sum, + ”*, from which n = 10. A rhombus is symmetric in its diagonals, Hence, lines 4@ and BD are lines of symmetry for the entire figure. Thus the opposite sides of quadrilateral @ are parallel, making it a parallelogram. Also, the diagonals of a rhombus are perpendicular, so that ACL BD. Since the pairs of sides of @ are parallel to AC and ED, the adjacent sides of @ are perpendicular, making it a rectangle. Wow the four squares drawn are congruent, so Ai = AI = BI = BJ (see diagram). Also, angle HAI = angle HAD + angle DAB + angle BAI = 45° + 30° + 45° = 120°. Similarly, it is easy to see that angle IBJ = 120°. Thus triangle HAI = triangle IBJ, and @ is in fact a square. It is not hard to see that HA = 3v2, and the law of cosines then shows that HI = 754, so the area of @ is 54. Since 625 = 25%, we first need to find all right triangles whose hypotenuse is 25. There are two, 7,24,25 and 15,20,25. This produces two solutions in the first octant and four solutions in the first quadrant: (7,24), (24,7), (15,20), (20,15). Altering of signs yields 16 solutions in Quadrant I - Quadrant IV. These, plus the four solutions on the axes, yield a total of 20. One may also proceed by using the relationship: 257 = [k(m? + n?)]? (see 2.1). Then (k,m,7) = (1,4,3) or (5,2,1) or (1,5,0) generate the 20 solutions. If we notice that the given number is slightly larger than 100°, it is not hard to see that it is, in fact, 101°. Since 101 is prime, the greatest proper divisor is 101° = 10201. -84- lu. 12. 13. 4. 15. Solutions—Spring 1976 The intersections are (5,4), (4,5), (-5,-4), and (-4,-5), forming a rectangle whose dimensions are V2 and yT62 and whose area is 18. If he bought = oranges at 3 for 16¢, he paid a total of Wea. 2 mal rofit, he must make 22° (+ ABs ae + b)e. to make 20% profit, he mst make 12% of this; that is to be equivalent to selling 3 oranges at £ ¢ cach. 120 (162, ade) _ | k ‘ . rus 120(1E 22) 0 sy E reading eo i = 9. Let the number of adults and the number of minors be represented Ae 2 so mm - m4 = a. by 4 and M, respectively. Then “= i 242 ‘ . Also, ¥ = 2 °T: leading to x = 12. Method 1: ‘The center of circle 0 lies on angle bisector AD = 5v3. 6._ Thus Tn 30-60-90 triangle A0G (see diagram), 0G sv3 - 6, the circle lies inside triangle ABC, and OD so (a,b) = (5,6). 3, so AO AD ~ AO Method 2: Once we see that 0 is inside the triangle, we can use 3.7. This theorem shows that 0G + OH + OD = S¥3, so that OD = S¥3 - 6. A B D ¢ Using mass points, we assign weights of 2 and 3 to B and C, respectively. Then 2B + 3C = 5D; now assign a weight of 6 to A to have the preper ratio of AE to ED. Then 6A + 3C = 9F, so BE EF 2" -25- Solutions—Spring 1976 eet eter ere 16. Wehavel+>-Z-stigtu-g@ Combining the terms in groups of two, 3 3 B.3,35.5, 2 28 ew S23-5+5-ie ame ST SF E+ (See 1.523.) a) 17. We have a® = c? - b? = b? + 2+ 1- b? = 2b +1. Let a = A*. Then A® = 2b + 1, which is an odd integer, so A is also an odd integer. For the least possible ¢, we need the least possible value of a, Taking A = 1 leads to b = 0. Taking A = 3 gives A® = 729, b = 364, and c = 365. 18. Since triangle Bé# = triangle FOE = triangle FCB, triangle BEF is equilateral. By the law of cosines, EB* = 100 + 48/3, and the area of triangle EBF = B00 + 4875) = 25/3 + 36, so (a,b) = (25,36). 19. We have 3(t + u) + 10t + u = 10u + 4, which simiplifies to u = 2¢. Solutions are 12, 24, 36, 48. 20. Method 1: Extend median BP to point D so that MP = PD. Then AMCD Will be a parallelogram (since AC and MD bisect each other). Now triangle AMD is a 6-8-10 triangle, and angle MAD is a right angle, so AMCD is a rectangle and AC = MD = 10. 8 4 c > /, > Method 2: Using formula (3.2) in triangle AMC, we have 1036 + 64) - 22, soc = 25 = 5(56 + 64) - Fe*, 10. -86- a. 22. 23. 24. Sok ions—Spring 1976 Using similar triangles: 4-y_3 ts eHeSsyen, yrs 4 pa’) z+1.3,,.2, ve, tegn e(2, aL hea) thus, (ey) =(2, 1). G aN . x (2 a3) + Subtracting the third equation from the second gives y = 2. Adding the sane equations gives e +2 = 6. Thus v= vz + vz - 72, or Fe TE: "Squaring and simplifying, “=e © 202, From which za= 8. but 2 ri= Gr Solving these to equations simulceneously, (@sys2) = (4, 2, 2) or (2, 2, 4). If the legs are a and ¢, we have a? +c? = (¢ + 1)? = 0? + 2¢ +1, Simplifying shows a? = 2c + 1. Thus we are looking for an odd square (a*) that is greater than 201. The least such square is 225, so a® = 225 and ¢ = 112. Method 1: Since the successive differences of the sequence b Form an arithmetic progression, the second order of differences (i.e., the differences between the differences) is constant, making 5, quadratic, Substituting several values into b, = px? + qx +r and solving the resulting three linear equations, we find sntsin- figher Algebra 2 by Sn = 1, See Hall and Knight, Higher Algebra (p. 322) or Spiegel, for a fuller discussion of these finite difference techniques. Method 2: (A telescoping sum): It is easy to see that a, 7 8+ 2(n- 1) = ant 4. Then b, - by = 4, = 2-1 +4 =2n-2+4 BDH) san 1) Gee 1.53.) Since by = 3, we again find b, =n? + 3n- 1. -87- Solutions—Spring 1976 2s. Let 2 =J10+/10 + /10... . Then a = 10+ fio + vo + lors, u Solving, and rejecting the negative root, we find 2 = L*V4T , and (a,b,c) = (1,41,2). For a discussion of the rather advanced topic of convergence of this sort of expression, see Rudin (P. 78 problem 3) or Larsen (problem 7.6.8). 26. When the graph of the graph of |x| + Tongs The area'= 3+ 4+ 4 book on analytic geometry. el » + |yl = 2 is centered about (0,1), we have 1-2) a'square whose diagonals are 4 units 8. See translation of axes in any 4 27. In right triangle Ai, HY = 3, Since triangle 4BM is isosceles, we have HE = HM = 3; and since AM is a median, BM = MC Hence from right triangle AHC, AC? = 4? + 9° 2 97, so AC = / c Byuam 6 28. Let the integers by , y, and = +12. Then (e+ 12)? - y? = y? - a, and 2? + 122 + 72 = y?, or (a + 6)? + 6? = y?, Since the only right triangle with a leg of 6 is the 6-8-10 right triangle, 2, y, and 2 are, respectively, 2, 10, 14. 29. Let BB = x. Then CB = AE = 2x, and from right triangle ECB, EC = aV5. Hence EC:CB = D ¢ 2x x A ak Ex B -88- 30. 31. 32. 33. 34. Solutions—Spring 1978 Let a = 16x? - 9, b = 9x? - 16. Then a+b = 25% - 25, and we van use (1.22), “The possibilities are 16x? - 9 = 0, 9x? - 16 = 0, or 25c* - 25 = 0, giving rise to the six solutions: 4,-+,4, rh Squaring produces sin’e +2 sin © cos = + cos’s = 1+ sin 22-2, 24 so sin 2x Zt. Recognizing the ratios from a 7-24-25 right triangle (or otherwise), we find cos 2r = (a positive value, since 2e is in quadrant IV). Subtracting the first equation from the second gives ay - ya = 123 adding this to the third equation gives zy = 16. From this, ya = 4 and ez = 9, Multiplying these last three equations gives (aya)? = 16+ 4+ 9, so myz = 4+ 2+ 3 = #24, If we now divide ae 6° *2> 8. Noting also that # and y must have like signs, this equation by zy, xs, and ya successively, we find « w= 46, and y as must y and s, the only acceptable triples are (x,y,2) 8.3 83 (6 $ > 3) and (-6, -$ 5 -3)- We wish to find a linear equation whose graph passes through the points (°C,°F) = (25,40) and (125,280). ‘The slope of the line determined by the two points is 2, so sp—ae- 2, and ope loc. F= Bec - 20, In general, if two triangles are situated as in diagram 1, then : =(2 pq sin B):( Lae sin B) = PL area(PBQ) :area(ABC) = (5 pq s 8):( Sac sins) = 22. tn this problem (see diagram 2), the area of triangle PR can be obtained by subtracting the areas of the three outer triangles from that of triangle ABC: ken ee ee ee eee eC - = 2 -(3-% eee G arg d ®) FE sok = 10. For more such problems, see "the Menelaus triangle," in Kay (p. 208). 8 8 eJe ele a A iS A R Diagram 1 Diagram 2 -29- Solutions—Fall 1976 35. 36. Since y- == 2 ~ 6, we have y = 2¢- 6. And since 22-2, we have lér = y? = 4x? - 24¢ + 36. Thus 4x* - 402 + 36 = 0, so Lor 9. Then (x,y) = (1,-4) or (9,12). « Substituting the points into the equation produces: “Msta-bee (ay Leatbee (B] 42dat+ bere [Cc] Equations [4] and [B] imply that b = 6, Substituting in (8] and [cl leads to a= -1, ¢ = -4, Now the vertex of the parabola ys -s? + 6c - 4 has abscissa -Z-= 3, so its ordinate is “9+ 18 -4= 5. Hence (#,y) = (3,5). Solutions—Fall 1976 Le 4 4 let =a, ory=4. ey ory st = f+ 1 = at a+ Se" + 100? 4 lon? + 5241-29 41, oF Sa(a* + 2x? + 22 + 1) = 0. The factor theorem (1.311) shows chat x +1 is a factor of the polynomial. Dividing, we get se(e + Ge? +241) = 0 ands asi8, 0, 1 = Lttang _ cos c+ sine ta(T + 2) = PEERS + SETHE (see sen. 4+ 28 sin 2x 4 +8 sin 20 A sec Qn + B tan de = os ae costs - sin? Comparing the denominators suggests that we convert the first into cos x + sin the second by multiplying by 282" Sn 2 getting geste + 2 sing cos a + sin’e | 1+ sin 22s (4,8) = (1,1). cost ~ sinte cost - sinte Since triangle #86 triangle co, aE. 12, ans ee M. 5. 10. Solutions—Fall 1976 Using mass points, we assign weights 1, 2, 1 to A, B, C, respectively. Then 2B + 1C = 3D, 14 + 10 = 28, and 2B + 26 = 3D + 14 = 4F. Hence ® eo 9 & ‘Oo Subtracting the second equation from the first, (t - y) - #(n - y) (@-y)Q- 2) =0. Thus2=lorg=y. If z= 1we have e+y=6and 1+ cy = 6, and (#,y) = (5,1) or (1,5). This produces (5,3,1) and (1,5,1). If = y we have x + 22 = 6 and 2+? = 6, Subtracting, we find 2? - 2 - cz + 2=0 or x(x - 1) -a(@@-1)*(@-2)@-+0. Thsz=lore=2. Ife=1, we have y= 1 and z= 5 producing (1,1,5). If # = 2, we have f= y= 2, sog+e2 = 6 and x = 2 or -3 producing (2,2,2) and (-3,-3,-3). Method 1: Using Hero's formula (see 3.61) with ¢ = 8 gives area K = W(@-2)(@-2). Since the factors of the radicand have a constant sum, their product is maximal when they are equal (see 6.3); this gives «= 5 and X = 12. Method 2: The locus of points such that the sum of the distances to two fixed points is a constant (e.g., 10) is an ellipse with the fixed points as foci. Clearly the triangle has maximum area when the third vertex is at the end of the minor axis, giving a triangle of sides 5,5,6. Again, we find X = 12. 2° 4 2h¥ = 2 + 2Y") = 25*, Thus the factor in parentheses mgt be a power of 2, which is only possible if 4y = Se. Then (1 +1) = 2° leads to 3e + 1= Sz. Trying multiples of 4 for x (since 3¢ = 4y) quickly produces (8,6,5) as the required solution. For a more general treatment, see Liff. Note that 4624 = 17? + 2" so 2? - a which # = 17. x?(@ - 1) = 17? + 16, from We can solve a slightly more general problem just as easily. Let R and Y represent the radius-lengths of the large and small circles, Tespectively. Clearly, angle AOX = 60°, so Axis a 30-60-90 triangle. Ror =. Then -91- Solutions—Fall 1976 10. lL 12. 13. (Continued) 3R 3+ WS Here, r = 3. For generalizations of this problem of the "kissing circles," see Coxeter (pp. 13-16). Solving, we find that r= Since logion = L ++ 10% = n, we have logllog(log «)] = 10° Then, log(log x) = 10' = 10, Then, log « = 107°. 10 Finally, = 10°? , and k = 107? The left side of the given expression simplifies to (cot_# + tan x) (cot # - tan 2) SS SES ER SD . which, after cancelling the common factor, converting to sin x and cos #, and simplifying the resulting complex fraction, becones cos?x - sin’. When this is added to 2 sin’, we get the expression cos*z + sin’x. Since (see 4.11) the given expression is identically equal to 1, we have (ul) = (41). Using mass points, we assign weights of 4, 6, 9 to A, B,C, respectively. Then 44 + 6B = 10D, 6B + 9C = 1S, and 15E - 10D = SF. Hence DE:EF = 1:2. Mm) @o E@ -92- 14. 15. 16. Solutions—Fall 1976 Squaring both sides, we get sin’ + cos’s + 2 sin « cos @ = x 24 24 1 5 Thus, 2 sin # cos # =-34, or sin 2¢ » and cos 20 B i _ sin 2 (see 4.21 and 4.11). Since tanz= ~S22— (see 4.25), we have 24 2B 24 4-24 4 3 5 tan a = Sg eH ord Gor -F + Bot =F doesn't check, “3 as this implies (in Quadrant II) that sin 2 = and cos 2 =-2, 1 5 5 so sin = + cos # = ‘This extraneous root came from the original squaring we did. Method 1: The given series is equal to (f+3)*G a) (are as)? etre ete) sa mst ee 52 re. as 3 Method 2: The given series can also be written as (1.523) For particulars about rearranging terms of an infinite series (without affecting convergence), see any advanced calculus text. At the end, the amount of wine in one jug must always equal the amount of water in the other. Hence, both jugs must be } wine, } water. We examine first the jug that started with all wine. After the first transfer, it has (« - 2 wine):(@ total); the second transfer removes wine in that ratio (and adds wine in the ratio 2: from the other jug). Thus the amount of wine there at the end is (@ - 2) -(252)+ 2 +(2)+ 2, which mst atso equal ©. simplifying produces 2 -93- Solutions—Fall 1976 17. 18. 19. 20. Let M be the midpoint of BC, For all real numbers k, 0< k <1, and for each triangle ADP (with vertex P on iB), altitude 7 + k, there is another triangle ADP’ (with vertex P’ on MC), with altitude 7 = K. tence the average arca = SU .* 80-1) 6 55 The above solution is the one given in the original contest, and seems intuitively clear, However, depending on how one thinks of the concept of "averaging" an uncountable set of data, one can arrive ata variety of onswers to this type of problem. Such continuous probability distributions are discussed in any advanced Book on probability. fo’ A @@- VE +240 -6@ set e0= wt +e+ DE -2-6) alt is = (z? + @ + 1)(@ - 3)(@ + 2). The solutions are -2, 3, Note that when dealing with base b, a numeral such as 123.456 (for example) os in base ten, 1(6)? + 2()! + 3(6)° + 46) 7 + 5()"" + 6b"? +... . The "decimal" part is therefore # zz 6 g.i.2,1,2 ter tees Now (see 1.523) .17 gtartgrtgrts Ee] ,2+7 -een(tek eR tert e @ n(s = = @+n(ghy). then £ e+ » leading to x = -4 or +9. ‘There can be no digit 7 in base (-4), so x = 9 only. & Relating perimeter, diagonal, length, and width gives Be a+3, = a+ orvtt= VTE +3, sows 2. GTS GE . squaring each 2 +402 my? o 22 and t 5 gz 2 side, v2 +4un +402 = S . de, v’ wi w? +2? and Sut = 20%. thus ~94- 2. 22. 23. 24, 25. 26. ‘Solutions—Fall 1976 Let b and ¢ represent the respective rates of bicycle and car, and say they pass each other after h hours, 2 has gone a distance bh and Chas gone ch, Obviously, B must now go a distance ch and C distance must go Di to complete the trips. Since time = “stan » we have $= as), or fF 25(2). mis toads to 2-2, Let the side-length of the square be s and the side-length of the regular hexagon be h. Then cs? = 6h?V3/4 and e?/h? = /27/4. Thus, s/h =V2774. Since 48/6h = 28/3h = 'V(I6 + 27)/(81 » 4) 3s we have k = 4/3. Let the first twoprimes be p and p + 2. Of every three consecutive integers, exactly one is divisible by 3. Neither p nor p + 2 is divisible by 3 (since each is prime and greater than 3), so 3 is a divisor of p +1. But (p) + (P + 2) = 2(p + 1). Since p is odd, p + 1 is both even and divisible by 3. Hence p + 1 is divisible by 6. Thus the expression 2(p + 1) is divisible by 12. Since 5 +7 = 12, the answer is exactly 12, and not more. Let the center be 0, and AT =a, ‘Then 4 = 2v2. By the law of cosines in triangle A#0, (2/2)? = 32 - 16v2, or 2x? = 32 - 16/2. The area of the quadrilateral is the sum of the areas of the 12 : = om? g(1OZ triangles pictured. ‘Thus, the area = 2x? + a1") = 16(2 + /2) and k = 16. T. we B x X e 6 > F € cose ty - 2+ -$- B= R. (See 4.515.) The following solution is adapted from Niven, Mathenaties of Choice (pp. 27,28). Let £ denote the act of driving one block east, and — NM denote the act of driving one block north. Thus, to each route, there corresponds a string of five E's and five N's arranged in a row. ‘The nunber of ways one can fill 10 spaces with § E's (the remainder being l!'s) is given by the binomial coefficient (*¢) Ae = 252 sI5T ; -95- Solutions—Fall 1976 27. 28. 29. 30. Method 1: Since b = a+ ¢, we have ax* + (a+ e)e +e (ae + e)(@ + 1) = 0. The larger root is -1. Method 2: Using the quadratic formila, the two roots are seen to be ate) + Var eye = fae , -(a + 0) + Va- aF 2a 2a qq! and the larger root is -1. Note that @ -asince e>a. The two vertices must be endpoints of a major diagonal. Of the two possibilities that exist when two adjacent faces are unfolded, the first diagram provides the shortest path, and the length of the hypotenuse is 20. & Figure 2 VB Since To we have k = 98, 1- Since c? = a® + (49 - e)?, we have a= 7/2e- 49. Since 2¢ - 49 must be an odd perfect square Less than 49 (since the hypotenuse < 49), the values of a are 7, 21, 35. A solution using (2.1) is also possible, although rather more involved. -96- Solutions—Spring 1977 Solutions—Spring 1977 i) Im each of the three methods below, we first find that the ratio (© + yz equals 4:1 and the ratio #:(y + 2) Solving for a and y in terms of then shows that 22. Method 1: Using mass points, assign weight 1 to point 4 and weight 2 to each of points 2 and C. Then 2B + 2C = 4M, 2B + 14 = 3D, and the center of mass of the system is at G, Hence 1 + 4M = 5, so (e+ y):z = 4:1, To determine ©: (y + 2), we assign weight 1 to each of points 5 and C and weight 2 to point A, and proceed in the same way, Method 2: Use Nenelaus' theorem (see 3.83). We first use the theorem in triangle ABM, with transversal CD. We find that (/23C@ + y)/s) (1/2) ='1, so (@ + y):2 = 4.” We then use the theorem on triangle 4BY again, but this time with transversal CB. We find that (2/1) (@/(y + 3))Q/2) = 1, 50 @:(y + a) = 1. A c Method 3: Use areas, We use the fact that the ratio of the areas of ‘triangles with equal bases (altitudes) is the same as the ratio of their altitudes (bases). We will use absolute value to denote area. First we draw line BG. Since BM = MC and triangles BGM, OMG have the same altitude from @, we find that |BGM| = [CWG|. Since triangles BAY, MAC have the sane altitude fron A, we find also that [Pau|"= nc]: Subtracting, we have 1943) = [010]. Since 4p:08 = 2:1 and triangles AGP, BGD have the same altitude from G, we have |agp|:|Bop| = 2:1. Let |BcD| = 4k. Then |AGD| = 8K and |CAB\ = 12k, so |C4D| = [aac] + |AGD| = |GaB| + |AGD| = 12k + 8k = 20k. Now triangles CAD, CBD have the ae altitude from . so |cap|:|CBD| = AD:DB = 2:1, or |CAD| = (2/3)(|C4B|}. Hence |C4B| = 30k. But clearly |aaB|"= cas} + \cécl + |pxa]’ + Joucl = 24% + 2]2ua|, so that [BMG] = 3k. Lastly, we find that \GAB|:|GMB] = 12k:3k = But triangles GAB, BYG have the same altitude from B, so this ratio is also the ratio of their bases, or (© + y):2 To calculate z:(y + #) we draw segment BF and proceed similarly. For more on this method see Kay (pp. 197-207). -97- Solutions—Spring 1977 There are 199 integers in the given set. Let M(n) denote the nunber of integers in the set that are divisible by m, and let M(? or 11) denote the number of integers that are multiples of Tor ll. Then: M(7 or 11) = M(7) + MCI) - M(77). (Gee 7.2.) Using [2] to represent the greatest integer function, we have: MCT) = [299/7] - [100/7] = 42 - 14 = 28 MCL) = [299/11] - [100/11] = 27 - 9 = 18 M(77) = [299/77] - [100/77] = 3- 1 = 2, and M(7 or 11) = 44, Hence the desired probability is 44/199, For more on the inclusion-exclusion principle, see Niven, Nathenatics of Choice (Chapter 5), letting y = 0 gives f(z + 0) = f@) + f(0), or £(0) = f@) + £(0). Since #(0) # 0, we can divide both sides by f(0), obtaining f() = 1 for all real values of xz, Hence f(1977) = 1. squaring both sides, we obtain sin? $= 2 - 2/77 FW. Noting that 1 - sin*e = cos"#, we have sin? 5 = 2 - 2/c0s"@ 2- 2lcos a]. Using 4.24 we have (1 - cos x)/2 = 2 - 2|cos =|. We consider two cases: Case I: 0 < x < 1/2, In this interval, cos # > 0, so [cos x| = cos a, and we have (1 - cos 23/2 = 2-2cos . Solving yields cos # = 1, so that tan x case Il: 7/2 << 7, Here cos x <0, so |cos =| = -cos 2, and we 3 4 5° so that tan 2 =-. The required solutions are therefore 0 and 4 3 have (1 - cos #)/2 = 2+ 2 cos #, Solving yields cos = Let a= vey and b= /e-y (where a > 0 and b > 0). Since a? =a+y and b* = x - y, we may rewrite the given equations as: a@+a=56 bP +B = 30. Then a? + a - 56 = (a ~ 7)(a + 8) = 0, or a= 7 (rejecting a = -8). Similarly, we find that b= 5. We now know that z + y = 49 and - y= 25. Solving this system, we get (2,7) = (37,12), -98- 10. Solutions—Spring 1977 From the binomial theorem (see 1.21), we have (w + y)? = 23 + 307y + Sry? + y3, Hence the given information shows that 2° + y? = 3x%y + Sny? = Scy(x + y). Since « + y #0, we can divide both sides by = + y to get Smy = a? - ay + y? (see 1.321), or 2xy = a? = Quy + y? = ( - y)*. Hence (a - y)*:ay = 2:1. We have the equation x - y = a - y?, Ifz-y #0, this leads to x+y = 1, Substituting in the equation ay = 2 - y gives ¢ - 2? = 2-1, This leads to (#,y) = (15 em 3 pair of values that are not both positive. We can be sure, in dividing by 2 - y, that we are not dividing by 0. For if - y = 0, then the problem asserts that y = 0 as well, which would mean that either z = 0 or y = 0, contrary to the conditions of the problen. )and another This problem is an exercise in factoring. Let us write « for sin © and c for cos 8. Then: of + o® = (97)? + (07)? = (e? + e?)(84 - ee? +o") (1.321) 20% = 1 - 3e7e?. 2 Hence 1 - 38°¢' But this implies that 4 sin?6 cos*8 £072) = £(9) + f(8) = 2b + 3a. It is not hard to guess that / must be a logarithmic function. For a nice development of these functions from the functional equation f(xy) = f(x) + f(y), see Apostol, (Vol. I, pp. 174-181). We have (@ + Vy)? = a? + y + 20/y = Ah 345. Comparing rational and irrational parts, we see that a? + y = 7h ona 209 = 3/5, or any = 27. Eliminating y by substitution gives 4x" - 21x? + 27 = (4x? - 9){? - 3) = 0. -99- Solutions—Spring 1977 10. Le 12. (Continued) The only positive, rational value for 2 is $, so (ey) =(F, 3). Note that the equation 2av/y = 3/3 guarantees that 2 must be positive as well as rational since if 2 < 0, 2avy <0 as well. In general, we can Solve this type of problem by noting that (va + v5)? = (a+b) + 2¥ab. In the present problem we can express the radicand in the form (a + b) + 2vab: 2h, syq ef 2h 12/3. [21 + 27708 | voi + 2/t08 4 YF YO a a Letting @ + b = 21 and ab = 108, we quickly see that ¢ = 9 and d= 12 is a solution. Therefore, Zi + 2103 . SVE 3, yg. For references, see problem 10, Spring 1975. In base @, the numeral 7y3 has the value 7x” + yx + 5. Similarly, 3y7 has the value 3c’ + yx + 7. Hence we have 7x? + ya + 3 = 2G3e" + ye +7), or 2 - ay - 11 = 0. Since both = and y are positive integers, we can write a? - ay = x(x - y) = 11. Since Ll is prime, x = llorg= 1, If@= 11, theny = 10. Similarly, z= 1 implies that y = -10, which is impossible. Therefore, ety 2. et #1, 025 e35 «++ be the Lengths of the sides of the successive hexagons. For each c,, we find (see diagram) that 9),,22, = ¥3! Hence 63,,,:6e, = ¥5:2, and the perimeters forn a geometric progression whose first term is 12 and whose common ratio is 73:2. Their infinite sum is thus = 24(2 + ¥3), (See 1.523.) -100- 13. ua. 1s. 16. Solutions—-Spring 1977 e+ se T- (a) (4) © = (62 - 1)@+ 1) = 0, The root # = -1 is extraneous, as the Using 4.4, it follows that 1 or 627 + Se - 1 probten dengnds principal vatues for Arctan, soe = b. In the figure, note that triangle CFD ~ triangle EF and triangle CDF ~ triangle ABF. 24 _a 24. _b jl 24, 24 Therefore, = —2> and = 2. Adding, we get Sb + Fh = 1, which leads to «= 60, (See also 3.81.) 40 BD ap b F ‘The conditions of the problem are equivalent to 9x* Solving for # produces the two solutions « = 0 or « = The original expression may be regrouped as (a*b? + cd?) + (a’d? + b?c*), which may, in turn, be written as (ab + cd)” + (ad - be)* or as (ab - ed)? + (ad + be)*. Since a= 44, b = 10, ¢ = 33, and d = 5, we obtain the two solutions x = 605, y = 110 275. or = = $50, y A third, unrelated solution, may be obtained by noting that 2 + y® = 378,125 = 55 + 117 = (38? + 417) (0% + 117), Using the identity (a? + b*)(c* + a} = (ae - bd)? + (ad + be)? and letting a= 38, b= 41, ¢= 11, and d= 0, we get 2? + y* = (38+ 11 = 41+ 0)? + (38+ 0+ 41+ 11)? = 418? + 4517. Since @ > y we have « = 451 and y = 418. For a further discussion of these and related identities, see Beiler (pp. 141-145). -101- Solutions—Spring 1977 17. 18. 19. 20. By expanding and simplifying we obtain (stv ae b)s(v ere b)s(auts ae sty) (47 v1 eats +1437) ze We may rewrite the given system as 3x - dy = 6 ~ p 6x + dy = 15 for some positive number p = 6 - (3t - 4y). Adding the equations yields 9 = 21 - p or@ ak and hence y = 4,2 . by the nypothesis of the problem, 2 - 22" ana 7-8 . We therefore rewrite our solutions as < Le 2 ¥ G—= to match the given form. Hence » = g and e z and we have pre = 1:2, Simplifying the original equation, we obtain log, 28 = 1og,vé. Hence 28 = Z or « = $28. For another approach to this problen, note that logy = } for all 2 >0. tence log, 28-3, 2° = 8, 625 and again 2 = 5? Method 1: Completing the square gives (ef +erh)eaey et, or(eed) - or 4y® - (2x + 1)? = 11S. Factoring the difference of two squares on the left produces (2y + 2x + 1)(2y - 2c - 1) = 115. Since & and y are positive integers, each of the factors on the left are also integers and hence mist be factors of 115. This leads to eight possibilities: Yr d+ l= 1s 235 51 dy - de -1 21 % 5% 238 us * We may ignore the last two cases, as the first factor must be larger than the second. Adding, we have: 4y = 116 or 28 y= 2or7 =102- 20. a. 22. ‘Solutions—Spring 1977 (Continued) and by substitution, x = 28 or 4. Hence («,y) = (28,29) or (4,7). Method for & using the quadratic formula, we obtain a = “1+ Jz Ts See. that 4y? - 115 be a perfect square. This means that 4y? - 125 or 4y? - k? = 115, Therefore, (2y - k)(2y + k) = LIS. Following the plan of the first method illustrated we find that y = 29 and Writing the equation as x® + x + 29 - y* = 0 and solvin fs Pao = PT 2 For « to be an integer, it is necessary ors @ k= 7, ory = 7 and k= 9. Solving for « gives the same pairs found above. We want to yelate the given equations to the binomial expansion of (« - y)?. Multiplying the second equation by 3 yields 3(a*y - zy?) = 18. Since 2° - y? = 19 we have (@ - y)? = «? - y? - 3(@*y - ay?) =19- 18= 1. Hence ¢ - y= 1 (rejecting possible complex values). Also from the second equation, we have ay(e - y) = 6 and since £. substituting in the equation © - y= 1, we find ay = 6 ory = - y= 1 and simplifying we obtain «® - x - 6 = 0, with solutions z= -2ora= 3. The corresponding values of y are y = -3 or Hence (x,y) = (-2,-3) or (3,2). Using 3.5 shows that AZ = AY = 2, 2C = CX = 1, and XB = BY = 3 {sce figure 1). We may now apply the method of mass points with BE = p and EZ = q, Assign weight 3 to A, 2 to B, and 6 to C (see figure 2). Therefore, BE:BZ = piq = 9:2. A® A 2 Z| t 3 @ 1 ery 3 8 6® X® Figure 1 Figure 2 -103- Solutions—Spring 1977 23. 24. 25. 26. 3 1s Let Arcsin 2 = 4 and Arccos 7? 3 Then: sine = sin Arcsin 3 + arceos 48 5 17 sin(a + 3) sin 4 cos B + sin B cos A 3,15, 8,4 Sap tipg s (see 4.310 eid s Method Squaring both sides yields k = 2c - 2ve7 = (Ze - 1) 2 de - WEES TE TT = de - WEA? = 2x - 2Iz - 1). Again, x > 1 implies that - 1> 0, hence |r - 1] = - 1. Therefore, k = 2x - 2(@ - 1) = Method 2: Since the given equation is an identity for all z > 1, it must certainly ho1d true for any specific value of « that is greater than 1. Therefore, we can choose a "convenient" value of @ such asx = 5 and WR = VS +3 - VS - 5 = JB - yD = 2/2 - V2 = V2. Hence k = 2. The value = = 5 was chosen so that 2 ~ 1 would be a perfect square. Since ¥ is a numeral in base x, it is helpful ¢ to xpress Nasa polynomial in x. By long division, V = a" + 2? anal. Therefore, N= 11111. See 1.33, with y Let us first solve a more general type of problem. Suppose we wish to count the number of distinct paths from point A to point B (on the grid shown below), allowing only moves in a "northerly" or "easterly" direction. One such path is darkened in as an example. We may describe the darkenea path by saying, "Go 1 block east, 2 blocks north, 1 block east," etc. B i, LEHH -104- 26. 27. 28. 29. Solutions—Spring 1977 (Continued) If we symbolize 1 block east by the letter E and 1 block north by the letter W, the darkened path may be written as ENNENNENNEEN. Now any path from A to B must go 5 blocks east and 7 blocks north. Hence any such path corresponds to a "word" containing five E's and seven W's, Also, any such "word" corresponds to a unique path. ‘The problem is therefore fully solved by counting the nunber of distinct permutations of the letters of the "word" EEEEENNNNNNN. 12! 5 treatment of this problem and many other interesting problems of this type, see Niven, Mathematics of Choice. Returning to the present problem, we first calculate the total number of paths from (0,0) to (4,4). ‘The above discussion shows - 6 paths from Hence the number of distinct paths is For a thorough this to be 70, Similarly, there are x57 7 (0,0) to (2,2) and stip = 6 paths from (2,2) to (4,4), Thus there are 6 + 6 = 36 possible paths from (0,0) to (4,4) that pass through i ity is 38 or 28 (2,2). Hence the desired probability is 5} or 3 From the given information, we have (1) 2 = 120 and (2) 2a + b = 120. Solving this system yields (a,b) = (40,40). ‘The key to the solution of this problem is a well-labeled diagram. Since BQ and 0@ are angle bisectors, and BCF||DEQ, we have angle DBQ = angle CBQ = angle BQD, and angle BCQ = angle FCQ = angle CUE. Therefore, triangle DBQ and triangle ECQ are isosceles triangles. Hence DQ = 8 and EQ = 6, so ED = 2. We use the fact that if [x] = m and x is not an integer, then [-«] = -n- land ([x] + [-2])° = (1 - n- 1)8 = (-1)° = -1, See 5.22. -105- Solutions—Fall 1977 A+B A-B 30. Method 1: Since sin 4 + sin B = 2 sin (see 4.321), we have sin Sz + sin 3x = 2 sin 4x cos x. Therefore, 2 sin 4x cos x = 0, which implies that sin 4x = 0 or cos 2 = 0. Since 0 < @ <1, we have the three solutions « = Zor Zor 2. Method 2: We write sin Sx as sin(4z + =) and sin Sr as sin(4x - 2). ig Formulas 4.311 and 4.312, we get sin Sx = sin(4n + 2) sin dz cos ¢ + sin © cos 4x and sin 3x = sin(4z - 2) sin 4x cos x - sin cos 4x. Adding these two identities yields sin Sz + sin 3¢ = 2 sin 4x cos x. Now proceed as outlined in Method 1. ns—Fall 1977 1. Adding the two equations gives Sz’ = 5, so x = 41. Substituting these values in either equation gives Solutions (#1,0), (1,2), (1-2). Using the law of cosines in triangle BEF, and noting that cos angle EBP = sin angle ABE = £, we have: 2? = 9 + 25 - 2(3)(5)(4)= 10, soz = VIO. We can avoid using the law of cosines if we draw a square with adjacent sides AF and BF, calling its fourth vertex H_(see diagram). If we drop perpendiculars from point £ to sides 4B, BF, the Pythagorean theorem alone will give the required result. c 5. Method 1: tn right triangle 480, 20 = 24, Im right triangle 4¢D, AC = 15. Then, using Ptolemy's theorem (3.91), BC * 25 +7 + 20 215+ 24 ang 30 = 44, a 8 + A oO -106- 3 ‘Solutions—Fall 1977 (Continued) Method 2: We have, from right triangles ABD and ACD, = 24 = 20 cos angle ADB = 5¢ cos angle ADC = 5° ; z A 15 sit are si yg = 1S im angle ADB = 3 n angle ADC = 33 Hence cos angle DC = cos(angle ADC - angle ADB) M18, 7, U7 2 25° 25 25 125 ° Now we can use the law of cosines in triangle BDC to find «. a z Meeting Point 3es-8) + Hes-4) a———_+—_—____+—___—_-3 Rate ® e x -10 Consider 4's motion. He first travels 5 hour at = km per hour. Distance Since A took § hours to complete the entire trip and the problen 4, £225 = 4x) | 3(225 - 3x) implies that «> 0, 5+ - Sp 20? - 1092 + 450 = (2 - 9) (x - 50) = 0, soz = SO. Then Since each of the given polynomials has a lead coefficient of 1, if either of them factors into a quadratic and a linear polynonial, then the lead coefficient of each must be 1. Let the required quadratic factor be =* + px + q. Since any common factor of two polynonials is a factor of their difference, we must have 2 a + pe + q = HI(a - b)s* + 3e + 3] For some integer i. Since P and q are integers, 4 must divide 3, sa = 1 or # If N=1,a~-b-=1 as well. Suppose x? + be? + 62+ 3 = (* + Se + 3)(@ +”) for some integer r. Clearly » = 1, and multiplication produces b = 4, soa = $, We then check that x? + Sx* + 9x + 6 is divisible by 2? + 3c + 3, which it is. If W = 3, the common quadratic factor would be «* + 2 + 1, and we would need 2? + be? + 6x + 3 = (x? + @ + 1)(# + 8) for some integral -107- Solutions—Fall 1977 10. (Continued) e. Clearly ¢ = 3, but then (? + @ + 1)(@ + 3) does not produce a 6x term. Thus only 2? + 32 + 3 works, and (a,b) = (5,4). For a more detailed look at common factors of polynomials, see Borofsky (or any thorough book on the theory of equations). ‘The probability that they roll equal numbers is +. Hence the é probebility that they roll unequal mabers is £. Since neither player has an advantage over the other, the probability of either 5.5 ; wel. rolling higher than the other is + Z= yy - Let @ and ar be the first two terms of the progression. The given condition can be written as ad teeter a ry oes), For real », it is not hard to see that the nunerator of the last pee nee foree Mi eeeteeteerctneret = teeta ata To see that the numerator can never be zero, note that (PGMs Phare red) = Po - 1, (See 1.33.) Thus the values that make the nunerator vanish are the four complex fifth roots of unity. Since = 2- Oo? . Gna e-a. 38. we have (1.12) ~a@ “ova $= 4. tet o = 41k and a= 9k, Then d= 40k. Since X is a common factor of a, b, and c, we conclude that k = 1, Therefore (a,b,c) = (9,40,41). By (4.323) cos 3x + cox # = 2 cos 2x cos # = 0, so cos x= 0 or cos 2n = 0, The last equation leads to cosa =+t2, We have (1.22) a’ +b? = (a+b)? ++ a= 0 orb = 0 ora=-b. Letting a= 22 - y, b= a - 2y, we have y = 2x or x = 2y or Letting Jeet @, the second equation can be written asc+tarel ord tie eis ase+h=2+4,soe=2ordande+i=dyore+i=%. -108- 10. lu. 12. 13. 4. 15. Solutions—Fall 197 (Continued) ‘There are six ways to combine the conditions into pairs of simultaneous equations, Each set of equations leads to one of the six solutions: 1 2 1 8-4 1 ol a4 (FF) C9. (3). (F > F)G +s) (F> Since 1 < = < 2, we can write z= 1+ k, where 0 0), 1< 1+k< V2, and lsa< v2, ‘The quadrilateral made up of the two triangles (see figure) always has opposite angles supplementary. Hence it is always cyclic (3.92), and we can use Ptolemy's theorem in each case (3.91). For example, in the first case pictured, 25¢ = 7+ 20+ 15 + 24 and = 20, The other cases give 227, 25, 24, 38, and 25 again. ‘There are altogether five possible distances. dX Sh 7 Me ES es, If the roots of the equation are r, r, and s, then (1.4) 2r +5 = 1 and r’s = -1, Eliminating ¢ between these produces ar? r= (v- I)Qr? +P + 1) = 0. Since 7 is real, we have » = 1, Substituting this value for = in the original equation gives m= -1. We expand, regroup, and simplify: (4z%b? - 12abay + 9a7y?) + (dee? - daces + a?a) + (Qy7e? - 6beyz + b?2") = (2nb - 3ay)? + (2me - az)” + (Bye - ba)? = 0. Since each term must equal 0, $a, 2. 2-38, 4. bh. B® Bez le bea se & An integer that is both a perfect square and a perfect cube is a "perfect sixth power." There are four such integers less than or equal to 4? = 4°, The probabitity is thus (4)#(%)= ge -109- Solutions—Fall 1977 16. We will use absolute value to denote area. In parallelogram ABCD (see diagram), let BM = MC = DN = WA. Since the centroid of a triangle divides each median in the ratio 1:2, we have MG:GE = 1:2 and HG:GH" = MG:(GE + BW) = 1:(2 + 3) = 1:5 and GH =(3)m. Now |4BcD| © HEY whi slap. (2) -(% AD + aH" white [Ago] = 5 a0 «(Ean = (qh) laBcol. Similarly, |a¢c| = (5) |48co|. Finally, a line through ¢ perpendicular to AB and CD would be bisected by the parallel lines AB, NM, and DC, so |ABG| = |cap| = t |azcp|. Hence 4Go| is the largest possible, and |4cp| = 5 + 180 = 75. 8 aM ¢ A WaT 7. Using 4.321 and 4.323, the given expression may be written as 2 sin 60° cos 20° _ os Fcos G0" cos 20" ~ tan 60° = V5. 1+242+ 243224 cane 2 + n- ens 2 and 18. Let S Then 25 = 27 + 2+ 2+ 28 - $= -(24 224224... 4 Mane Dig ns 2%" 25, and 6-2 e aan Ye om ay +2. When » = 21, S$ = 20 + 2°? + 2 or 83,886,082. logan? _ 2 1s. Since 271082" ~ = 2*, we have 2x? = 8 and x = 2. 20. Let us write p(a) = 2? +2 +1 and q(2) = (@ +1)"- 2" -1, A quick way to do this problem is to note that if p(x) divides q(#), then the integer p(1) must divide q(1). Now p(1) = 3 and q() = 2" - 2. Hence we can try n = 1,2,... to find that » = 5,7 are the two smallest values that work. -110- 20. a1. 22, Solutions—Fall 1977 (Continued) But » = 5,7 are only necessary conditions for p(x) to divide q(x). We must check to see that these values do actually work. For a more thorough investigation of the problem, we need to know something about the complex solutions to the equation x* = 1. For a full discussion of this topic, see Hall and Knight, Higher Algebra, pp. 440-444. We use the factor theorem (1.311) and its corollary (1.312). The roots of z° +z +10 are the complex cube roots of unity. We denote these by w and w*, and we need both to be roots of the equation g(z) = 0. That is, we need: wey" -e"-120 @? +1)" - ou” - 10. If we note that w +1 = -u* and u? +1 = -w, the above conditions become (-w*)" - w - 1 = (-u)" - wo - 120, First let us suppose that » is odd. Then we have from either equation, uw 4a" 410, sou” satisfies «? + +1 = 0 and is a complex cube root of unity. This happens whenever n is a nonmultiple of 3. If m were even, the first equation in (*) above would say that (-w*)” - w - 1+ 0, sow” - uo" - 1-0, This says that ul” is a root of y* - y - 1 = 0, which is false (since the roots of this equation are real, while odd powers of w are complex). We can denote the roots that sum to zero as r and -r. If the third root is e, then (1.4): Gore (Cr) +es-a or a=-a Gi) -r’e = r’a = -e Git) re - re - rt =r? =, From (ii) and (iii), ¢ = ab. If the center of one circle is (#,y), the radius v, and the point of tangency (#,0), we have y > 0 and: @ (@- 1? + y- 97 = Gi) @ - 8)? + y - 8)? = (iii) y=r (see diagram) From (i) and (ii), we have y = 7 - 23, Then from (ii) and (iii), we have (x - 4)(« - 124) = 0, Hence x = 4 or 124, and y = r= 5 or 845. -111- Solutions—Fall 197 23. 24, 25. 26. Draw altitude SH. Isosceles triangles MRT, TSR are similar, hence angle AST = angle TAM. Now HR = RP; and RE = 2 my 2 = FPP (see 3.1), so cos angle RST = cos angle TRM = cos angle HRE Ge) (de) d. Ss Rar To get a picture of how f(n) behaves, we can writ FQ) = 1, f(3) = 341, f(a) = 37 +341, £05) nea In fact, it is not hard to see that f(n) = © i=0 (An easy induction will verify this.) Hence f(n + 3) - fe +0 = HO"? - 1) - B=) Leg (32 a4: ZB = 4s" Letting » = 1000, we find f(1003) - (1001) = 4 + 3100° = 22 + 32000, By Hero's formula (3.61), the area of the triangle is VEO E SEs = TSS Ge D+ 2+ 16 = 240. Since 20h = 240, h = 12. We shall prove that if a+b +¢= 0, thena? +b? 4c? = 3abe. We know that (a? + b*) + c? = (a + b)(a? - ab + b*) + c? (see 1.321) and that a + Then (a,+ b)? = a? + b? + 2ab = c?. Thus, a? +b? - ab = c? - 3ab and a? +b? + c? = (a + b)(a? + B® - aby + 0? (-e) (ce? - 3ab) + 0? = abe. thus, abe = AS = 72. For a fascinating application of this result to the solution of cubic equations, see Shkiyarsky, Chentsov, and Yaglon, The USSR Olympiad Problem Book (problens 161, 162, and 167). -112- Solutions—Spring 1978 27. Let 2 = 2”, We have 2 - 64 = 64 = 64+ 1- 32+ 2= 16 are integers). Since = + y ys ora? - y? = (e+ y)(@ - ¥) 4= 8+ 8 (since both # + y and x - y ay, we must solve w+ y = 64 z+y = 32 wey ze z-yel ways? zy a- Only the second and third give valid solutions, and x* = 100 or 289. 28. If a quadratic polynomial has rational coefficients, then irrational roots occur in conjugate pairs (see 1.82). Now sin*15° = 3(1 = cos 30°) = 2(2 - /3). Hence the other root of the given equation is £(2 + /3). Then by direct calculation of the sum and product of the roots, b = -1 and o= sb. 29. We havet 60 _ 60, Oo re7ts a 60_ 80 Gai) +3 Bei 7+i From (i), ty = 12y - 12x and from (ii), xy = 19y - 21x - 1, Equating the righthand members of the above equations leads toy = 22 supstituting in (4) then gives 9? = 28 - 12 = 0, so (9x + 4)(z - 3) = 0. Rejecting the negative root, we have (ey) = G4). ©) ate 30. We have 3@)logsf(@) _ ;slossf(@) | 9) 2 e 2)", f@) 2 Hence we must solve (x + 2)” == + 2. In this equation, all quantities are real (this is implied by the inequality in the original question), and the base of the exponent is positive. Hence either the exponent is 1, or the base (« + 2) is 1. Therefore, the only solutions are 1 and Solutions—Spring 1978 1. ‘There are (3) 120 ways of choosing the three numbers. There are ten choices such that the sun is 15: 10,4,1 9,452 8,4,3 10,3,2 836.1 736.2 945.1 8,542 735.3 65.4 Hence the probability is 10/120 = 1/12. -113- Solutions—Spring 1978, Since the area of the triangle is 6 and its perimeter is 12, we find that the radius of the larger circle is 1 (see 3.62). Then PD = PE = 1 (see diagram) so AD = AF = 2. Solving right triangle APD shows that AP = V5. Now we use the usual trick for common tangents to two circles: we draw rectangle @SFH. ‘Then QP = Ox + 4P = 1+ p and PS = PF - SF = PP - gH =1- 2. Fron similar triangles PQS and PAF, we have QP AP oor FS * Pr Drawing the altitude to the shortest side, we find a 5-12-15 right triangle and a 9-12-15 right triangle, so sin 6 = 12/15 = 4/5. Another method involves calculating the area of the triangle using Hero's formula (see formula 3.61) and equating it with the expression (4) (4) (15) (sin 8) (formula 3.64). We have (84 + A)? = 8°B + 87B + 8C + C or A?(8 + 1)? = 87B(8 + 1) + C(8 + 1). Dividing through by (8 + 1) gives 947 = 648 + Cc, 2, OBC 7p BHC soa? = SPC yg BT. so if B + C is a multiple of 9 we must have B+ C= 9, Hence A? = 7B +1, or B But B and C are each less than 8, Lea + 1) ~ 1)- For B to be an integer, one of these last factors must be a multiple of 7, and since A < 8, we must have 4+1=7,4-6,8=5, andcC=4. -114- 5S Solutions—Spring 1978 The angle subtended at the origin by the line segment is 60° (see diagram). Using the law of cosines, 2° = 225 + 49 - 14 + 15 cos 60° 169, so x = 13. ° ie 2+ D. We know that the sum of the first » natural numbers is >). (see 1.53). Let the first odd integer be . Then « + (@ + 2) + @ +4) + (@ +6) +... + (@ + 198) = 100, or 100r +244 +64... +198 = 100r + 201+ 2434... + 99) 100e + 99 + 100 = 100'°* Then « + 99 = 100°°, and « = 100°? - 99. Using the "chain rule" for logarithms (5.11): logaz + logs3 + logsz + logei3 * logs = 7 logst + dlog3@ + tlogar = 7 (logse)(1 #242) =7 7 logs = 28, and x = 3° = 81. There are ” pairs whose sum is x + 1, so we need the largest integer n such that 1+2+3 +... +n < 1978, or n(n + 1) <2 + 1978 (see 1.53). But n® < n(n + 1) < 2+ 1978 = 3956, and 62? = 3844 < 3956 < 3969 = 637, son = 62. Now1+2+... + 62 S2= 63. 1055, so the 1954" pair is (1,63), and this is the 2 first whose sum is 64. We need 24 more pairs to reach the 1978" pair, which is (25,39). This is the arrangement used by Cantor in his proof of the denumerability of the rational numbers. See, for example, Kasner and Newman (pp. 27-64). h First we note that if a? +? = c® has a solution in rational nunbers, then clearing fractions from the equation will produce a solution in integers. Thus the condition abe = 0 must hold also if the equation has rational solutions (see 2.2). We have 6x(2c + 1) + 1= 12x? + 6x +1. Using the hint entails expressing this as the sum of the two cubes. One way to do this is as 8? + 12n? + 6r + 1 - Se? = (2r + 1)? + (-22)*. -11s- Solutions—Spring 1978 10. le 12. 13. (Continued) Hence we must have 2 + 1 = 0, -22 = 0, or y= 0. Only the first two possibilities lead to rational solutions; these are (-3,1) and (0,1). The theorem given in the hint is one case of the classic “Last Theorem of Fermat. For a more complete listing of such results, see Edwards. The sides of a "primitive" right triangle--in which the lengths of the sides are relatively prime integers--are given by a=u? - v*, b= lw, c= u> + v*, where u,v are relatively prime and of opposite parity (see 2.1). Tn the present case, a + ¢ = Qu’ = 2’, Now if a prime number p divides a cube, then p* divides that cube, sow? must be a multiple of 4 and u itself must be even. Let u= 2". Then x? = 2u’ = 8r?, and r? must be & cube. Hence 7 must also be a cube. Letting r'= 1 yields the the solution given in the problem. Letting r = 8 gives u = 16. For the least value of c, we can choose » = 1 and get ¢ = 256 + 1 = 257. Let the length of the diameter be d. Since DP + PG = MP + PN, 12 + 12 = 4(d - 4), d= 40, and the area of the circle is 4007. N We have tan(20" + 40°) = tan 60° = v= ~ta%. 20" + tan 40 T - tan 20° tan 40° ° (See 4.341.) Clearing the fraction, we find that the value sought is /3. Since 1 = (sin’x + cosz)? = sin’x + costs + 2 sin®x cos*#, 1 - 3(4 sin’x cos’«) we have sin‘x + cos*s = 1 - 2 sin®x cos’ 2 (sin 29) -1 (sin 2 1-2 (sin 28) (8)! 337, 625 * (See 4.21.) -116- 14. 15. 16. 17. 18. Solutions—Spring 1978 ‘The wording of the question assumes that P(s) is unique. Since the fraction #@*2)-- He) . ©. 5 is constant, G@+aQ-2 "2 we can take P to be linear with slope 3. Hence P(x) = 3x +b. Since P(0) = 2, P(x) = 3¢ +2. That P must be linear is guaranteed by a simple application of the method of finite differences (see problem 24, Spring 1976). Alternatively, if P(e) has degree n, n we can write P(z) as E a,2”. ‘then Pw + 2) - P(e) n = E ale +2)° - J. The righthand side of this equation is a polynomial of degree n - 1. But the difference on the lefthand side is the constant 3. Hence» - 1 = 0, andn= 1. We have angle OVE = 30°, VO = 2(08) = 210, and YP = 210 + 105 = 315. Since this is the sum of the lengths of the diameters, the sum of the lengths of the radii is 1572. A more general nethod, involving summing the geometric progression whose terms are the successive radii, is more cumbersome in this situation. g A "symmetric" representation of the four terms greatly facilitates the algebraic manipulation. Let the terms be a - 3d, a - d, a+, and + 3d, with common difference 2d and product (a? -'d®)(a? - 9d*). ‘Since 2a? + 18d? = 200 and 2a” + 24° = 136, d® = 4 and a® = 64. Therefore, (a° - d?)(a® - 9d”) = 60 + 28 = 1680. By the “chain rule" for logarithms (5.11), the given expression is equal to 2*(1og,b) (1og,2) (1og,a) = 2*log,a = 8. See why we call it the "chain rule"? We reflect point 4 in the a-axis to get A’ (0,-2), and point D in the line y = 10 to get D' (5,11). Now any path from 4 to B to ¢ to D will be equal in length to the corresponding path from A' to B to C to D', obtained by reflecting 4B in the z-axis and ®D in the line y ='10, The shortest reflected path clearly lies atong Line 43", which has stope 23. This tine crosses the -117- Solutions—Spring 1978 18, (Continued) s-axis at 8 (32, 0) and the ine y = 10 at C($2, 10). Note that this problem calls for finding a minimal value, yet is easily solved without using calculus. For more problens like this, see Yaglon, Geonet: mations 1, II, and IT 19. If PX = @ (see diagram), then PC = 2a, XC = BX = av3, and BC = AC = 203, Hence 2-22 - 1. Bd PA gaV5- 2a VE -1 ie and k = VB +1, c do of olc* of A B 20. We can write: f(2) = f(1) +2+2 £03) = FQ) + 3 . 7; F@) = 7G) Fn) = f= 1) +n Adding, we have f(n) = f(1) + 2(2+3 44+ ... +n) 1+ 2042434. 4m) 2 1s 2(2@ PN) - 2 (oe 1.53), -118- 2. 22. 23. 24. 25. Solutions—Spring 1978 (Continued) and f(m) =n? +n- 1. Alternatively, a simple application of the method of finite differences (see problem 24, Spring 1976) shows that f(n) is a polynomial of degree 2. We can then write f(n) = an’ + bn + 0, substitute n= 1, 2, 3, and solve the resulting system of equations for a, b, e. 2 Since W is odd and T = 4, we have W = 1. Since 4(100¥ + 10f + 1) = 1000D + 100F + 10W + 4, we have 30H ~ 68 | 3(13W - 25) 100 160 But 13 - 28 < 13 - 9 - 2+ 1 < 200, so 13 - 28 = 100 and D = 3. Finally, we need 13V - 28 = 100, Clearly, W cannot be too small: in fact, if M <7, then 13V - 26 < 91-2. The case = 9 does D . Hence 100 divides 13 - 28. not lead to an integral value for F. Hence = 8, F = 2, and N+e+ws 84241211 We have sin‘ + cos*s = (sin?x + cos%«)? - 2 sin’ cos*« 1 - dsin?22, (See 4.21.) Now sin 2c has a basic period of ™, so sin®2x has a basic period of } , as does the function under consideration. Since 2¢ + Sf = 87, f must be odd, and 1 < f< 17, The nine possible solutions for (¢,f) are (1,17), (6,15), (11,13), (26,11), (21,9), (26,7), (31,5), (36,3), and (41,1). Of these, only two ordered pairs give prime values for both t and f, so the required probability is 2/9. 1 Summing the infinite geometric progression gives —=27 = 4 i 30 (see 1.523), which equals .0204081632653..., and the required product is 180. Let BM = MC =m. Then BH = AH = m and HC = v3, Using mass points, ve assign weight /3 to A and weight 1 to B and C. The Weight at Mis then 2 Since APiiY = 21/3, k = 3/2, For a nore traditional approach, we can draw MY| BH. ‘Then angle BMY = 30°, so MY = m/3/2._ From similar triangles APH, MPY, we have AP:PM = AHLYM mv3/2, and k = V3/2. Solutions—Spring 1978 26. We can solve the given equation for « in terms of y: ee TEE = TD since x is a positive integer, 196 - 3y? must be a perfect square. This occurs only when y = 3, 5, 7, or 8 and each of these values leads to a corresponding value for 2. The solutions are (8,3), (8.5), (7.7), (5.8), and (3,8). Another approach is to write the given condition as 20* + 2y? - 2my = (a - y)? + x? + y? = 98. Now the problem is to Tepresent 98 as the sum of three squares, and trial and error leads to the solutions quickly. 27. IE the number of heads is % and the nunber of tails is 5 - h, we must have h - (5 - hk) = 1, so = 3. There are($)= 10 ways in which the three heads and two tails can be distributed and 2° = 32 possible outcomes altogether, making the required probability 10/32 = 5/16. n n 28. We have £ (2¢ - 1)? = Eo. tl a1 and (a,b,2,2) =(4,0,-3, 0). an n We can also express the required sum as I i? - £5 (24)? t=1 i=l 2 an? : -[+4% ~ 4-2 as before. 29. Let A = 64 - 2? and B= x - 36. Then A + B = 28 and we seek the minimum value of -AB, or equivalently, the maximum value of AB. In view of formula 6.3, we take A = B = 14, and the minimum for -AB is -196, Alternatively, we can write (2? - 64) (2? - 36) = 2" - 1002? + 2304 = (2? - 50)? - 196. Again, the minimum occurs when x* = 50. 30. Let CX = CY = » (see diagram). Then (30 - rj + (40 - r) = 50, so r= 10 and BD = 30. Now cos B = 4/5, and in triangle CBD we have cp® = 40* + 30% - 2 + 30 + 40 + (4/5) = 580, and CD = Y580 = = 2VT45. See also Appendix B, (3.5). A ° y| ex 6 -120- Solutions— Solutions—Fall 1978 i Let BC = AB = x (see diagram). Using the Pythagorean theorem in triangle DAC shows that DC = 18. Then from similar triangles DAC, 8BC, we have DC:AC = EC:BC, or 18/30 = 15/z, so x = 25. Bab D 1% a pew © Since the denominators are conjugates, neither their sum nor their product contains a radical. Hence, clearing fractions gives 4x = 2e(e? - 1), which leads to x= 0 or x? = 3, The only integral value is 0. Uf the number is 100h + 10t + u, we have h + ¢ + u = 20, or 2(100h + 10e + uw - 16) = 100u + 10F +h. Eliminating ¢, we find that = 2+ w+ 3u/4. Thus u must be a multiple of 4. If u = 0, the reversed number is not greater than 100, If u = 8, # is greater than 9, Hence u= 4, h= 9, and t = 7, Let BB, = ©. Then in triangle ABB,, k = 1? + a? -2¢ 12+ cos 120°, ork =1+2 +27, and in triangle ABB sy 5 we have: 3k = 1+ (@ + 2)? - 2+ 1+ (@ + 2) * cos 120° or 3k =7+Se+a%, Eliminating k produces: 3(1 + @ + 27) = 7+ 5c 42%, Qe? - Qe - 4 = 0, 2 - De+N 20,22 2andk=1+2+427, -121- Solutions—Fall 1978 s 2 2 We have: 2° = 229; 35° = 3255 5? = 595 2748 = (24)°9°79, 375 = (37)'7*°. clearly, 16°°°7* > 917** > 5%, as both the exponents and the bases are in decreasing order. The event for which we want the probability can be represented as the union of the following disjoint event: Py: HE... hoooonx (the x's denote any outcome at all) 11 times Po: THN, . Hoexxxx 1 times yt xTHH., Hoo 11 times ‘The probability of the first happening is 2~'', and that of each of the others is 2”'*, ‘The sum of these is the desired probability, ge PLM geotools gig? so P= 2 6-222 ge rteg 42D egy ‘The best way to attack this problem is to write out a list of the prime numbers under 100, noting the differences between successive primes on the line below: 2 3a 15,17 39 25,2931 37 41 47 SE SV? 67 71,75 7985.89 97 N2w/s2 42462642 4 26426468 ‘The situation in the problem corresponds to cases where consecutive differences are equal. ‘This occurs only at $ and 53. The solution in this biological context is probably unique, as the next such occurrence is at 157. Calculating a few tems, it becomes clear that a, increases exponentially with ». A good guess and (1.33) then leads to the Sn Sn a the answer. Nore formally, we have = - 27 = +, ss 3 ka, a, 4 k <0 (3 - Be +) £ 4.. the sum on the left "telescopes" 2\3" 3 23 consecutive terms cancel each other out--so we have: -122- 10. 1. 12. Solutions-—Fall 1978 (Continued) rf). _4 6 seo kel | Leg 42 8) + 2" - 3) = 1 $ and a, Hence aiove = 3(31978 - 1), For many more such techniques, see Spiegel (Chapter 5) or Markushevich. tyeesh 2 +50 =0, andy = ore Let y . Then 6y? - 35y + 50 = 0, and y ord. wee Wl galore sd ade nel, on S Thus + 5or Zr 2+z, anda 145. We can express each product as a sum: (4.331) sin 40° sin 80° = 3(cos 40° - cos 120°) = 3(cos 40° + cos 60°) sin 80° sin 160° = 3(cos 80° - cos 240°) = 3(cos 80° + cos 60°) sin 160° sin 320° = 3(cos 160° - cos 480°) = 3(-cos 20° + cos 60°) Hence the required sum can be expressed as 3(3 cos 60° + cos 40° + cos 80° - cos 20°); and since cos 40° + cos 80° = 2 cos 20° cos 60° = cos 20° (see 4.323), the required sum reduces to 3(53 cos 60°) = 5 5-H. By the "chain rule" for logarithms (5.11), (10g710) (10g103) = 1ogy3 = $ogy9. And since log.y = we can write — Tos * (1/b) (a) = $1og79 and 10g79 = 2a/d. Since angle C = angle # = 90 and angle CAE = angle EAB, triangle ACD ~ triangle AEB. We will use this fact to solve for BE. By the angle bisector theorem (3.31), 7:25 = CD:DB = AC:AB. ‘Thus we can let AC = 7k, AB = 25k, and by the Pythagorean theorem, (71)? + 32? = (25%). Solving shows that k = $ and 4p = 100, AC = 8 . Now in triangle ACD, the ratio of the legs is AC:CD 28:21 = 4:3, so triangle ACD has sides in the ratio 3:4:5, and triangle AEB has sides of the same ratio. Hence BE:AB = 3:5. . 2 3 Tf BE = 2, we have aoy-= 3» 3 or @ = 20. -123- 12. 13. 4. 15. (Continued) We need the smallest positive cube that is the sum of two squares. Listing the first few cubes, we quickly see that 125 = 100 + 25 works, and a little more experimentation shows that no smaller cube works. Thus 2? = 125. For a full discussion of this Diophantine equation, see Liff. The calculations for this problem are relatively simple if we use the definition of logsx directly: gp = gl-2 + 102100] [logsv2} _ [stoen2]t2 + 10g2100) -1 + #1oge100 = [vay{-2 + loge100] _ (92)[-2 + 10g2100] = 2182 4. es. Letting « = y= 1, 2, 3, etc., we have £2) = FO +1) = FG) + FO) = 2f0) £03) = F@ + Y = fF) + FO) = 370) FC0) = 9 + 1) = F(9) + FA) = 10F() = 30. In fact, we can show that for all rational numbers q, f(q) = 3q: 1, For natural numbers , we can use an induction based on the above. 2. f(0) = FCO + 0) = F(0) + F(0) = 2f(0), so f(0) = 0. 3. (0) = fC) + f(-1) = 3 + F(-1), so f(-1) = -3. Another induction gives the result for all negative integers. 4. For navural nonbers ny A(@) = 5=s(b+ be. 2) Q), 1 1 imi f so f(4)= 3-2. a simitar argunent holds for negative integers. 5. Finally, any rational number q can be expressed as os 8 = on a> 0andp #0. Then ¢(2) r(F* wok yee, aa eeey, b -124- 15. 16. 7. 18. Solutions—Fall 1978 (Continued) If we assume in addition that f(«) is continuous, the result holds also for all real 2. The larger side lies opposite the larger angle. By the law of sines (See 3.4), 2 n(® + 60°) _ sin 6 cos 60° + cos @ sin 60° 1, V3 To Ge ye = mB ces Gor Ge sin OOP Ft For 0. Hence cot 8 = Y3 and 6 = 30°. The largest angle of the triangle is thus 90°, (In competition, the answer can easily be guessed.) ax There can be at most one unpaired sock of each color, so if Mr. Smith packs 13 socks, at most 6 will be unpaired. Also, since the total number of socks is odd, the number of unpaired socks must also be odd and hence there cannot be more than 5 unpaired socks. Twelve socks may not be enough, since if he takes one of each color plus any 3 matching pairs he will have 12 socks, but only 3 matching pairs. The answer is 13. Method 1: Labelling the points as in the diagram, we see that B+ AF = AD + Ai! = area (rectangle ABEF) and AC + 4G = area (square ACEG). Also, AB = 2. On the other hand, area (rectangle ABEF) = 4+ area(triangle ABO) = 4+ } + 04 * OB sin angle BOA 4+ 1+ 1+ sin 45° = 72, and area (square ACEG) = 3+ AE + 0G = 2. Hence the required product is (2)? + 2+2= 8 Method 2: We can solve this problem nore generally using complex Tumbers- If n points are spaced equally about the unit circle with center at 0, and one of the points is zy = 1+ 07, the other points are at x, = cis ™™ for x= 1 ton - 1. By Delloivre's theorem (1.6), 2 = 1 for each k, and all the points aside from -125- Solutions—Fall 1978 18. 19. 20, (Continued) ,, xo satisfy 2) = 0, By the factor theorem (1.311), the left side must factor into (#-#1)(@ - #2) ... (@ - =, On the other hand, long division shows this same polynomial to be fel ne? x +2 +... ++ 1. Taking absolute values, we have (for pel ne any 2): |e - mille - eal. [x Jet aaa, eas]. Setting # = 1 shows [1 - 2|[1 - 22[|1-as]-..[1- 2, = \L +1414... 61] =n. ‘The expression on the left is the required product. See also The Hungarian Problem Book I (1899 Competition), where the present Situation is connected with the topic of Chebyshev Polynomials. Four points form a parallelogram if and only if the sums of corresponding coordinates of opposite vertices are equal. If A = (1,1), B = (3,5), and C= (-1,4), there are three ways we can choose a pair of vertices to be opposite: Choosing A and 8 opposite puts the fourth vertex at (5,2). Choosing A and C opposite puts the fourth vertex at (-3,0). Choosing B and C opposite puts the fourth vertex at (1,8). If @ is the man's federal tax (in dollars) and y is his state tax (also in dollars), then: # = .2(9800 - y) y = .1(9800 - =). These lead to the solution (#,y) = (180,800). This problen can most easily be solved by trial and error. Two ways of achieving the result z*! in seven multiplications are piven below: a, 2%, 2, 28, a, 2%, 239, 23? ey 22, 2%, 28, 0, 28, 227, A "tree" diagram, tracing all possible paths, can be constructed which will show that seven multiplications is indeed the smallest number. For more on this rather complicated but interesting problem, see Knuth, Vol. IT, p. 421. -126- 22. 23. 24. Solutions—Fall 1978 Cubing both sides gives (@-a +@-+3VEOE-D | at) fa-e, But the original equation says Ve- a+ Va-b = Va-, so we have have (¢ - a) + (© - b) +3 V@@-a)@-Dy@-e)=2-cor 3V@-~ADe-He- )V--c-cra+b. At this point, it is not hard to see that cubing both sides will give a cubic equation in z (but not an identity!) which can have no more than 3 roots, real or complex. To see that 3 roots are possible, take for example a = Oat, We get: SVG? a) =n, Dele? = Ie ne bre” 27m fg, 26e" = 21a) = 0 and the 3 real roots are = = % We can generalize the problem just as easily as solving the special case given. Let @ be the nunber of lattice points on the graph of || + |y| b=0, =k, Let x, y > 0. For each integral value of « < k, (@, k - a) gives a lattice point solution to |z| + |y| =k. There are k - 1 such solutions, Reflecting in the axes, and counting also the vertices of the figure, shows that a, = 4(k- 1) + 4= 4k and a, = 1. nol nel Hence Ea,=1+ 5 4k 0 i 1 1 + In(n - 1) (see 1.53) =n? - n+. This sum gives the number of lattice points such that [zl + |y| b>c¢>0. Then ab=a+b+e< 3a, so ab < 3a and b < 3, If b = 1, there is no possibility for c. Hence b = 2, ¢= 1, and a= We have, with a little "hindsight": 2 cos?30° - 1 = cos(2 + 309 cos 60° (see 4.21, 4,22), so 1 + cos 60° = 2 cos*30°, Also, sin 60° = 2 sin 30° cos 30°. Hence the given expression is equal to 2 cos? 30° + 2isin 30° cos 30° = 2(cos 30°) (cos 30° + isin 30°) = ¥3 cis 30°. By DeMoivre's theorem (1.6}, (1 + cos 60° +Zsin 60°)! = 3®cisl2 + 30° = 3®cis 360° = 3° + 0c, and the imaginary part is 0. A quicker solution to this problem is furnished by looking at it geometrically. On the complex plane, let A represent 1, 5 represent cos 60° +isin 60°, and C represent 1 + cos 60° +isin 60°. Since complex numbers add as vectors, Oi + OB = OC, and OBCA is a parallelogram. Since |@| = |B] angle COA = 50°, and upon raising the number C to the 12th power, the amplitude will be 12 + 30° = 360° and the result will be reel. 8B c 1, OBCA is a rhombus. Hence, -129- Solutions—Spring 1979 Solutions—Spri Le 1g 1979 Let # = 1ogs3, y = logs4. Then we have 2” = 3, or 2? = 37/, Also, 9 = 4, or 2? = 3°. Hence 2/x = 2y, or zy = 1. Since PA + PC is constant (see diagram 1), we need only maximize the sum PB + PD, A good guess in any minimum or maximum problem is an extreme point, which here would be one of the vertices of the trapezoid. It is clear that if choosing a vertex will give the maximal sum, then Y is the best possible vertex to choose. Let us place point P at Y, so that P2 = 0. To compute PD, we extend WZ and X¥ to intersect at V. Then, as Wx:YZ = 3, we find that VW = 2, VX = 3, and we can use the law of cosines in triangle WX to find angle Wid: 7=9+4-2+ 2+ 3+ cos 6 -12 cos 8 = -6, cos @= 2, and 9 = 60°. Hence PD:V¥ = PD/6 = sin 60° = 73/2 and PD = 3V3. Also, if VF is the altitude to BF in triangle ZvY, vP = 2(PA'+ PC).' Computing the area of triangle 2VY in two ways, we have (See 3.64): 3+ 2V + VY + sin 60° = 2 2y + VP Be 4+ eBags a+ 20RA + PO) 6/3 = 277(PA + PC) mere 38. 34 WF and Pf + PB + PC + PD = 33+ St (or equivalent). y w, x 6 r z ui? fi Y Diagran 2 Diagram 2 Diagram 3 Let us now show that placing P at point ¥ will indeed result in the maximal sum PB + PD. We first need to prove that if R is any point on the base of isosceles triangle STU (see diagram 2), then the sum of the perpendiculars from F to S? and SU is constant. This can be shown by computing the area of triangle S7U in two ways. We use absolute value to denote area. On the one hand, -130- (Continued) |stu] = 3+ uy + sr, On the other hand, |s7u| = |s7a| + |sRU| 2+ eR sP+d+ Are su = 3(87) (HR + RI) Hence H¥ + RI is constant and is equal to UY (the altitude to the Jeg of the isosceles triangle). In the original problem, we can draw a variable line FG (see diagram 3) so that triangle YFG is isosceles. For any point P on FG, the sum of the perpendiculars P3 + PD is equal to the perpendicular from G to line VZ (the altitude to the leg of isosceles triangle YFG). We need to choose line FG so that this altitude is maximal. This will clearly occur when FG passes through point ¥ (so that P and ¥ must coincide). Note that for any choice of P, we can move in a direction parallel to that of FG and keep the sum PB + PD constant. The line FG is called a "level line" for the function PB + PD. Such level lines (or "level curves") are often useful in finding maxima or minima. In this problem, the argument given also shows that the minimum value of PZ + PD occurs when P is at vertex W. The actual total cost of all the items was $28.97 - $.65 = $28.52. The largest possible correct cost is (25)($1.10) = $27.50. Hence the smallest possible error is the difference, or $.82. Since the maximum amount of each error is .09, the number of errors is more than 82/9, or at least 10. We have sin 20° + sin 40°=2 sin 30° cos 10° (see 4.321) =cos 10° = sin 80°. P@,ysa) = ys(e - 3) + (2 - $). Both (@ - 3) and (20 - 5) attain maxima for 7 = 2, But then « - 3 = -1 < 0, so we want the factor ys to be minimized. This is achieved if y = 2 = 1, and f(2,1,1) =2-344-5=-2, That the maximum will be achieved at a corner of the cube can be predicted by noting that the expression for f(«,y,s) is linear in each variable. For every $1 difference in price, Joe will go down 10¢, while Moe responds by going up 9¢. Hence the concessions are in the ratio 10:9, and the price will ultimately be reduced by 12¢$200) to $A94.75, -131- Solutions—Spring 1979 10. ul. 12. Writing f(z) = az* and 2, we find that + be +c, and substituting in turn 2 = -1,1, a- bee =-2 a+ bee = 0 4a+ bec = 7, Solving simultaneously gives (a,b,¢) = (2,1,-3), and y = f(3) = 18. 2_. siny , cosy | sin’y + cos’y 1 ‘cost * cosy * siny ~ (sin y)(cos y) * Siny cosy We have or cos 2r = 2 sin y cos y = sin 2y. ‘Thus the least positive value for 2x + 2y is 90°, and for z + y it is 45° (see 4.11 and 4.21). ‘The first man occupies the following series of places in successive maneuvers: 1,3,5,7,9,11,12,13,14,15,16,17,18,19,20,2,4,6,8,10,1. ‘Thus after 20 maneuvers the first man is back in place. Note that the others are also back in their original places. All the men go through the same cycle, starting at a different position. For odd k, #* +1 is divisible by « +1 (see 1.34). We have (2°19 - 1) = (2195 + 192198 - 1) and 225 + 1 = (2'8)7 41 = (221)8 + 1 = (235)? +1, The four solutions are 2'°5 + 1, 241, 2241, 28 41. 60_conts/gallon _ 60 ne 3 ; Te now costs $-cents/gation . 6 conts/nite » $ cents/aite to run the car. The man needs a cost of 20cents/gallon _ 70 zniles/galton == conts/ nile = 3 cents/mile. Hence 140 = Se and z= 28. He needs to get 4 more miles per gallon. It can be shown that any simple planar polygon of n sides has at least ” - 3 interior diagonals. Since the total number of diagonals a (") = tT) ee is (3) -m there can be at most (3) - an +5 exterior diagonals. The first diagram shows an example of an n-gon where this maximum is actually achieved. For m = 12, this number is 45. To count the minimum number of interior diagonals of a simple n-gon, let us first prove that any such figure has at least one interior diagonal. Suppose the vertices of the figure, in order, are Ai, Aa, see A, (See diagram 2). We can choose any vertex A, and look at the angle A, ,A,A,,,. If we find any vertex A; of the polygon inside this angle, then there must be a closest such vertex to 4,, and then 4%, is an interior diagonal. If there is no such vertex A;, then 7,_\j,, is an interior diagonal. -152- 12. 13. 4. Solutions—Spring 1979 (Continued) n-2 sides Ak Hence any polygon must have at least one interior diagonal. We can now prove by mathematical induction that a polygon of n sides must have at least m - 3 interior diagonals. For n= 4, the statement merely asserts the existence of one interior diagonal, which we have shown above. Suppose that for n= 4, 5, 6) «++, k we know that an n-gon must have n - 3 interior diagonals. To complete the induction we must show that any polygon of k +1 sides must have at least k + 1 - 3 = k - 2 interior diagonals. We proved above that even our (k + 1)-gon has at least one interior diagonal. If we draw this in, we will separate the (k + 1)-gon into two polygons (diagram 3). Suppose one of these has p sides and the other has q sides. Then p +q =k + 3 (we count the diagonal twice as a side) and the induction hypothesis applies for these Two polygons, so they must have at least p - 3 and q - 3 interior diagonals respectively. Now any interior diagonal of one of the ‘smaller" polygons is an interior diagonal of our (k + 1)-gon, so in all we must have at least (p - 3) + (q- 3) +#1=p+q+1-6 k+ 3-5 =k - 2 interior diagonals for the (k + 1)-gon. This completes the induction. Although this reasoning is difficult to complete in the time limit given for the problem, a little experimentation can quickly lead fone to the correct conjecture. Dividing the first equation by the second, we find z Substituting, we have 24 = oe = 18, y = 64, and x = 576. aly One can prove that if a rectangle has four vertices on the sides of a given square and no side parallel to a side of a given square, then either the rectangle is itself a square, or each of its sides meets a side of the given square at right angles. Since the rectangle in this problem is not a square, we have (see diagram) AX = PX = XS, PQ = RS = XY, QY = RY = YC, so that the perimeter of the rectangle is PQ + QY + YR + RS + SY + XP = XY + YC + YC + XY + AY + AX = 2(AC), or 272. Since one of its 7153- Solutions—Spring 1979 14. 1s. (Continued) sides has length 1, the other side must have length JZ - 1. In general, the perimeter of a rectangle inscribed in a square as shown is twice a diagonal of the square. It is not hard to see that the largest rectangle of the type described in the problem must have one vertex on each side of the square. For if one vertex is inside the square, we can rotate the rectangle about a different vertex. Then three vertices are inside the square, and a small dilation will give us a larger rectangle than the one we started with, which still fits into the square. A slightly longer argument will show that the sides of our rectangle must meet the sides of the square at 45° angles. For suppose RD = a, GC = b (see diagram). Without loss of generality We may assume a>, It is not hard to see that triangles SRD and ORC are similar, so we have: 2 2b. Be — }alagrest 1 1 1 J2a- 2] =42(a-4) (since we assumed a > $). so either b = a or b=1- a. The first alternative implies that PQ = QR, so that the rectangle is a square. The second implies that PQ’and 4B meet at an angle of 45°. Atak o RY cs Ne Using absolute value for area, we have |48c| = oacl - Joas] ~ Jozc|. Now Joac| = $+ 6 + 4 sin angle aoc = 12 sin 120° = 63 (see 5.64), Similarly, |oa8| = 373 and |oBc] = 2/3. Hence |aBe| = 6v5 - v3 = V5. -134- 15 16. 7. 18. Solutions—Spring 1979 (Continued) Let a = a + bY2, where a and b are integers, and let us look at the function f(a) = f(a + bY2) = a - bv2 (the "irrational conjugate" of intermediate algebra). Some easy, if tedious, calculations show that if a and @ are numbers of the form a + b/Z, then f(a + 8) = fla) + FCB) FC08) = F(0) FCB) a + f(a)’ is always an integer. Note that the second property implies that f(a”) = [f(#)]*. f(a®) = [f(0)]°, and in general that f(a”) = [f(a)]” for any natural number (a formal proof would use induction). If a = 1 + v2 (the number given in the problem), then f(a) = 1 - /2 and (1 + /3)% + (1 ~ 39% = a2 + [f(a)]®? = 09% + f(a"%) is an integer. Now -1 < 1 - 72 <0, so that -1< (1 - /2)°* < 0, and (1 + ¥3)9% + (1 ~ ¥2)99 = (72 + 19% - (YZ - 1)%° is the integer just below (1 + /2)°°. But this is exactly the W referred to in the problem. Hence N= (v2 + 1)°? - (v2 - 1)%° and r = (v2 - 1)°%, so that (W + rr = (2+ NZ - 1) = (V2 + DWF - 1%? = (2 - 1)°* = 1, The function f given above is called an auto- morphism for the ring of numbers of the form a + by2. For much more on this topic, see Herstein (Chapter 3, section 3). If we denote the three roots by 2, a, ax, then $+ a+ ar=a?=1, and a= 1, since a, being a root, must be real (see 1.4). Also, peokeaeh, men-2ord. a Seataersit atan 1 5 1 * * The roots are Z 1, 2. ‘The central angle subtended by a side of the 60-gon is 6°, while the same angle for a 30-gon is 12°. The area of the 60-gon is 60 times that of an isosceles triangle with legs » (the common radius) and vertex angle 6°, or 60+ 3r’ sin 6° (See 3.64). Similarly, the area of the 30-gon is 30+ 3? sin 12°. The required ratio is 2 sin 6° | __2 sin 6° ° Sin 12°“ Tsin 6 cos oF ~ SC 6° (see 4.21). -135- ‘Solutions—Spring 1979 18. 19. 20. ai. 22. (Continued) GOsides 30 sides ir ov fr pe 12° The expected waiting time at the stop for bus A is } + 8 = 4 minutes, s0 the expected value of the time until boarding a bus is 14 minutes. Going to line 3, the expected time is 5 + $+ 15 = 123 minutes, so the better choice is line 3, We have (1.4): P+ q+ r=p pq + qr + pp=q par =r From the first equation, q +” = 0, From the second, p(q + ¥) + ar =0+qr=qr=q. Since q #0, we must have r= 1.” Then gq = and from the third equation, p = -1. We have sin x + cos x = 3, or sin*x + cos*x + 2(sin x) (cos =) 2142 sina cos «= i, (see 4.11) so sin z cos @ =-3. Now (1.321) sin®x + cos*e = (sin x + cos «)(sin’x - sin x cos x + cos*z) = 2 ~ sine cos 2) = H( + 3) ete ~ 8 16 We will show that angle BFC = 90°, so that BF? + CF? = BC? = 36. Suppose 5F and CD intersect at X. “The angles of quadrilateral ADIE must add up to 360°. Hence, if angle 4 = a°, a + 80 + angle DIE +100 = 360 and angle DYE ='180 - a. Then angle BYC = angle Diz = 180 - a, and in triangle BXC, angie CBX + angle BCX + 180 - a = 180, or a = angle CBX + angle BC. Nisa,’ angle DCA + angle A = 100° (since angle ADC = 100° is an exterior angle in triangle ACD), so that angle DCF = 50° - 3a Similarly, angle BEF = 40° - 3a. Now we look at triangle BCF: angle FBC’ + angle CBF = angle CBX + angle EBF + angle BCY + angle DCF = a + 50 - 3a + 40 - 3a = 90°. Thus angle BFC = 90° and the : proof is complete. -136- 22. 23. 24. 25. 26. Solutions—Spring 1979 (Continued) A o é 100 c 6 8 tet y= 2/"_ then y? = 2", and the given equation can be written as y? - dy + 3 = 0, or (y - Ny - 3) = 0. Solving for @ gives © = lore = 3%, Let us call the smallest distance between two points in the figure a "unit." We first count triangles whose sides are parallel to the sides of the large triangle. There are nine such triangles with unit side, three such triangles with sides 2 units, and one such triangle with side 3 units. Hence there are 15 triangles with sides parallel to those of the larger triangle. But there are two more triangles whose sides are not parallel to those of the larger triangle. These are shown in the figure. Let us look at the possible vowels, represented by + 7. The numerical positions of A,E,1,0,U in the alphabet are, respectively, 1,5,9,15,21. ‘Thus only z +7 = 9, 15, or 21 are possible. For 2 = 2 the message is BIHG. For x= 8 it is HONM, and for x = 14 it is NUTS. (For a discussion of the anecdote in American history on which the problem is based, see Rawson (p. 194)). In the diagram, the coordinates of P are (h,k) and those of B are (9,k). Since PA = PB, and P is on the given curve, we have K?’= 20k - 100, which’ leads to k = 10. (ae 7 KI -137- A * Solutions—Fall 1979 27. 28. 29, 30. The given expre = minGe,y,2). ion is equal tom +y+2-2-+a-ytera If the radius of a cizele is R, then the length of a chord of a central angle of measure 26 is given by 2% sin 8 (see diagram). In the present case, we have 2f sin 6 = 4, 2R sin 36 = 11, where the given arcs are 20 and 68. Now sin 30 = 3 sin ® - 4 sin?@ (4.51), 6 : soit-$. 3, reading to R= 8 or -8. the radius of the circle is 8. ‘The numbers in question are of the form 301 + 1. Since 3000 < 30m + 1 < 4000, we have 99 < n < 133 and there are 34 such numbers Forming an arithmetic progression. The first and last are 3001 and 3991, so formula 1,512 shows their sum to be 118864. ‘The last game must be won by the Mets, Hence the Mets may win any three of the first five games and also the last game. The probabiticy of this event is (8)(3)(2)(8) = .20736. Solutions—Fall 1979 i We have: 2+ 59 . vives + 15 @ + 39 = 3 VI08e + 39 27(108x) (81) (36) © = (9)(6) = 54. 8 8 We haves 1004 + 108 + 0 Torso 9, which leads to 54 + 58 - 4c = If 5(4 + B) = 4c then C= 5, so 4+ B= 4. The maximum occurs where 4 = 4 and B = 0, or y = 405. -138- Solutions—Fall 1979 We have: 80 + 45= 3600, so log 36 = 1og S60? = tog 3600 - log 100 =a+b-2 Using absolute value to denote area, laDF| + 3(4D) (AF)sin 60° = 3(1) (V3) (V3/2) = 3/4 (see 3.64), (+ ¥3)7V5 _ 6 + av3 Cee. oes, so [pee] = |4ac| - 9/4. then |48c| Z =3+ 43 a so | D8 + and (a,b) =(-3, 1). See probien 34, spring 1976. - TT Sint A = - V0 sin AC ~ sin ay = - vI-¥ 24725yC1 ~ 24725) cos A = {teed ee = [= $. Gee 4.11, 4.26.) If we let a = E41 + VIE , we are looking for solutions to the equation a? + a? +a = 14. One real solution that we can get by inspection is @ = 2. From this we find: ViETT + VOTE = 2, or FETT = 2- VTS. Squaring and simplifying gives us 4x - 5 = -4/2 >=. Squaring again yields 16x? - 242 - 7 = (4e - 7)(4e +1) = 0, ande=F,-7. A check shows the value 2 = 7 to be extraneous. ain -159- Solutions—Fall 1979 6 10. (Continued) ‘The equation a? + a? + a - 14 also has two complex roots: a? +a? +a - 14 = (a - 2)(a* + 3a + 7) = 0 when a = 2 or a= 3+ 2/19/2, We can show that these complex roots do not lead to real values for 2. For suppose /P+ 1+ /T-a=p+qi, where g #0, Then, as the square root of a real number is either positive, zero, or "pure" imaginary, either p = 0, or p = /3e +1 and qi = YE=@, or p = /E~@ and gi = J@+T. In each case, p20, which is not true of the complex roots we have found. Let x = the sixth term, Then 1 + 2 + 4 + 8 + 16 + 32 = 63, 63 = $1 + @) (see 1.512), and 2 = 20. thore axe (5) = 10 nays in which to choose the two munbers thet are to be in their correct places. ‘The other three numbers must be in incorrect places. The first of the three empty places can be filled in two ways and the remaining two in only one way. Since the total number of ways to fill the five spaces is 5!, the answer wor 21 sor 6" $ = log (1/2) (2/3) (3/4)... (99/100) = log 1/100 = -2. Without Ioss of generality, we may assume that both lines pass through the origin, that OP = 7, and that AP = 1 (see diagram). Then the Pythagorean theorem shows that OB = 7/2 and AO = Sv2. Using the law of cosines in triangle 4B, we find 36 = 50 + 98 = 140 cos 28, so cos 26 = 112/140 = 4/5.’ Using 4.24, sin @ cos 26 10 3yT0. 1 / A= ge8 28 = Gp. Then cos o = 370 and tan 8 = 5. tan The slope of line L is tan(a + 8) = 7 >. (4.341) A similar argument will show that -140- 10. lL. 12. 13. se 1s. Solutions—Fall 1979 (Continued) L 2 Asset Lis ista ae. ASSOC 5 tan B+ 1 = tan (225% - A) +1 (4.342) = TEESE +1, so the product is tian ae (LEH) a. w+ y= (+ y)? - Sey(e + y). Also, 2ny = (e+ y)? - (a? + y*). 3ab - a Then ¢ = a? - 3a * 7 or equivalent equation. Let [2] denote the greatest integer not exceeding x. There are [343/7] = 49 factors of 7 in 343!. There are (343/77] + [343/72] 7 + 1 additional factors of 7, since each multiple of 7? or 7° contributes more 7's. The sum is 49 +7 +1 = 57. +? Using formula 4.341, we get PZ, 7 ora’ + = 0, so 27(a? +1) = 0, so the single real solution is = = 0. 2 4 sin @ and sin © is maximum when 6 = 90°. This gives a=]. (see 3.64) -141- Solutions—Fall 1979 16. 7. 18. 19. 20. (= + Y _ 4 ang 22 WE-YD 29 so - by z- y +d Gand “Je yaa 7 0 ley) = 0 and (x - 3y)(x - y) = 0, and x = 3y satisfies both equations. Setting « - y and z + y equal to zero leads only to z= y= 0, which is included in the above. Thus all simultaneous solutions are given by x = 3y, PROVIDED: 2-%y+1#0 2m - 3y- 240 By +14 6y - 3y- 240 ye y # 2/3 a#-3 z#2 The number of unit squares touching the sides is 2 + 2y - 4. This provides for half the area of the rectangular region. Thus Bay = 20 + 2y - 4, or ay - de - dy + 16 = (2 - 4)(y - 4) The only factors of 8 are 1, 2, 4, and 8. Therefore, « - 4 2, 4, or 8, while y - 4 is equal to 8, 4, 2, or 1, respectively. Thus (2.y) = (5,12), (6,8), (8,6) or (12,5), and zy = 48 or 60. We have P(x) = @i(a)(e - 2) +2 (a) P(e) = @2(e)(e + 2) - 2 Gi) We need to find a and } where P(e) = @a(w)(@? - 4) + ar +b (iii) From (i) and (iii), P(2) = 2= 2a +b. From (ii) and (iii), P(-2) = -2 = -20 +5 Solving simultaneously, we get 2 = 1, b = 0, so the remainder is ‘The greatest power of § to divide 127! is [122] + [222] + [ZF = 25 +5 +1-= 31. A much higher power of 2 divides 127!. Therefore, 10°? divides 127! to make it end in 31 zeroes. wet y+ + Yh. then XZ. —v ys WE Squaring leads to 2x? - 5x + 2 = 0, which gives « -142- 2. 22. 23. Solutions—Fall 1979 We have: 1 2 2! 6 (tar) - Safar 6n?(m - 1)? = Sn?(n? - 1). Since n #0, #1, 6(n - 1) = S(n +1), andn= 11. The given equation is sinte 4 sine costa ~ os = +1 Multiplying by cos’, we get: sin’e ~ 4(sin 2) (cos «) + cos’e = 0, or 1 - 4(sin x)(cos 2) = 0, 5. (See 4.21.) or sin 2x We have: mya? = 27+ 3 ayiz = 33+ 2 2°3 aya Multiplying the above together gives: ax'y'a5 = 35+ 25, or ays = 6. Dividing each equation by 2yz = 6, we have: a? = 4, y? = 9, 2 = 1, and all three can be positive or two can be negative, giving (1,3,2), (-1,3,-2), (-1,-3,2), and (1,-3,-2). Since FG = GH and m angle GF# = 60, triangle FGH is equilateral. With # and 9 as in the diagram, in triangle AGF, =" i555 = pags implies x? (See 3.4.) The required ratio is (avs _ x? 6(87/4) V3 -143- Solutions—Fall 1979 2s. 26. 27. 28. 29. 30. Wet be the base. Since 2 W produces £ = 3.333 ..... Then -333 ... , multiplying both sides by a 3, so = 16. Alternatively, we can use 1.523. Let a = ve+y and b = ve-¥y (note that a> 0 and b> 0). Then a? +a= 12 and b? +b = 6, or (a - 3)(a + 4) = 0 (b- 2) +3) =0, so a=3andb=2. Therefore, 2+y=9 By = 4, soz = 15/2, y= 5/2. The probabilities of success on the first question for the four students are 3, 4,2, and}. The probability that they all : is 2 3.3, 9° fail to answer the First question correctly is 2+ 2+ 3. $= 9 au : 9. 91 Thus the probability of at least one success is 1 - qhy= qhy- The given equation is equivalent to log logy? + logy4 + logy6 + logy8 + log,10 = logy2 + 4° 6 + 8+ 10, sow = 3840. (See 5.12.) -3 + V3. If 2 = -1, then y? + 2y = 0 and y = 0 or -2. The points which are furthest apart are (-1,0) and (1,-3-¥3). If = 1, then y? + 6y +6 = 0 andy The distance between them is /16 + 6/3 , sop + q = 22. By the "extended law of sines" (see 3.4), (since BH = sin 4), This gives » so that and (a,b) = (4,-1). -144- Solutions—Spring 1980 There are 72 = 0, so (a - 2)(a* + 2a + 3) Letting a = vz, to real solutions for x. + VB - Vii) =~ L+ sina. 30. (Continued) dB ' A c # = Solutions—Spring 1980 1. There are 5! = 120 seating arrangements altogether. (4+ 2) + 3+ 2+ 1= 48 arrangements in which the Bronx and Canton coaches sit together. Therefore, there are 120 - 48 arrangenents as required in the problen. 2. ei tat det + de? 4 22 fe? +1) + 22? +1)? (2 + N@ + 2@7 4) = @ + DOs +242. If (a - 1)a(a + 1) = 6, then a? - a - 6 = 0, which has one real and two complex solutions. the complex solutions for a do not lead Therefore, we need 1+ 2+ 3 = 6, anda=2, soa= 4, . Rationatizing the denominators, we have WE - V+ WE- D+ VT- D+. This is called a "telescoping sum." 5. Squaring: sin? 40° + cos? 40° + 2(sin 40°) (cos 40°) = (See 4.21.) 1+ sin 80° = 1+ sing = 80° 4 Piers (5- x)? sonst. -145- Since two tangents drawn to a circle from the same external point are equal in length, BP = BC = 4 so AT = 1, In right triangle ATO, Solutions—Spring 1980 6. 10. ll. 12. (Continued) I£ # represents the number of students, x(x - 1) soz = 30. 870 = 30+ 29, a ~ 2ct - 120 = (e* - 12)(@4 + 10) = 0, Solving for x”, we find that /I2 is the only positive real value. Thus, area = 22+ ¥5 23 7 z° A number that is divisible by 22 is divisible by both 11 and 2. Hence, if the number is 100h + l0¢ + u, u must be even. For u=2, 4, 6, 8, ¢ must be 9, 7, 5, 3 respectively. Using criteria for division by 11, we find 792, 374, 638 as the only possible numbers. By 1,81 and 1.82, the roots of the polynomial are */-7 and +73, Hence, by 1.311, the polynomial must be divisible by z* + 7 and x? = 3, or by 2" + = 21, Since its coefficients have no conmon’ factor greater than 1, and the polynomial is of degree 4, it can only be a + 4x* - 21, Adding: E*Hy = 5. dividing, 18 F. ory = 2x. Substitution gives 2+ 5, soe by y= 2 +0 yields no sotution). Triangle 0:0203 is a 3-4-5 right triangle. Using absolute value for area, |010203| = 6, |AQsC| = 3+ 3? sin a = (9/2)(3/5) = 27/10. |B02c| = 4 + 27 sin B = (4/2)(4/5) = 16/10. (See 3.64.) [4oyB| = = 5/10, [4Bcl = |0,0203| - []40sc| + |B020| + |40,8[] = 6 - (27 + 16 + 5)/10 = 12/10 = 6/5. See problem 34, Spring 1976. -146- 12. 13. 14. 15. 16. Solutions—Spring 1980 (Continved) Since log b = —b—, logo + logo# = 4, so logee = 2 and w= 2° = 4 a? * Toa (See 5.12.) We could use DeMoivre's theorem (1.6) and convert back to rectangular form, or: 4" + 1 = dah + an? 41 - ae? = 0 (22? + 1)? - (2m)? = 0 (2x? + Qn + 1)(2n? - 2 +1) = 0 Setting each factor equal to zero and using the quadratic formula gives 2 = 23230, Let r = the radius. Then (7 - ¥)2 + 36 = (P+ 5)2, The outcome of choosing from urn B is either two balls of different colors (with probability 2/3) or two black balls (with probability 1/3). In the first instance, the probability of drawing a white ball from urn A becones 3/4, and in the second instance it becones 1/2. Thus the probability of drawing a white ball from urn 4 is (2/3) (3/4) + (1/3) (1/2) = (1/2) + (1/6) = 2/3. A tree diagram makes this solution quite clear: -147- Solutions—Spring 1980 16. (Continued) 8 A nite . rae re Jwhite, 4 Tan < I~ black, a white 2 blk << Zblack 17. cos*A + 2 sin A = 1 - sin?A + 2 sin A =1 (see 4,11), so sin A(2 ~ sin A) = 0, and A= 0°, 180°. 18. If the sides are 2x - y, 2, and 2x + y, the semiperimeter is 3x. Using 3.61 and 3.62, (3x)(3) = 126, so. = 14. From Hero's formula 42 * 14+ (14 + y)(14 - y) = 1267, or 196 - y? = 27, or y= 13. Thus 2c - y = 15. 19. For any value of n, n° - 1= (n- I)(n* 43 4m? 441), so that n> ~ 1 is not prime unless one of these factors equals 1. If n is a natural number, this happens only when n = 2. (See 1.33.) 20. ‘The graph of y = f(x) must cross the z-axis once between -4 and 0 and again between 0 and 4. By 1.81 the third root must also be real. See the "Intermediate Value Theorem" in any calculus book. 21. If the numbers are x and y, we have: {5H vay = 50, sow +y = 2%] = 100, or (VE - V)* = 100, and VE - JG = 10. 22. We have: [Q(x)]? - [P()]? = 2? + 1, or [Q(w) + P(e) + [@(e) - P(e)] = (22? + 1) + (2). So either: @ Gi) Qe) + Pe) = 27 +1 or Me) + Pe) = 1 Qe) - P@) = 1 Qe) - Pe) = 2? +1 and and Q(z) = 22 +1, P(e) = 2. Q@) = x? +1, P(z) = -2? The solution obtained from (ii), however, does not fit the conditions of the problem. Notice how closely the arithmetic of polynomials resembles the arithmetic of integers. -148- 23, 24. 25. 26. 27. 28. Solutions—Spring 1980 I£ the roots of 4x? + ax + b = 0 are r and e, then r so the equation is (= + )fe - = 0 (see 1.4). We have: (4) +a2+...+a )4a = nla Gta, = De, 1 so that a, = for n> 1. n Thus az = (1/3)(1/2) = 1/(3*2), as = (2/4) [1/(3+2)] 1/(4*3), ay = (3/5) [1/(5+4)] = 1/(4+5), and as9 = 1/(50*51), so that: a tart... tas = 50 + ts There are three ways to color card 1. Then there are two possibilities for coloring card two. This leaves only one color for card three, and the coloring of the first three cards (or even the first two) then determines the color of all the others. Thus there are 3+ 2+ 1 = 6 colorings altogether. 721 = 1°721 = 7°103, soz? - y3 = (@ - y) (a? + zy + y2) = 1+721 = 7103 (see 1.322). By inspection, if z, y > 0, the second factor on the left is the larger, so either: @ or aiy e-yel a-ys7 2 sy ey? = 721 a? + ay + y? = 103. Substitution leads to: 2 ay - 240 = yo + Ty - 18 =0 y= 1S, -16 y= 2, -9 The ordered pairs of positive integers are (16, 15) and (9, 2). Logs (logs (logez)) = 0 when logs(logaz) = 1, or logew = 3, or z= 8. let $ = area of triangle ABC, x = AB = DC, and y y(@ + y) = 2%, or x? - gy - y? = 0, s0(F y’ aa ya BD, Then Solving for Fi and rejecting the negative root gives = -149- Solutions—Spring 1980 28. 29. 30. (Continued) Since the triangles have equal altitudes to line AD, Area of triangle ABC |S 2 1+ 95 nie tena Area of triangle CBD ws ¥ 2 . Ps Teads to S = 5+ V5, and (a,b) = (5,1). A cK! Let the original triangle be ABC and let y = k intersect AB, AC, and the y-axis at D, E, and P respectively. Then triangle ADE ~ triangle BC, and since their areas are in the ratio 1:2, corresponding altitudes are in the ratio 1:72. Hence, if 0 is the origin, AP:A0 = 1:V2, or (6 - K)/6 = 1/V2, so that k = 6 - 3V2. Note: In the limit, as BC approaches 0 in length, the line bisecting the area of the triangle does not approach the perpendicular bisector of 7 ” no a é 6 0 © % If the man has n children, then (n + 77)? - n* = 12(n +n? - 3), or 2n? - 1ln? - 12n + 36 = 0. Using the rational root theorem (1.7), we find (n - 6)(2n? + - 6) = (nm - 6)(2n - 3)(1 + 2) = 0. Only n = 6 satisfies the conditions of the problem. We can avoid the use of (1.7) by writing the original equation in the form (n + n°)? - 12(n + n*) + 36 = (n? +n - 6)? = n*, so that nt n = 6 = tn -150- Solutions—Fall 1980 ‘Solutions—Fall 1980 Cost = m+ (k= 12)e m+ ks - 128. If we let y = @ +3, then z = y - 3, and P(y) = (y - 3)? + Tly - 3) +4=y? +y ~ 8, so that P(x) =a? +2 ~- 8, and (a,b,c) = (1,1,-8). Altogether, 6 + 5/2 = 15 games were played. Clearly, the winning ‘team could not have won more than 5 games. If this team had won four games (or fewer), then two teams would have to win the same number of games. Hence, the winning team must have won five games. By the law of cosines, AC? = 7? + 8° - 2+ 7+ 8+ cos 120°. BD? = 7? + 87 - 2+ 7+ 8+ cos 60°. The difference is4+7+ 8 + cos 60° = 112. Compare 3.84. 8 a A c 8 o By trial and error, 17= 11+ 3+3=5+5+723+747. For an interesting discussion of the sense in which the statement about odd integers is "known," see, for example, Sierpinski. Let = = Log, 19525 V2. then (she) = 25 + as'/? = 25/?, -on | gs/3 8 _8 sa 5°?, so xe = 8, anda =-8. F(@) = Oe) + fe =a) + R= FQ) + Sfx - a] +R, The remainder is not affected (1). Therefore, (gn) =(4, 1). Let AE intersect FG at Y, and let FG =e, so BY = a/3/2. Clearly AB = 4. From similar triangles AYG, ABC, we have: 4 _ 4 - 83/2 ov3 3 e/2 e 4e = 24 - 50/3, 0 = 24/(4 + 3V3) = SCA - 3V3), and (a,b) = (44-3). -151- Solutions—Fall 1980 10. ue 12. 13. (Continued) eyes © n= (a + 2/)(b + 2/a) = ab + 4+ A/ab. But ab + 4/q, or equivalent. We have a+ (a +2) + (a+ 4) +... + (a+ 2n- 2) = 105, or, summing the arithmetic progression, (n/2)(2a + 2n - 2) n(a*+n-1) = 105= 3+ 5+ 7. ‘Since both n anda +n - 1 are integers, we can have n= 1, 3, 5, 7, 15, 21, 35, 105 only. The last four lead to negative Values for a. The other values of m lead to a= 105, 33, 17, 9, respectively. Hence the smallest possible valve fora is 9. (See 1.512.) Let log # = 4, logy = B, so log ay =A +B, Then log (4 +B) log 4 + log B = log 4B, or A+B = AB, and B= A/(A - 1). For integer solutions, we need A - 1 to divide 4, sod = 8 = 2, and (,y) = (100,100). Extend AM its own length to D. AM = 770, so AD = 2V70. Then ACDB is a parallelogran, so (see problem 4, Fall 1980) AD? + BC? 2aB* + 24c*, (See 3.84.) If BC = x, then x? + 280 = 296, so BC = VIB = 32. D Using mass points, we can assign weights of 2, 1, 1 to A, B, C. respectively. Then F has a weight of 3, D of 2, and of 4. We find FE/EB = 1/3 and AP/FC = 1/2. The required sum is then 5/6. s@ xo 14. 15. 16. a, 18. ‘Solutions—Fall 1980 ‘The form of the numbers is 2” + 9" - 1 = 18"-1, Since the polynomial 2 - 1 always divides the polynomial =” - 1 without remainder, 17 = 18 - 1 must divide 18” - 1 for all n. To see that 17 is the greatest common divisor, take n= 2 and = 3. The two numbers 323 = 17 * 19 and $851 = 17 * 343 have no greater common divisor than 17, so 17 must be the greatest common divisor for the entire set (see 1.33). The least common multiple of 3, 4, 5, 6, and 7 is 420. Adding each of these numbers to the common multiple will produce multiples of the number added, so m= 423. A smaller value for m would make m- 3a smaller conmon multiple for the five numbers. Let D be the foot of the altitude fron B on AC. Then AD = ¢ cos A A+ a cos ae and DC = a cos 0. Hence S aie L€ the three angles are 3¢, 4z, and Sx, then 3x + ae and z = 15. The required ratio is Sn 8... sinoo" "3 Lop =a, and ow given areas, rh = 150, eh = 120, so rs a 3-4-§ right triangle. Let r= Se, tee 6 BAB = ae h. Then €rom the 34, and triangle OMC is 4z, and h = 3x. Then 2! =r-s8 =a, and CB = JOT? + TH? = /T0z? = 2/10. To find x, note that rh = 15m? = 150, so m* = 10 and CB = 10. -153- Solutions—Fall 1980 19, We have: sins = 2252, so sin’e = 1 - epste » cos ay of costs + cos = 1-0, and cosz= tS gue Lo <4, so the only possible value for cos = is aAr8 . 20. Suppose the runners meet at point R. Let PQ = d and QR = a. ‘Then, if B increases his speed by x MPH, we have the time equations: -d+a,,d+a_d-a 7 i = S59 ana G22 = S54 | Prom the first equation, we find iS 3 dta_9+e second we fin: ees » and from the second we find that $*2 = 242 , 21. We want © + y + 287 = 10414, sow ty = 10 and ay = 4. Thus we are maximizing the product of two numbers, given that their sum is constant (see 6.3). We would want to make the two numbers equal, but this is contrary to the conditions of the problem. Thus the best we can do is to let x = 6, y = 4, giving q = 96 (letting n= 4 and y = 6 gives the same answer). 2. Z||4G. Then if ap = 2a, ex = PSPC. 3, and since triangle BXP ~ triangle GDF, XF:FD = EX:D0 = 3:4, so OF:FD = 10:4 = 5:2, SG 3 4 € £ Gq . A 25. -1 < log # < 1 for .1 < # < 10, so possible values for # lie within these bounds. A sketch of the graphs of the two functions shows three points of intersection. 24. The subject is right about the jack 1/3 of the time, about the queen 4/11 of the time, about the king 4/10 of the time, and about all three (1/3) (4/11) (4/10) = 8/165 of the time. -154- 2s. 26. 27. 28. 29. 30. Solutions—Fall 1980 Since D is the midpoint of base 4B of the isosceles triangle, we have BD = AD = 13/2. And since £ lies on a circle with diameter AB, DE = DB = 13/2. Segments Ac and DE are sometimes said to be antiparallel (see, for instance, Kay). c —>— let 2 = [+6 -? [B~e. then 2? = (VIE + 6) - (VB - 6) - 3 VB (fe 6 - [TE ) sow? = 12-3 ¥-8+ x. Thus «? = 12 + 6x, or x’ - 6 - 12 = 0, 80 (a,b,c) = (0,-6,-12). (cos 3¢)(cos 2) + (sin 3x)(sin #) = cos(3x - «) cos 22 = SE, and 0 < 2¢ <7, 50 Qe = 31/5, x = 3n/10. (See 4.314.) cos 2, so Suppose there are @ white balls and b black balls in urn 4. Then the probability of picking a white ball from uyn 4 and then picking a white ball from urn B is (a/8)(1/2) = a/16. Similarly, the probability of picking a black ball from urn 4 and then a white ball from urn B is b/24, Thus we have (a/16) + (b/24) = 3/8, and a+b = 8, These two simultaneous equations have solution (2,6), soa 2. Lay Pe ee phptdebs so cept =a ast say ey = 0. Tus ( 1-0, and $+ LAMTET 88 ote that F = w or u®, the complex cube roots of 1. Let S = sin 10° sin S0° sin 70° = cos 80° cos 40° cos 20°, Then 85(sin 20°) = 8 sin 20° cos 20° cos 40° cos 80° 4 sin 40° cos 40° cos 80° = 2 sin 80° cos 80° (using 4.21) = sin 160° = sin 20°, so S = 1/8. -155- Solutions—Spring 1981 Solutions—Spring 1981 wb. so 222 rel, a= rb, so 42 If the area of triangle ABC is 1 and the area of triangle CDE is A, we have: which leads to 4? - 31 + 1 = 0, whence A = Since the 2 ratio is clearly less than 1, only the negative sign is valid. ce A 8 The discriminant of the equation is p* - 4q, and this must be positive or 0. Thus p? - 4q=1- 49> 0, orq = sin T= a, so that r -156- 10. 1. Solutions—Spring 1981 Note that if the number of heads is equal to the number of tails, the man will have returned to his starting point. And if the number of heads does not equal the nunber of tails, their difference must be even (since the numbers add up to 6), and thus must be at least 2. Hence we need only calculate the probability of not getting 3 heads and 3 tails, This is 1 - P(3H,3P) = 1 -(8) avon) = 44/64 = 11/16. If each student who passed saw only ten out of eleven failures, and each student who failed saw every failing grate but his own, there could have been exactly eleven failures. There could not be fewer, since then a failing student could not see more than nine failing grades. ‘The diagram shows the circumscribed circle. The inscribed circle has radius OM = 1, It is easy to see that angle MOA = 1/7. Also, since 0, the center of the circle, is on the bisector of angle FAQ (by symmetry), we see that angle OAP = (1/2) angle RAQ = 1/14, Hence: x = OA sin 1/14 = 1 + sins, * cos w77 Sin 1/14 _ sin 3/14 b i “ cos 2n/14 = 2b? (using 4.22). ” We have «” = >> A B ¢- OD OD AB 1. Tare aad say If I is Inwin's age and @ is Gilbert's, we have I - G = 8 and I+1=nG. Hence n@-1-G= (n-1)G-1= 8, or 9= (n- 1G. This means that G is a factor of 9 and can only equal 1, 3, or 9. (a + bi)? = b+ at, so a? - b? + 2abi = b + ai, a? - B? = and 2ab = a. From’ the second equation, 5 = 1/2, and from the first, @ = 73/2. Thinking of the complex musbers in polar form helps to motivate this solution. -157- Solutions—Spring 1981 12, 13. 4. 1s. 16. The four points determine in all six line segments. If two of their midpoints coincide, the endpoints of their segments form a parallelogram, and none of the other midpoints can coincide (some of them lie on parallel lines, while others are midpoints of segnents with a common endpoint; if these last coincided, the other endpoints of the segments would also coincide). ‘The minimum number of midpoines is thus five. 2 2 et + Day + y? + 4b, and ¢ > 9, so (@? = 2ay + y®) + day = o= TH. In m minutes, the minute hand will move 6° around the face of the clock, while the hour hand will move (1/2)m°. Thus, at 12:10, the hands make an angle of 60° - 5° = 55°. ‘They will make this angie again shortly before 1:00. If this time is p minutes after 12:10, we have: 5 + Ep + (600 - 69) = 55. (since this tine the hour hand is clockwise ahead of the minute hand). This leads to p = 500/11 = 45 >. Compare Problem 12, above. The four points determine six Line segments. For any selection of three or more of these Segments, at least two segments will have a conmon endpoint. Hence no three (or more) midpoints can coincide. Let us next see how many pairs of midpoints can coincide. Call the points, from left to right, 41, Az» As» and Ay. The only possible pairs of segments without common endpoints are Ariz and Aydu, Aida and Aody, and Aréy and Ards. The first pair is disjoint, 30 they cannot share a midpoint. Suppose i’ were the common mid- point of AyA3 and Azdy. Then M'mist be between Az and As, on the overlap of the two segments. If Ag < Mz, then Mly = AaM < Mts, and A, is to the left of 43, contrary to assumption. If AzM > Mis, then AM = ly < pM, and Ay is to the right of Az, contrary to assumption. If AaM = MA3, then Mly = Mls = A2M, and two of the three points A2, 43, and Ay mist coincide. Hence only Ardy and A243 can share a midpoint. This will happen, for example, if the four points are equally spaced, and five is the minimal number of midpoints. How can this be generalized? The existence of a block of length 2 implies the existence of a 1 before or after the block (or in both places). If the block occurs first, we have 001xx, which can happen four ways, If it appears in the next two places, we have 1001x, or two possibilities. If it appears in the third and fourth place, we have x1001, for two more possibilities. If it occurs last, we have xx100, for four more possibilities. ut we have counted twice any arrangement which has two blocks of 2: the only one in this case is 00100. Thus there are 11 possibilities altogether. A general solution to this complicated sort of problem is given in Feller's Probability -158- iB. 19. 20. 21. Solutions—Spring 1981 5 2 1,1. . Tes eee eet * eT P5908 = 20. Let the vertices of the triangle be at (0,0), (4,0), and (0,3). If (2,y) is the point where the minimum occurs, we must have a minimal value for 2? + y? + (@ - 4)? + y? +a? + (y - 3)? = 30° - 8 + 16 + 3y? - 6y +9. We can minimize each quadratic separately, hence their sum, by taking x = 4/3, y = 1 (think of the formula for the turning point of a parabola). This gives 50/3 for the minimum value. Note: It can be shown, by generalizing the above, that the minimum value always occurs at the centroid of the triangle. It is interesting to note (and challenging to prove) that if X is any point in the planeof triangle ABC and G is the centroid of triangle @ + pte? ABC, then XA® + XB? + XC? - 3x0? = + Let y = ‘VB, so that y? = ve. Then y? + y = 12, soy = 3, -4. The latter is extraneous, so Va = 3 and x = 81. Using Ptolemy's theorem, we have (3.91) To + 7y = 7+ 8, som +y = 8. Also, angle APC = 120°, so in triangle APC, the Law of Cosines gives 2? +? + xy ='49, ‘These two equations lead to ay 15, soe = 3 andy = 5 8 P One could use Hero's formula (3.61), but a glance at the diagram will show why this particular triangle was used. The shortest altitude is drawn to the longest side (can you prove this?), so its length is 1. a 1s -159- Solutions—Spring 1981 22. 23. ea 28. The probability that the cables don't all break in one hour is 1 - (2/3)", where m is the nunber of cables. So 1 - (2/3)"> 4/5, or (2/3)" < 1/5, and m> 4. At least 4 cables are needed, ‘here are many proofs of this result. A very simple one uses mess points, assigning weights of 1 to each vertex of the tetrahedron (see Appendix 8). A more straightforward proof follows. let P be the midpoint of edge CD of regular tetrahedron ABCD. Then ZB, BP are medians in triangles ADC, BCD respectively. Let X and Y'be the respective centroids of these two triangles. Then AXLE = BY:YP = 2:1, and A, B, X, Y, and P are all coplanar. Hence 4B||X¥, If AY and BY intersect at M, then M is on the plane through 4, B, and P, triangle 4B¥ ~ triangle YX", and AM:MY = AB:XY = BM:MY = 3:2, Also, triangle APB ~ triangle XPY, so ABLYY = APLHP = 3:1 Note that this proof, as well as the mass points proof, holds for any tetrahedron, and not just a regular one. In a regular tetrahedron, it shows that the altitudes as well as the medians are coincident (since the medians are perpendicular to the sides to which they are drawn). For more tetrahedron problems, see Aref and Wernick (Chapter 8). A The equation is 3M = 4(@ - 2)(W - 2), since the unpainted region is a block whose dimensions are 4, (l7'- 2), and (WV - 2). Rearranging the equation, we have 3M = 4il - 8(M +H) + 16, or (M = 8)(W - 8) = 48 = 1-48 = 2°24 = 4e12 = 68. The possible values for (u,1) are (56,9), (32,10), (24,12), (20,12), and (16,14). The smallest is 9. If John's age is J and Mary's age is M, then J = rl, We want the : M 1 ratio MIM = (1-1 = at gs apg = ae r <1 or» > 2 there would be no time such as that referred to by the problem. + Note that if -160- 26. 27. 28. Solutions —Spring 1981 AC is a diameter of the circle, so AC = Sv2. By the Pythagorean theorem, FC? = Ac? - AP? = 34.” Now we can use (3.91) Ptolemy's theorem in quadrilateral PADC. 4+5 + 5/34 = (PD)5Sv2, so PD It is not hard to see that all such polynomials are given by f(x) = x for k an integer greater than 3. To prove this, note first that the lead coefficient of f(x) must be 1, and suppose that /(e) included other terms as well. Then we can write f(x) = 2” + q(#), where g(a) #0, and if the degree of q is m and that of f is m, we have m 5, the only new solutions would be f(x) = 0, 1, #, 27, 2°. The centers of the four spheres form a regular tetrahedron of edge 2. The circumcenter of this tetrahedron divides each median, which is also an altitude, in the ratio 1:3 (see problem 23, Spring 1981). If z is a length of an altitude, = 3, sox = V873. wet ae The radius of the large sphere is equal to the radius of the sphere circumscribed about the tetrahedron plus the radius of one smaller 2+ V6 oe sphere, or 1 + 3 875 See also problem 10, Fall 1976, or Coxeter. -161- Solutions—Fall 1981 28. (Continued) z 7 29. To get to (3,1), the particle must move three times along the z-direction and once in the y-direction. The probability of this event is 4p°(1- p). Similarly, the probgbility of passing through G,3) is 4p(1 > p)®. Thus 16 + 4p(1 - p)? = 4p°Q1 - p), or IC - p)* = p?, This leads to 4 - 4p = p and p = 4/5. 30. By dropping altitudes from C and D and using the 30-60-90 triangles and the rectangle that result, it follows that 4B = 8. From similar triangles ECD, EAB, DE:EB = CE:EA = 1:4, Thus BG:EB = AP:EA + 80 EG:EB = EP:EA , and the triangles EFG, E4B are similar. Finally, FG: :EB = 3:4, sox = 6. Solutions—Fall 1981 iy If one Munchie bar costs M¢, the students had M - 7 and M - 2 cents. Together they had 2M - 9 cents, and 2-9 o then y > 2. Since y = 2 when x = 0, we have that the smallest value of y is 2 (or use 6.1 and note that 2° = 1). -162- Solutions—Fail 1981 We can inscribe the regular pentagon in a circle. Then arc & = & = #e 72°. Angle BKA is the average of these two arcs, which is also 72°. c 6 D A é * - 64 = (x)? - (2? + 8)(@? - 8) = (© 2)(@? + Qe + 4)(e + (a? - 2 + a) (See 1.321, 1.322.) Method 1: The two "pieces of the original parallelogram are two trapezoids with equal altitudes. Hence their areas will be equal if the sums of their bases are equal. Since 43 = CD (see figure), we have simultaneously: ¢ + y= 9 +0 yew Subtracting, we find y- @=q-Y, Soy =Q. Adding, we find 2x +y+q=Ww+y+q, 502 Since AB = FC = 3, DF = 5. Axe 9 6 AX Dy Fee a an Method 1 Method 2 Method 2: Line BF must pass through the center of the parallelogram, 50 DF = EB = 5. If it's not clear to you right away that BF must contain the center symmetry of the parallelogram and you think that line XY, for instance, might do the job, draw x'¥'||X¥ through the center #. It's easy to prove that area(AX'Y'p) = area(x'BCY') = 3 area (AaCD). But area(AX¥D) is clearly less than area(ax'Y"D). af(x) + fC - 2) = 22 42. Replacing @ by 1 - 2 we get (1 - x)f( - 2) + f() = 2? - 2 + 3. We solve these two simultaneous equations for the unknown f(s): 2(Q1 - 2) f(@) + (1 - @) FO - 2) = + 20 - 2) F@) + (1-270 - 2) = 2? - 2+ 3. -163- Solutions—Fall 1981 10. ue (continued) subtracting: [22 42 - ft) = - land Fey = tbe wei. (see 1.521) In any right triangle ABC, if M is the midpoint of hypotenuse 4B, then CM = AM = BM. Therefore, angle MCB = angle A5C = 30° and ; elaoel. 8. triangle BXC is @ 30-60-90 triangle, so BX = 7 BC = 5° + AB = w3. A x M c 6 24 ett a0 a ° oa is not possible) 2y7_3 eee ree) (B= F so.e= toe, band a = 3 3 1000027 = (100° + 3*) = (100 + 3)(10000 - 300 + 9) = (103}(9709)(see 1.321) 103 + 7 * 1387 (and 2387 has no prime factor between 192 and 1000) The sum is an infinite geometric progression whose limit is: 110 1/10 T-¥ ~ T- 1/10 ~ 9/10 Alternatively, the weight of the sack can be expressed in decimal f tram. (See 1.525.) notation as 1111... = 4 gran, If we represent the numbers as (n - 5), (# - 4),..65 (H- Ys my (m+ 1), (n+ 2),..., ( + 5), (m+ 6), then their sum is 12n + 6 = 4(3n + 1) + 2, so the remainder is always 2. -164- 12. 13. 4. 1s. 16. Solutions—Fall 1981 Any line bisecting the common area must pass through the midpoint of the centerline, which is (2,4). Since (2,4) satisfies 14r + 3y =k, we have 14+ 2+3+4=k, and k = 40, gp? «hs —1__. sin2IS + c0s?15 | 1 cos7Is * Sin?i5 cos?15 sin?15 cos*15 sin?15 sin730 ~ = 16, so 4B = 4, (See 4.11, 4 (sin? 15) (cos? 15) alniS. 4.21.) Since only one number is chosen, the required probability is the sun of the probabilities of picking 3,6,9,... + mis is simply Eby, a geonetric progression whose sun 1s a iS Bye. (oe 1.525.) leg Set (a + bi)? = a? - b? + 2abi = -3 + 4¢. Then a? - B= and ab = 2. A good guess gives the answer, or substitution shows that a" + 3a? - 2 = (a? + 4)(a? - 1) = 0. Since a is real, a= #1. Taking a = +1 leaves b = By the law of cosines, 2* = 75 + 4 - 4 + Sv3 Ge 49 so BC = 7. BC. Z sin 4 ~ Sin 307” Then by the extended Law of Sines (3-4), 2% so R= 7, Without using the extended law of sines directly, we can draw diameter COA’. Then angle A' = angle A = 30° and angle A'BC =90°, so A'C = 2R = 2BC and again R = 7. Ae 8 8 A \) i ; aN Io Cc -165- Solutions—Fall 1981 17. 18. 19. 20. a. Use 4.4, or: If a= Arctan 2, = Arctan > a= = Z 5,3 . __tana+tanb 574 _ 27 then 2 = tan(a +b) = 7S ay ceam By i. ee 20 (See 4.341.) The given number, is slightly less than 10"? and is in fact 101? - 4096 = 10:7 - 21, This factors as 21? + (57? - 1) and 57? - 1 = (5° - 1)(5? + 1)(5? + 1)(S" = 57 + 1) #5 - 0G? +5+ D6 + D6? -5+ DG? +). ( - 5? +) 2? + 31+2+ 3+ 3+ 7+ 2+ 13 + 601, thus the given number = 2! « 37 + 7 + 13 + 31+ 601. (See 1.321 and 1.322.) Since AC||WZ, we know that triangle DPZ ~ triangle DMG and both are isosceles. Hence DP = PZ, in the same way, we find BQ = @Y. Since ZY = PQ, PZ + 2Y + YQ = DP + PQ + QB = DB. Thus the perimeter of the parallelogram is equal to the sum of the diagonals of the rectangle, or 20. 8 y There are 6° possible outcomes for this experiment. There are 6 favorable outcomes: 12345, 12346, 12356, 12456, 13456, 23456. Hence the required probabitity is & aus: ‘The number must be self-descriptive, in the sense that it counts the number of letters in its own name. For larger numbers, the English names are too short. fence, the number must be small By trial and error, it is not hard to see that only "FOUR" fits its own description. For another example of self-reference, read this sentence. -166- 22. 23. 25. 26. 27. Solutions—Fall 1981 5 4 4, 3 an B = 12 sind = %, sotanA= 25 cosB= se, sotanB= 2. Now tan C = tan(180 - (A + B)] = -tan(A +B) tan A+ tan B Teva etan 4,32 (see 4.341) Tt is not hard to see that neither /@ +t nor /z + 34 can be irrational, and in fact both must be integers. Set # +1 =m? a+ 34577, Then n? - m? = 33 or (n + m)(n - m) = 33 = 1 + 33 = 3 +11. Equating factors gives (m,n) = (16,17) or (4,7), leading to z= 15 or 255. Let W= 204%. Then = (20+)? + 20+ 4-16 = t? + 41¢ + 404 is a multiple of 101 Therefore t(¢ + 41) must be a multinie of 101. Since 101 is prime, either t= 101 or ¢ + 41 = 101 and ¢ = 60. Rence W = 80. If 2 grams of gold were cut off, then » 62 = 72 - 12e, sors 4. If the triangle is 4BC, with angle C = 90°, then by 3.31, AC:0B = 2:5. If the altitude is CD, we need the ratio 4D:B). From AD_ AC 2,4 0D, AC _2 similar triangles, 22 = 4222 ma Be 4-2. maiprying ing 2. DAD. 22 these two equations, we fina 42. 22. 42.2. 2 c a 8 The first few triangular nunbers are 1,3,6,10,15,21,28,36,45,... It is not hard to guess (and to prove by induction) that two out of every three triangular numbers are multiples of 3. These are the elements of the Sequence with subscripts of the form 3k - 1 or 3k. Since n= 100 is not of this form, there are 2+ 99 = 66 miltiples of 3 in the set described. -167- Solutions—Spring 1982 28. 29. 30. We have x = son - 10 must divide n. If m - 10 divides n, x 10 it also divides n - (m - 10) = 10. Hence n - 10 = #1, +2, 45, #10, and n= 11, 12, 15, 20, 9, 8, 5, 0. Now if m< 10, then n - 10< 0 and z <0. Hence possible values of = are 11, 6, 3, 2. The largest is 11. sin 8 + 2 sin 8 cos @ = cos 8 +1 - 2 sin’é We hav 2 sin?e@ + sin 6 - 1 = cos 6 - 2 sin 0 cos @ (2 sin @ - 1)(sin @ + 1) = cos A(1 - 2 sin 8) If 2 sin 8 - 1 #0, this means that sin 9 + cos 6 = -1. Squaring, sin®e + cos?6 + 2 sin 9 cos @= 1+ sin 26 = 1 and sin 20 = 0. No acute angles satisfy this. (See 4.21 and 4.22.) If 2 sin @- 1 = 0, sin @ = 3 and @ = 50°, First note that BC? = AB? + AC? - 2+ AB AC + cos A = 25 + 49 - 35 = 39. Also, since angle AX = angle AXX = 90°, quadrilateral AUXK is cyclic: that is, X is on the circle circumscribing triangle ANK, If the radius of this circle is t, then by the extended Law WK nA maximal. But, since AY is a diameter of the circle circumscribing triangle AIK and is also a chord of the circle circumscribing triangle ABC, its largest value occurs when AY is a diameter of the latter circle. In this case, N= aC = V39. (See 3.92, 3.4.) of Sines, = 2t, or WK = t¥3. Hence MK is maximal when ¢ is BAN x A ‘Solutions—Spring 1982 ie Im base ten notation, squares of integers can end only in 0, 1, 4, 5, 6, or 9. If the difference of two squares ends in the digit 2, the two units digits of the squares themselves must be 6 and 4 or Land 9 (with a "carry" necessary to subtract). In either case, the units digit of the sum of the squares is 0. -168- Solutions—Spring 1982 3087 = 3? + 7°, so we need the sum 1+ 39+ 1+ 3! +1 + 3? 75S a7 47s 47% e347 Bla Te 6 52 477 5? +7? + 31472 + 3%. Factoring, we find this is (1 + 3 + 3*) (+74 7? +73) = 13 + 400 = 5200. For a full discussion, see Dudley (or any book on number theory) . ‘The sum of a set of integers is equal to their average times the number of integers. If M is the average of John's previous tests and W is the number of previous tests, we have 90(W + 1) = MY + 97 a7(v + 1) = M+ 73. Subtracting, (V+ 1) = 3+ 3 24 and 7 Using absolute value to denote area, we have: triangle XBY| _ 3x8 + BY sinB 4 ,1_ 4. Jave| . laxz] [triangle ABC[ “ 3AB BC sin # ~ 5° 5 ~ 35 ~ Yaecy ~ JaBcy (See 3.64.) Hence |triangle 4xZ| + |triangle BxY| + |triangle cYz| 12 bons : 13 = 22 |eriangle aBc|, so [triangle xv2| = 33 |triangte 48c|. B Y A ZC If a, b, @ are the numbers of 1¢, 2¢, and S¢ stamps purchased, respectively, then a + 2b + Se = 100 and a= 10b, Thus 12b * Se = 100, or = 20-422, and b must be a multiple of S. If 2 > 10, 2B > 24, and o <0. Hence b= 5, a= 50, and c We have (a + y)? - 2(@ 4 y} = (ety - les y) = 0 (i), and @ + y)@ - y) - 2@-y) = @ - Wry -2)=0 (ii) From (i), either ae +y-=2org=-y. If+y=2, (@,y) satisfies (ii) as well. If x = -y, we find x = y = 0 from (ii), which doesn't satisfy the conditions of the problem. A similar analysis starting with (ii) shows that 2 + y = 2. -169- Solutions—Spring 1982 ‘The answer (6) is easily read from a good diagram. For a discussion of the general case, see Polya: Mathematics and Plausible Reasoning, n(n = 2) z Vol. I. There it is shown that » lies create tnel regions. It is not hard to see that 2n of these are unbounded. Zi. Sa p $00 4, 50 wo can multiply 5* by any power of 2 up to 2° and get one of the required numbers. This sum will be 5°(1 + 2 + 22) = 5°(2% - 1) = 125 + 7 = 875. Continuing, 59 = 20 > 2", so more such nunbers are (see 1.522) S2(1 + 2 + 22 + 27 + 2%) = S%(25 - 1) = 25+ 31+ 775. 52% = 100 > 25, so we mst add in S(1 +24... + 2) = 5 + 127 = 635 and of course 500 > 2° and 1+ 2+ 2% + +27 + 28 = 29-1 = 511, The sum is 875 + 775 + 635 + 511 = 2796. Let angle ADX = angle AXD = a, angle BAX = angle BXA = b, angle CBX = angle CKB Since angle D and angle C are supplementary, a + (180 - 2c) = 180, soa = 2c. Since angle D and angle DAB are supplementary, a'+ (180 - 24 + 5) = 180, sob = a. Summing the angles about point X, we have a +b + ¢=a+a + (a/2) = 180, so a= 72°. ~170- 10. 1. 12. 13. M4. ‘Solutions—Spring 1982 ah + 64 = 2* + 162? + 64 - 162? = (2? + 8)? - (4z)’, P(x) = 2? + 8, The trick of adding and subtracting the same quantity is difficult to see, until one notices how similar it is to the process of completing the square. We have ——-—908.20 =. c08_2b , _sin®bcos*b Sin7Z(cot® - tan?) ~ sin?a * Tos™ - sin’ sin®b cos*h__ 1 asin? cos’ ~ 4 + (See 4.21 and 4.22.) Let AX:XB = m/n. Then we can write AX = mr, XB = nx, AB = (m+ n)ax, C2 = my, 2A = my, CA = (m+ ny. leriangle 4xz| _ 3AX + az sin A _ mm [triangle ABC] ~ 34> AC sin A” G+ ny? * |triangle xBY| _ |triangie yzcl mn aes txangle 200] ~ Tertangle asc] "OF WF 5 leriangle x¥2] ingle ABC] ‘Then Similarly, Jtriangie 4x2| - |triangle yl - |triangle ¥¢ Jeriangle ABC 3% Cr) : . 1B 3k 2 Writing m/n = ky we have 3e = Te pyr or 6k? = 13k +6 = 0 and & = For 2. Since the problen requires k <1, k= 2. (@ + VB)? = 20 + 3584, soa +b = 20 2vab = ¥384 = 2796 ab = 96 and (a,b) = (8,12). Let BX bisect angle ABC. Then angle XBC = 36° = angle AB, and angle BXC = 72° = angle BCX. Hence triangle BXC is isosceles, and it is similar to triangle ABC. Let AB = AC = a, BC = BY = XA = be xc who phen Lily Then 5G *a° lfp= yrP- ley -1 1+ +5 , But yr > 0, sor a ae For much more on this "golden triangle," see Huntley or Coxeter. -171- Solutions—Spring 1982 4. 15. 16. av. (Continued) Color the first row red, white, blue. There are 2 possible choices for square 4 (white or blue), After this, all colors are determined. ‘There are 6 permutations of the first row, hence 6 x 2 = 12 colorings of the square. Method 1: Use undetermined coefficients (see Hall and knight, Higher Algebra). Method 2: We have, in general: (a? + b?)? = a® + 2a7b? + bY = a - 2a°b? + bY + 4a?b? = (a? - By? + (2ab)?, In the present case, we can let a= @ +1, =<, so (a? - b*)= R(@) = 2a +1, and 2ab = Q(a) = 2e(@ + 1) = 2? + 2e, Land =? + (y + 1)? wey sed ay +2 We have 2? + y? or equivalent. -172- vv. 18, 19. 20. 21. 22. Solutions—Spring 1982 (Continued) c & x A 6 oo By the distributive law, the required sum is: (ebebedey Geddes Since all series are geometric (see 1.522), (AWM) EEE: T T Tog ol-g3 1-5 We have tn gs Ml therefore, nl = 6 and 7 = 3 ve at crwtistar + Therefore, mt = 6 and » = 3. 25 = 4.48, 4.6 50 the answer is at least 8. But is it as much as 97 We have (2V5 + /2I)? = 4+ 5 + 21+ 4/T0S = 41 + 4/105 and (47105)? = 16 + 105. 4? + 10? < 16 + 105 < 4? + 117 so 40 < 4VI05 < 44 and 81 < 41 + 4VI05 < 85 so 9 < 2/5 + /21 < V8 < 10 and the answer is 9. If his salary was 5 before the first year, it was (1 + ba) 5 100 after r the decrease, it we + os) ae fter the increase. After the decrease, it was Q $s) °0- Bs =(1- fe) s- The percent of decrease is : S22 TH) oy (1 1 Ey) t00 = BE 272 4 1 = (284 1? - 20% = (2th 4 1+ 28)(22 + 1 - 2°) = 2113 + 1985 = 2113 + 397+ 5 -173- Solutions—Spring 1982 23. 24. 25. We want to reproduce the situation of problem 14, Spring 1982. Since 108 = $+ 72, we can do this by drawing the trisectors of angle C. If these are CX, CY (see diagram), then angle XCR = angle CXB = 72°, so we can let CB = XB = a. If AB = b, then \X = AB - XB = b- a, Also, angle ACY = angle CA¥ = 36°, so triangle ACY ~ triangle ABC and 8 & ax” aC? Ba Hence if Bb Hs] alo webs we tind eee ee vy? -p- 120 and r= Since b > a, » > 1 and only the "4" sign applies. A *Y B 2 tan B_ T > tan?z First note that tan B = 4 (see diagram). lence tan 28 tan A + tan 25 T= tan A tan 2 1 (see 4.341) and A + 2B = 45°. ‘There are 100 integers in the given set. Let us find how many are multiples of squares. If d(x) is the number of integers divisible by ”, we have d(4) = 253 d(9) = 113 d(25) = 43 d(36) = 23 d(49) = 2 Now multiples of 16 are counted among those of 4, and multiples of 49 cannot be multiples of any other square. But d(4 or 9) = d(4) + d(9) - d(4 and 9) = d(4) + d(9) - d(36). (4 or 25) = a(4) + d(25) - d(4 and 25) = d(4) + d(25) - d(100). Hence we must subtract two multiples of both 4 and 9 and one multiple of both 4 and 25 (the other squares are too large to necessitate this)(see also 7.1), and d(some square) = 22 + 11 + 4 + 2 = 39, and there are 61 square-free numbers. Note that we have included "1" in our definition of square-free. For more on this topic, see Greger. -174- 26. 27. 28. 29. 30. Solutions—Spring 1982 rg m= Y20 + 1472, n= Y20 - 14v2 , then = men, Hence 2? = m? +n? + 3nm(m +n) = 40 + Sm = 40 + 30( V400 - 2+ 196) = 40 + 32 YB = 40 + 6x, son’ - 6x = 40. The equation can be solved by writing: (sin # + sin 3x) + sin 2x 2 sin 2 cos x + sin 2x 2 sin x cos x (2 cos x + 1) = 0, sosine = 0 and z= 0, 7, a 2° 2 or cos x = 0 and x > (see 4.321). and 2 and there are 6 solutions. or cos # =~ Let B be the center of a circle of radius $. Then, looking at segnents of intersecting chords, (4 + 5)(1) = 4 som=2, ‘The probability of getting exactly a heads in b tosses of a fair ‘ ‘ bY 4b 35) 5-35 (36) 5-36 coin is given by(®)2°®, ere we have (38) 279° « (38) 2 or 35! : 36! or 1-736 RIGS = HIB” KS TT - R= Te Tks or k= 17. Draw DF||4Z. Then since D is the midpoint of AB, F is the midpoint of BE, and also DF = AE = CD. Letting angle FDB = angle FAB = angle EAC = x, we have angle CDF = 90 - x, angle CBD = angle CAD = 2x, angle CFD = angle DCF = 3x (it's an exterior angle in triangle FDB). In triangle CDF, 90 - © + 3 + 3e = 180, Therefore x = 18 and angle ACB = 6x = 108°. -175- ‘Solutions—Fall 1982 30. (Continued) Solutions—Fall 1982 One could solve the quadratic resulting from multiplying both sides by 2, or merely nore that 2+ 2=3+1sor=3,1. there camot be more roots as the equation will be quadratic. The following argument shows that there is only one such ordered triple. The second condition given can be written as (a + 5) (a - b) =e. This implies a+b =e anda - b= 1, since c is prime. Then a=b+1, soa=3andb = 2 (since 2 and'3 are the only consecutive integers that are both prime). Finally, c = a+b = 5, and the unique ordered triple is (3,2,5). If the original number is k, we have added 100 - 10n - n = 89” to the nunber by multiplying by ”. That is, nk = k + 89, or (n - Nk = 897. But 89 is prime and n - 1 is relatively prime to n, so n= 1 must be 1, m= 2, and k = 178. (ies Fi)’. Vey? If TU is the common tangent, draw OpX parallel to 7 diagram) ¢* = 144 - 16 = 128, so TU = VI2B = 8/2. Y20 + 2736 = Y32 = 2. Then (see For each marble, there are 3 choices for an urn to place it in. This gives 3'° possible arrangements--including some where an urn will end up empty. If we choose any one urn to stand empty, there are 2'° arrangements of marbles in the other two urns. Hence -176- 10. 1. Solutions—Fall 1982 (Continued) 3+ 2!° arrangements leave an empty urn, and this number must be subtracted. However, we have now subtracted off twice the 3 arrangements in which two urns are empty, so we must add back 3 (see 7.3). This gives 3! - 3 + 21° + 3 arrangements. If the numbers are A and B, we have AB = 588000 = 2° + 3+ 5? + 77. We need to "distribute" these factors over the numbers A and B as evenly as possible. Doing this, we find that GCD(A,B) = 27 + 5 +7 = 140. Compare with problem 1, Fall 1982. Let y= /=2*4. then LS, soy = a y=2, E24. Ly ysheS,soys2ory=$. y= 2, 2%4t-4ande- $8 vel 2c iW wyed, Spe 4 ane --. Since « + 2= 5 (a constant), the minimal value is achieved when 2 2 » or @ = YS, and « +2 = 2/5 (see 6.4). For more such problems, see Beckenbach and Bellman, or Korovkin. ‘The two figures have a common apothem. Since the area of any regular polygon is tap, the ratio of the areas is the ratio of perimeters of the two figures in question. If the side of the octagon is #, then the side of the square is e(1 + 72) (see diagram). Hence the required ratio is —**—~ = —2_ = 2/7 - 2. 4e(1 + V2) 1+ V2 5 6 If T is the required point of intersection, let 07 = 2, O27 ‘Then = + y = 12, and from similar right triangles 0,P?, 0207, $=}. sox = 3y. Hence x = 9 and y = 3 The distance from circle 0, = 3-2=1. -177- Solutions—Fall 1982 ul. 12. 13. M4. 15. (Continued) P Ce 2 4b? 4 ce? + 2(ab + be + ca), so We have 0= (4+ b+ 0)? =a (a? + b? + 0)? = 4(ab + be + ea)* = 4(a?b? + 87c? + o7a? + 2a%be + Zab e + abe?) = 4(a?b? + bee? + 27a? + 2abe(a + b + e)) © = 4(a®b? + be? + oat). But also, (a? + b® + *)* = a* + bY + ct + 2(a2b? + b2e? + c7a?). 26 BP 4 08)? - 2(02B? + Be? + ofa) = 2(a?b? + ba? + 7a). (a + be + 0)? ae be + ot constant. See Hall and Knight, Higher Algebra (Chapter 34) on symmetric functions. Hence (**) a* + bY + of Comparing (*) with (*#), we find that =2,a Note that 2' = 2, 2? = 4, 2? = 8, 2* = 16, and thereafter the last last digits repeat in a cycle or 4. Hence 25 of the nunbers given end in the digit 6. 1 asd. Let x= logue. thon 2+ tog,8 Cece 6.12), andy» E Hence logy = 4 or logyt = t and x = 256 or VF = JZ. o = (EEDOED 5 mintnar when o® t9 mininal. o? 234245 is minimal when 0? - 5 = +a is minimal. $+ = 4, a constant, so the minimal value of their sum occurs 2, or # = 2, ande -¥#- 3 (see 6.4). Also see references given in problen 9, Fall 1982. -178- 16. 17. 18. 19. Solutions—Fall 1982 Let the radius of the third circle be ©. From similar triangles we have 2-72 gp 249, 16+ ‘ oPa, PQB, we have Fr = FE, or 2+ $= E*= | Cross multiplying 2n 32 ives 2n? = 288, 212. or : gives 2n? = 288, soz = 12. Or, using 1.1, we have: >> = 2 2. _16 ZW oaae ‘ or esate ten Gs 48, mde = 12, For useful exercises on ratio and proportion, see Hall and knight, Higher Algebra (Chapters 1 and 2). 2a + 3b = 31. Since a - b is a multiple of 4, a and b are either both odd or both even: in fact, it is easy to see that they must both be odd. Looking at the equation modulo 3 we have 2a = 1, soa =2mod 3. The first few possibilities are a = 5,11,17. However, 2+ 17 = 34> 31, and a = 5 gives b= 7 anda -b which ig not a multiple of 4. Hence (a,b) = (11,3). Let the sides of the triangle be a= b-d,b, e=b +d, with b= 2, and let the inradius be r. The semiperimeter ¢ = (a+b + c)/2 = 36/2 = 3. Also, if the points of contact of the incircle with the sides are X, Y, and Z (see diagram), we have BY = BZ= 8 - b= 1, Now OBY is a 30-60-90 triangle, so OX = ¥3/3 =”, and the area is re = /3/3 + 3 = V3 (see 3.5 and 3.62). Let tan Ore. thm eede ss + so tan 6 = 3, } and (see 4.12) sec 6 = tVfan®G FT = +/T0 or 4 Since 0 <6 <4, sec 0 is positive. -179- Solutions—Fall 1982 20. a. 22. 23. 24. substitution gives 2a - 5 = 8. Hence (2,0) = ( a+ 4b= 8. and another substitution gives k = -66. sy +e) desert dig z = z G@+n0+2) 4 so +4 is smallest when « = 4 or 2 = 2, and z = 2 (see 6.4 and problem 9, Pall 1982). By Fermat's "Little" theorem (see Dudley), p [ 10+ 10°" = 1 mod p, so p|10. The largest such prime is 5. Let the radius = 2. Then (see diagram) Ox + PY = RP = 12. From right triangles JX and FOr, YET E~V? + OF a ay = 12 Vie + V3 = 12 sated. Note that cos(A - B) = cos A cos B + sin A sin B< 1 (see 4.314), with equality hotding when 4 = 8. In triangle ABC we have also, since 0 < sin C< 1, 1 = cos A cos B + sin A sin B sin C < cos A cos 8 + sin A sin B< 1, so cos 4 cos B + sin 4 sin B = 1, A= 8, sin@= 1, and C= 90°. Thus A = B = 45°. -180- 25. 26. 27. 28. 29. Solutions—Fall 1962 eo We can write 1 + 32 Lewenta a? + as follows: 3 Qn + ar +e + 2(m? + 2? see guts 1+2 1s 20serwte Le sesso ee pee Se 22s) t+, . (See 1.523.) Lettings = 4, y, T= aa xr we obtain the value 3. Let (4+ JIE)" =u. Then (4 + VIB)* + (4 - IBZ =u +2. As in problems 9, 15, and 21, Fall 1982, u + 1 is least when us,sou=1 (+0) and its value then is'2. Note w+ =2+ av, 2 (see 6,1 and problem 9, Fall 1982). ‘The sum of the digits mist be 9. The largest sum possible with no digits 3 is 8, and the smallest with three digits 3 is 10. Hence there must be at least one 3 and at most two. If there is only one 3, the other digits must be 2's: there are 4 such numbers. If there are two digits 3, the other digits must be 1 and 2. There are 12 of these, so altogether there are 16 such nunbers. W is of the form 10” - 1, By Fermat's "little" theorem (see Dudley), 10! - 1 is a multiple of 17. We must show that no smaller exponent will work. Let us first check divisors of 16: 10? = -2, 10" = 4, and 10° = -1. Now if 10Y = 1 mod 17 and y < 16, let the G.C.D. of y and 16 be d, Then we can find m and n such that ym + 16n = dy or 10+ 102" = a4, or 1 = 10%, This is impossible, as d is a divisor of 16. -181- 30. ns—Spring 1983 Multiplying out, we have: sin?3@ - sin 3@ cos ® = sin?9 - sin @ cos 38 or: sin*30 - sin?6 = Sin 36 cos 6 - sin 6 cos 38 or: (sin 30 + sin 6)(sin 38 - sin 6) = sin 38 cos @ - sin 6 cos 30. Using the formula for the sums and differences of sines (on the left) and for the sine of a difference (on the right), we have (see 4.321, 4.322, 4.312): (2 sin 28 cos 6)(2 cos 26 sin 8) = sin(30 - 8) 4 sin 8 cos 20 sin 26 cos 6 = sin 20 sin 20(2 sin 26 cos 26 - 1) = 0, so either (i) sin 20 = 0, or (see 4.21) (ii) 2 sin 26 cos 28- 1 = sin 49 - 1 = 0. From (i) we find 28 = 0, 7 and 6 = 0, 1/2. a 3a a $n Fo Fande-t, discussion, see Honsberger, Ingenuity in Mathematics (essay 16). From (ii) we find 40 = For a deeper Solutions—Spring 1983 1234321 = 11 + 112211. It is not hard to factor this last number. In fact, 112211 = 11 + 10° + 22 + 10% + 11 = 11(10° + 2 + 107 + 1) 11(10? +1)? = 11+ 1017. The square root, then, is 11 + 101 un. Take AB = 4, so that AP = 3 and PB = 1. Using absolute value for area, we have |4PS| = |P6Q| = |acr| = |x0s| = 3, and |aBc0| = 16, so that |PQRS| = 16 - 4|APS| = 16 - 6 = 10, and |PQRS|:|ABCD| = 5:8. Or: use the Pythagorean Theorem in triangle APS. A> iB @ ok eC Between 1 P.M. and 2 P.M. the hand travelled 27 radians. Between 2 P.M. and 2:30 it travelled 7 radians, and between £50 and 2:55 it travelled 7 radians. Altogether, it travelled 181 sagians, since the hand travelted eloclaise (by definition lot 6 we can write the answer by the usual convention as - -182- Solutions—Spring 1983, there are ($\ = 15 ways to form a first subset, From the remaining four elements, there are () = 6 ways to form a second subset. The last subset is whatever is left. Hence there are 15 * 6 * 1 = 90 ways of choosing a first, second, and third subset. Since we have counted each combination of three subsets 1s. 31 = 6 times, the number of divisions is 92 If the roots of x* - px +q=0 are rand, then p = 7 + 6 and gq =,P8- Hence p = r?e? = q? and q = r? + sé = (r +,5)? - 2re =p’ - 29. Substituting, we find q = q" - 2g, or q? so aq: Vandp= ¥. Mess cosy s cay”, We can write the given sum, modulo 11, as: 4 For odd if this is clearly 0. For even if, the sum is congruent to W 2 1 aa er eM eae Ms aca a sl) = 2c + 16%) = 2027" + 2) ee + 1 must be (1+ 2"), For this to be a mitiple of 11, 2% a multiple of 11. But then 4” = -1 (mod 11). A quick check shows that this never happens, so the answer is 6. (Je)? = (feo ae)" - ‘ pay: 1 —.l wy 8 In the diagram, G is the centroid and ZiT is a median. Then AG 2GA", and from similar triangles AGX, A'GY, AX = 24'Y, so a'y 5. Then, in trapezoid BCW2, A'Y is a median, so if CW = x, 2+5=a2+6, and = 4. Or: set up a system of coordinates in which one of the axes coincides with £, and use the theorem that the coordinates of the centroid are the averages of the coordinates of the vertices. ~183- Solutions—Spring 1983 10. ll. 12. 13. 4. Also, B= 2+3,s0B-+=2 sia A=1l+7, 504 Fl aie tus A+B - (Lel)s1s2+5. see also o1ds. a + cos A)? = 1+ 2 sii = 1 + 120, 289 (sin 4 + cos A) 1+ 2 sin A cos A 1 Teo * eo * Therefore, sin A+ cos 4 = 42 (see 4.11). ACB suppose 4 = k. Then 1004 + B= 10sk + Bk, or 1004 - 104k = Bk - B = B(k - 1). Since the left side is a multiple of 5, the right side must also be. And since the right side is positive, so is the left, and 1 Yc ‘The fractions described are all contained in the sequence 1/24, 2/24, 3/24, . . ., 23/24. Hence there are 23 of them. Since the function to be minimized involves addition and the condition given involves multiplication, we will try using the arithmetic-geometric mean inequality (see 6.2). Here, we have 2 sy? POSES VET for 20:* + Sy? is 140, occurring when 202? = Sy*, or y = 2x. See problen 9, Fall 1982. = YI00Ry? = 10xy 70, so the minimum Since 10° < 61224 < 10°, we have 10° < 61224? < 101°, and 612247 has 9 or 10 digits. Now /T0® = 10*V10 < 10" + 4, while 61224 > 6+ 10" > 4+ 10" > /T0. Hence 61224* > 10° and has 10 digits. Let us first assume that each face of a polyhedron with the maximal number of interior diagonals is a triangle. If such a polyhedron has F faces, F edges, and V vertices, then it has () ~ F interior diagonals. Hence we must minimize Z. In such a polyhedron, 3F counts each edge twice, or 3 = 28. Using Euler's formula (+ F=£ +2), we find 2 - p+ =2 ore = 3v- 6. Hence the maximal number of interior diagonals is (¥) - 30 - 2) W-39W - 4) 2 pyramids sharing a common base that is a (V - 2)-gon. For V = 10, the answer is 21. The reason we choose polyhedra with triangular faces is that if a face has more than three vertices, some of the diagonals will fall on the faces, and so will not be interior diagonals. See Coxeter (Section 10.3). This can be achieved, for example, by two convex If the factory actually produced ” fleebles per day, it produced 10W fleebles altogether. It was supposed to produce “ fleebles : per day, for 12 days, or $8! freebies. tence 480 + 42 « 107 = 32, or 42 = 2 and n= 108. -185- Solutions—Spring 1983 20. 21. 22. 23. 24. in ‘ 5 5 tf Arcsin & = = Fy and cos(90° - @) ~ & £ Arcsin 7z = 8, then sin 6 = Fy and cos(90° - 6) = 5, nd Arccos Fy = 90° - in Se = and Arccos 3 = 90' Hence Aresin yy + Arccos 3 = 8 + 90° - @ = 90°. 13 “|. rm We use the arithmetic-geometric mean inequality (see 6.2): Sw? ETT = amy, sot +? Any, or 4zy < 50. Hence (@ + 2y)? = 2? + any + dy? < 50 + 50 100, and x + 2y < 10. Equality occurs when x = 2y = 5. See problems 16, Spring 1983 and 9, Pall 1982. Let AE = kt, AC = ky, and suppose AD = 4 and DB = 3. Then, using absolute value for area, |ADE| = 3° 4+ ke + sin A, |ABC| = 3+ 7+ ky + sin A (see 6.34) and the ratio of the areas is 421 7 AE aa tye and ee 7. Hence = = y é 8 % If they bought g sticks of gum, r rolls, and ¢ candy bars, then g+r+e= 100 and g + 10r + 50¢ = 500, Subtracting, 9r + 49¢ 400. Hence 1 < ¢ < 8 (since 9” = 400 is impossible) and r _ 400 - 49 oa - me 44 - 5: 4 4e= 4(1- ec). Only e= 1 works. ‘Then » = 39 and g = 60. Since v is integral, 9 divides If the given polynomial is P(w), its roots are equal to those of Qa) = FP@). Since Q(x) can be factored as (x - 14) (a - 12).+. ( - Ps), it is not hard to see that the given expression is merely -@(-1), This is, mumerically, - (1+ $-1+3+$63 3°35 -186- 25. 26. 27. 28. 29. Solutions —Spring 1983 The numbers « and x* will always have identical remainders upon division by A if and only if the number 2? - x = x(@ + D(e - 1 is always a multiple of A. Since any set of three consecutive integers contains at least one even number and exactly one multiple of 3, a(@ + 1)(@ - 1) is always a multiple of 6, Trying the first few possible values of x, we find: 6 - 6-6-6765 P-7=7+8+6 8-8 =8-9+7=654+3°7 9? -9=9+10+11=6+3+5+11 At this point it is clear that 6 is the largest possible A. [2ab + (a? + B® - 07) }[2ab - (a? + B? - 07) ((a + by? - e7][e? - (a - 5)71 (a+b+eja+b-ad(e-at+bera-b) 2s(2e - 2a) (2e - 2b) (26 - 2e) l6e(s - a)(s - )(s - e)= 16 = K* = s(s - a)(s - bY - e) 1 (see 3.61). One of the squares mist be larger than 90/3 = 30, hence it must be 36, 49 or 64, Now 90 - 64 = 26 = 25 +1, 90 - 49 = 41 = 25+ 16, 90 - 36 = 54 is not a sum of distinct squares. The desired triples are (1,5,8) and (4,5,7). Squaring the first two equations and adding, we have: costa + cos*y + cos’z + 2 cos © cos y + 2 cos y cos 2 + 2 cos z cos z + sintx + sin®y + sin?z + 2 sin x sin y + 2 sin y sin = +2 sin # sin 2 # 0, or 3 + 2[cos(a - y) + cos(y - 2) + cos(s - «)] 3 0 (see 4.11 and 4.314), so that the required sum is - >. Note: If we think of 2 = (cos x, sin x) as a unit vector and define ¥ and 2 similarly, the given equations state that the three vectors sum to 0, Since they each have unit length, this means that their directions must differ by 120°. Visualizing this leads to a quick solution of the problem. Suppose a set of cubes displays all possible colorings exactly once. Place the cubes on a table so that their green sides are all face down. Then there are five possible colors showing on their top faces. For the "side" faces there are then, a priori, 24 colorings. But each cube can be rotated four ways to get another set of colorings on the sides, so there are actually only six distinct colorings for these faces. Hence there are 5 x 6 = 30 possible colorings altogether. -187- Solutions—Fall 1983 30. Let us suppose, quite generally, that the sides of the triangle are @b,e witha a2 > a3 > ay > as. If as = 1, a > 2, a3 > 4, a2 > 8, and a; > 16, which is impossible. Hence as = 0. If a, >2, an argument like the one above shows @, > 12, which is again impossible. Hence a, = 1. Reasoning similarly, we find a5 > 3 to be impossible, so a; = 2. The three solutions 94210, 95210, and 84210 follow quickly. asin (ASE) cos (AGL) = 2 os (£42) 4.sa1 and 4,523), s0 sin 422 2 cox AEB on A+B Ey son +0 - Zork - B=, 34, ote., shich is not possible within the given limits. Thus Note that F137 and PY. Tf we draw OZ||¥Z, with Z on PF, then in triangie OZ, op? = 02? + PZ? = 7 + (PY - ZY)? = 7+ (PY - OX)? = 16. Li IN xy Clearly W must be a multiple of both 2 and 3. Let +k then 4 = 21. 3) +k, If this is a perfect square, a - 1 and b must be even, and k must be a square. Also, = 2° Hence a and b - 1 must be multiples of 3, and & mist be a cube. The smallest a satisfying both of these conditions is 3, and the smallest b is 4, For to be smallest, we choose k = 1, so 23+ 3% = 8+ 81 = 648. ‘The shortest distance will be the smallest number of the form 75a + 330b (for integers a,b with a > 0). This is the greatest conmon divisor of 330 and 75, or 15. -189- Solutions—Fall 1983, 10. 1. Clearly, the center of the given circle is on the perpendicular bisector of line segment 4B. If this center is 0, then (see 3 = 10 + OY. From right triangle A0Y, (3 - 7)? +2? = 9, so9- 6r+4 diagram) OX = 04 = 0B = r, and XY 13 0 and r The diagram shows triangles drawn on the four sides of ABXY that are congruent to triangle ABC. These form a new large square (since angle XB2 = angle CAB is complementary to angle CBA and C, B, and Z are collinear), whose side is 12. OC is half its diagonal, or 6/2. po) ow cre 2 A number is a multiple of 9 a multiple of 9. Since 31989 = 9 + 319°, each a, is a muiltinle of 9, Now 32282 2 2. gi9e6 21. gs92 21. ygo92 Now 3 tes F-9? < 3-10 digits. Then a, < 9 + 992 < 10000, a3 < 4+ 9 = 36, and a < 9. Since a is a multiple of 9, we must have a = 9. For i> 4, £ and only if the sum of its digits is » SO a; has at most 992 A comparison of triangle ABC with a 3-4-5 triangle shows that angle B is obtuse, so angle ACB is acute. Since angle ACD is the supplement of angle ACB, cos angle ACD = -cos angle ACB. Using the law of cosines, AB? = BC? + AC? - 2BC + AC + cos angle ACB AD? = CD? + AC? + 2CD + AC + cos angle ACB so AB? + AD? = 16 + 36 + 16 + 36 = 104, and AD® = 104 - 9 = 95, Note that this method shows that if AC is a median of triangle ABC, then 4ac? = 248? + 2Ap? - BD. -190- lu. 12. 13. 14. rs. 16. Solutions—Fall 1983, (Continued) 6 4 € 4D We need zy = 10“, & a nonnegative integer. Since x,y < 100, k<4. I€K=0, (@y) = (1,1). For k= 1, sy = 10. There are four integral divisors of 10, none of them greater than 100, so there are four ordered pairs. For k = 2, we have (2,50), (4,25), (5,20), (10,10), (20,5), (25,4), (50,2). For k = 3, we have (20,50), (25,40), (40,25), (50,20). Altogether, there are 1 + 4 + 7'+ 4's 16 possible ordered pairs. Since sin*A + cos*A = 1 (see 4.11), cos*d = sin®B, so cos A =tsin B. If cos A= sin 8, A+B = 90, soC= 90°. If cos A in B, 4 +3 = 180°, which is impossible. The number 1440 (base q) must be a multiple of 11; that fs, 11 must divide q° + 4q° + 4q, or 11 divides q(q + 2)?. Since 11 is prime, this means that 11 divides q or q + 2, and since q < 10, q+2= 11, and q = 9. Checking, 14415 = 10901, = 11+ 99 + 1.— Any figure remains fixed upon rotation (about any point) of 360°. ‘The figure in the problem must remain fixed under any combination of rotations about P by multiples of 48° or of 360°; that is, under rotations by angles whose degree-measure is of the form 48a + 3605 (for positive or negative integers a,>). The smallest such nunber is 24= 48+ 8 - 360+ 1. ‘Triangle BXY is similar to triangle ABC, and its area is } that of triangle 4BC. Since the ratio of the areas of similar figures is the square of the ratio of the sides, the ratio of sides here mst be 1:72. Hence BY = BC/vZ = 8//2 or 4v2. A -191- ‘Solutions—Fall 1983 17. 18. 19. 20. 1 B (see 5.12), we have 55% Tear Since 1og,9 = 75 a or 3 log,n = log.n® = log.m, and m Draw O¥,PR. Since BC connects two midpoints of triangle PQR, it bisects any line segnent from @ to line PR. Hence (see diagram) QM = MH = BX, Now AX = YD = 3(AD - BC) = 1, so 3X = V36-1 = V35 = QM. Since BM = 3, BQ = V35 +9 = VA% and PQ = 244 = avTI, Q B/m \c PAX HYOR We have: ai? = 49, so az = 4 +9 = 13 ag? = 169, soas=1+6+9= 16 es" = 256, so ay = 245 46-15. Since a, depends only on a,_,, the sequence is 7,13,16,13,16, 15,16,... and aige3 = 16. Call the given property §. Clearly, if a set of m (> 4) points has , so must any subset with (m- 1) points, so we can "build up" the set one point at a time. Suppose we have 4 points, 4, B, C, D having @. Then 3 form an equilateral triangle, say triangle ABC, and D could be anywhere. Adding a Sth point #, we see that D and & must form an equilateral triangle with at least 2 of 4, B, C3 hence with exactly 2 (say, A and B), since in a plane 2 points can form an equilateral triangle with at most 2 other points. We now have A, B, C, D, E as in the diagram (note that the positions of D and & are determined only relative to each other). Try to add a 6th point F, Now F must form an equilateral triangle with 2 of D, E, C, hence must be an X or an 0; but {%,A,8,D} and {0,4,B,D} don't have f. Thus n= 5. A ° ehod 7% Bk ; Le bv¥ ° al. 22. 23. 24. 25. Solutions—Fall 1983, Adding, then subtracting, the last two equations gives x = 3(2 + a), y = 3(2- a). Substitution in the first equation shows that 2a+a?+2-a= 2, 0ra?+a=0, anda=0,-1. Both solutions work. Triangles AMP, ANQ, ACB are similar, and the ratio of their areas mg _ v2 B, so = aS is 1:2:3. Hence the ratio of their sides is 1: If wQ 4, then BC [[BC. since az:2B = aQ:ac, AQ = 2 AC, and QC = Yo = Sac. Also, 29 = AQ = }AC = XC, so triangles Z9¢ and YXC are congruent. Thus angle YXC = angle QZC is complementary to angle @CZ and angle XPC = 90, Note that this remains true as long as the ratios AX:HC, CY:YB, BU:ZA are equal. Write n = 3k +» where, k > 0 is a nonnegative integer and r = 0, #1. P= 92 4 ir +r? Ha 2 + 2h + Ey and since : yr Oorg, [55] = 52 + ann = K(Sk + 2m). This is prime only if k= 1 and r= 00or 1, These produce n = 5% + r= 3 or 4 (both check). ‘The two-digit multiples of 17 are 17,34,51,68,85, and those of 23 are 23,46,69,92. None of these nunbers begins with the digit 7. Every other digit begins only one number, except 6, which begins two. Hence the sequence can "branch" whenever a 6 occurs but must end if a 7 occurs. Hence all such finite sequences end in 7. The shortest such, for example, is 9,2,3,4,6,8,5.1,7- -193- Solutions—Fall 1983 26. draw DF] BO. Since ares AB and xD are equal, 4B XD = CD, so angle YXD = angle BCD = angle BAD = 30°, and xY = (¥3/2) xp = 4(V3/2) = 273. Hence xc = 4v3. BxY oe Zw 27. The original cone is similar to the cone formed by one of the new volumes. Since the ratio of the volumes of similar figures is the cube of the ratio of linear dimensions, 1 Le, onde 2. v2 28. Adding the three equations and noting that [«] + {x} = 2, we have 2 +y +2 = 3.3. Subtracting from this the sum of the first two equations shows that [y] + {z} = 0. The sum of an integer and a fraction (0 < {2} <1) can be 0 only if both are 0, so @ is an integer, and 0 10p,8 = 3, 106428 > 105927 = 3, so the sum is preater than 3, -196- 10. ul. 12. 13. Solutions—Spring 1984 First let us assune that the square is symnetric about the altitude of the triangle. Then, if 2 = MD (see diagram), angle AMD = 45°, ye ies " 2 so DE = BM = —, and ap = —*+=—. But AZ + EM = —+—= 6, YE ns AB 9V2 - 3¥6 and (psq) = (95-3). To prove that the position of the square must indeed be as indicated, it is enough to prove that the square is unique. This can be done by noting that point D is the image of point F when rotated about M by 90°. Hence D is on the image of line BC under the rotation. This image is a line and can intersect AC at only one point. c D, F ax Note that in this range, cos « - sin « is positive. Now (cos x + sin x)? + (cos « - sin x)? = 2 (see 4.11), so (cos # - sin 2)? = 2 - 25/16 = 7/16, and cos x - sin « = v7/4. We can take the sum in pairs. Let R() = the remainder when a is divided by ae Maar RO" + aw) The required sum is o 1 +0 1 RQ +34041+42400+1+4+1) +0) 10241 3 = R(S + 24 + 6) = R(S) = 5. 2 4 +4 0 3 041 1 4K 0 «+0 0 4ak+1 0 +1 1 4K +2 0 44 4 K+ 3 0 41 1 W must be a perfect sixth power, The only such number in the given range is 729. In fact, 729 = 99 = 272, ~197- Solutions—Spring 1984 ua. Angle XGW is supplementary to angle ACB. By the law of cosines, AB? = 5? + 7? -2+5 + 7 cos 6 and WW? = 5? +77 42+5+ 7 cos 6. Hence AB? + xW? = 2(5? + 7?) = 148. w 2ON ° A 8 15. IE the roots are ri, Yay .s+s M7» then their sum is 0, and >!” = Bry = 1, rat? = Bmp = 1, ryt? = Srp = Ly sees M7!? = Smay = Le Adding gives m2? 47,17 6 2. 4 mp!7 = 30m +m 4 0. + m7) = 17 = -17. 16. A train leaves Times Square at 10:30 and arrives at Main Street at noon, This is the first train met by the noon train out of Main Street. The noon train meets each train out of Times Square from 10:30 until 1:30 (when it arrives at Times Square). There are 13 such trains. 7. Since angle CED and angle CFD are both inscribed in semicircles, each is a right angle. Hence CFD is a rectangle and EF = CD. The area of the triangle is 54, which must be equal to 348 + CD scp = SA = 56 e+ 15+ CD. Hence EF = CD = te755 3 c F é S$ 8 -198- 18. 20. a. Solutions—Spring 1984 ‘ ‘ sin?2x) (cos 22 Write coste - sin’ + (in'2e) (cos 22). = cos*s - sinbz 4 « (sintz cos*z)(cos*e - sin’) (see 4.21 and 4.22) 4 cos‘ - sine + sin’s costs - sins cos*x (cos*x - sin*x)(cos’« + sin*x) (cos*s - sin?x)(cos’# + sin’x)(1) (see 4.11) (cos*s - sin*x) (1) = cos 2x. Now cos 2x = 0 if and only if 2x = 90° or 270° and x = 45° or 135°. Since 4(4B) < 100, A= 1 or 2. Since CA is a multiple of 4, hence even, A= 2. Therefore, 2 < 5. Now since 4(4B) ends in 2, 8 = 3, soc = 9, Let the rectangles be ABCD and AXCY, Then triangles ADP, PYC, (BQ, QKA are congruent (see diagram). If DP = 2 = PY, then AP = AY - PY 8 - a. Thus AD* + DP = 2? + 2? = AP? = (8 - 2)?, so4 = 64 - 16e ands =P. then ae D is an altitude in parallelogram APCQ, whose aren is thus 2(6 + 2) = 12. Y », p fe ) & 8 Let n= va. Then vb = kn, and if 1000 < a < b < 10000, then certainly 31 << kn < 100. Hence k can be only 2 or 3. Suppose a, in decimal notation, is ABCD. If k = 2, then 4(ABCD) = DCBA. Hence A is even and cannot be more than 2(or 6 > 10000). If A= 2, then 4(ABCD) = DCBA > 8000, so.D = 8 or 9. D cannot be 8, as no square can end in 8 (in decimal notation). And if D = 9, 4(ABCD) ends in 6, not 2. Hence k cannot be 2. If k = 3, 9(ABCD) = DCBA, so that A= 1 and D= 9. The only perfect square satisfying this is 337 = 1089. Indeed, 9801 = 99? and 99 = 3 + 33. ~199- Solutions—Spring 1984 2, 23. 24. We use absolute value to denote area. Since triangle ABX ~ triangle CDK, AX:XC = AB:DC = 2:3, and AP:PD = AX:XC = 2:3 as well. Since triangles APX, PXD have the same altitude from X, the ratio of their areas is also 2:3. Since triangle PXD ~ triangle ABD and DP:DA = 3:5, then |PxD{:|aBD| = 9:25. Thus if [AP¥| = 2k, then lexo[ = 3k and |4BD| = (28/9) + 3k = (25/3)k, and the ratio JaPx|:|4BD| = 6:25. For any point P outside the unit circle, the length of the tangent {see diagram) is ve +y2- TI. If (x,y) is on the given hyperbola, this distance equals J? ~ ay + 8 = VIG DF + 6 > VG, with ve. equality when y = +1, © Pcs) Let x = the required interval. If a bus passes the student at point A, then the next bus arrives at point A «-minutes later. Tt must then spend 12 - minutes catching up, so it takes 12 - 2 minutes to travel the same distance that the student walks in 12 12-2 1 meets a bus at point B, then she passes the next bus 4 minutes later, so that the next bus has spent « - 4 minutes going the same distance that the student has gone in 4 minutes. Hence the ratio of the rates is “j4. the two ratios must be equal (since the speeds are constant). Hence 452» 2—4 minutes. The ratio of the rates is then If the student and 2 = 6. -200- 25. 26. 27. 28. Solutions—Spring 1984 If there are b boys and g girls, then b = g + 1 and 2(g - 1) = be Hence 2g - 2=g +1, 9 = 3, andb There are 7 children in the family. By symmetry, OY (the perpendicular bisector of FS in the diagram) bisects angle 40B. ‘Then angle OQR = angle 09% + angle XQR = 30° + 90° = 120°. If QR = 2, then XQ = Fi and from 30-60-90 triangle 0X0, Using the law of cosines in triangle 0gR, 13* = OR? 132 - 2 ye : os 4 YS . gt (A2B) tence ot 39(4 - VB) ° 4+ 78 = 156 - 39/3. ve have 2? - bgt -Sa5 «Wats gg) - M2 ge - We have 2? - bat - S05 + ats og? - We +... + mMrararsrer7. These are just the “symmetric functions” of (1.4), so this sum equals 1 - ag + as - ay + a3 - az + a1 - ae = (2+1-5- 1740+ 419 - 372 + 10)/2 = 32/8 = 19. Amore direct solution is accomplished by using the result from problem 24, Spring 1983. Using this insight, if P(e) is the given polynomial, we need only calculate (-1/2 (-1). For more information on the symmetric functions of the roots of a polynomial equation, see Borofsky, or Shklyarsky, Chentsoy, and Yaglom (Chapter 9). We can write: y?=«@-1+. Since = andy are integers, © must be an integer. Hence « = 1,2,3,6, and y? = 6,4,4,6. But if y* is a perfect square, y? can only be 4. The possible solutions are thus (2,2) and (3,2). -201- Solutions—Spring 1984 29. 30. ‘The student can always choose from the three largest piles, whose sum must be at least 50. For suppose their sum were less. Then the smallest of the 3 could not have more than 15 pennies (if it had 16 or more, the sum of these 3 largest piles would be greater than 16 + 17 +'18 > 50 pennies). ‘Then the remaining piles could not total more than 14 + 13 + 12 + 11 = 50 pennies, and there could not be 100 pennies altogether. A set of piles of 19, 16, 15, 14, 13, 12, and 11 coins shows that disturbing three piles may in fact be necessary. There are 2 cases to consider: {ase 1: The right angle intersects two different sides of the triangle. Moving from the symmetrical situation BP ~ BQ to P' and @" we see Area 1 > Area II, since OP = 09, angle POP' = angle @0Q', and P'0 > @'0 (P10 is further from the perpendicular). So we lose more area than we gain and area OPBQ > area OP'BQ' (with limiting case P' = B, @" between B and ¢). Case 1 Case 2: The right angle intersects the same side of the triangle twice. We can start with BP = OQ and "rotate" the angle towards vertex 5. By an argument similar to that of Case 1, area triangle POQ < area triangle P'09' (with the same limiting case as above). Hence the maximum area = area OPBQ as in case 1. wove = 2 (4) «B, In case 1, from triangle BPR, 3B 2 v3 1 v3, or « = ————— and the area OPBQ = 5(PQ) (OB: ~ We $(P4) (08) 1 3 1 =e (8) -—_. 300 (5) 3a + -202- Solutions—Fall 1984 30. (Continued) A short calculation will show that the maximal area in case 2 is 1 only 75+ Solutions—Fall 1984 L Let AP = 2 and PB = 3c, Then triangles APS, PAQ, QCR, ROS are congruent, and the sum of their areas is 6c’. The area of the large square is l6r*, so the area of the smaller square is 10x”, and the required ratio is 10:16 = 5:8. AxP 3x 8 x Q 2x] s 3x x D 3x RC She will get all three questions correct $ of the time. Failing this, she will get only the first problen wrong § 1 of the time, only the second } of the tine, and only the third § of the tine. 1,1 s 1 she will get at least. two right $+ 242+ L- of the tine. Hence 3 Using (3.5), we have CX = Of = 2, XD = 7D = 4. If M is the intersection of BD and OP, then BM = MD (by symmetry), so MD = 5 and Mf = MD - TD = 1. Hence in right triangle om, OM = v5 and = 25. -203- Solutions—Fall 1984 3 (Continued) 6 c Pp wW y e eg Rk 4, The given cube is slightly less than 8 + 10°, so the integer is slightly less than 2000. Also the rightmost digit must be a 3 (otherwise the cube would not end with a 7). The possible choices are 1993, 1983, 1973, etc. Trial and error can finish the solution. Or, we can note that 19933 > 1990? = 10°(200 - 1)? = 103(8 + 10° = 3+ 4+ 10" +6 + 107 - 1) > 107(800 - 12) = 788 + 107, so 1993 is too big. Writing 1973? as (2+ 10? - 37)? and proceeding similarly, we can see that this number is too small. 5 We have: (i) x® - 2xy = 0 and (ii) x? = y +3 From (i), 2(@ - 2y) = 0, soe=0ore=2y. If z= 0, logs is undefined. If 2 = 2y, we have dy? - y - 3= 0, so (4y'+ 3)(¥ - 1) £1 and 2 = =, 2. only the positive values will satisfy (ii). = 0 and y 6. Expanding and rearranging the terms yields: (cos 15 sin 75 + sin 18 cos 75) + (cos 1S cos 45 - sin 15 sin 45) = sin(IS + 75) + cos(45 + 15) = 1+ $= (see 4.311 and 4.313). 7. A(z) = fQ - 2) + When h(2) = 8, we solve for x to yield 1 - 8 ste cos 15° + sin 15° e Let W cos 15" sin 15" * + sin215° + 2 cos 15° sin 15° ¥sintis" — 7 cos 15° sin 15" ‘Then i? (See 4.11, 4.21.) simfrotos Since cos 15° > }> sin 15°, is positive and so W = V5. -204- 10. le 12. Solutions—Fall 1984 Draw 44, WB, 20. since 8c cx and triangles 28¢, Zcx have a common altitude, [triangle 25c| = [triangle 20x] (using absolute value for area). Also, since 4B = BZ, |triangle 4cB| = |triangle cBz|. Similarly, we find that |triangle axC| = |triangle Axy| = |triangle ‘ABC| and |triangle AYB| = |triangle 2¥2| = |triangle ABC]. Hence the required ratio is 7:1. Y YSSS, pea z If the roots are x - d, r, and ” + d, then the sum of the roots is 3r = 6, So r= 2 is a root. Substitution in the original equation gives 8 - 24 - 48 + ¢= 0, and c= 64. A round trip for A takes 40 minutes. A round trip for B takes 30 minutes. A round trip for ¢ takes 64 minutes. We need the least common multiple of these three. Since 30= 2+ 5+ 3, 40 = 2° +5, 64 = 2°, the least connon multiple is 2° + 3+ $= 64’ 15 minutes, 1 or 64 + 7 = 16 hours. From the Pythagorean theorem, AB = 1. Also, the median to the hypotenuse is half as long as the hypotenuse. Therefore, sind 1 cos 6° 2° tan 6 = From this equation, cos ® is clearly the 2 longer leg, and by the Pythagorean theorem: (28 SY" + cose = 1 and solving yields cos @ = 28 5 A . c cos 8 6 -205- Solutions—Fall 1984 13. 14, 15. 16. A line connecting two midpoints of a triangle is equal to half the third side. Hence MP = 3FD and MP = 34C, so AC = DF, Similarly, BC = EF and AB = DE, so that the required ratio is 1. Letting # = 5, we have 2f(5) + f(-4) = 25. Letting # = -4, we have 2(-4) + f(5) = 16. These are two equations in two unknowns. Solving, we find f(s) = 3, f(-4) = 2. Im general, f(e) watt de-1 22+ . We use 3.95. Let angle ABC = 8, angle ACB = 1, and the radii of the circles be r and F (the circle of radius r is through 4, 8, and P). Then AP = 2r sin 8 = 2R sin y, so r:R = sin y:sin B = 5:8 (see 3.4). A c 8 ? 2 ey? wt - dey tyre yee We have 2—+-¥- = 2 = 2ey + y’ + 2ey _ (e - y)*? +2 wry ony ony 2 2 = @-yesty. wt ese-y. Thenz>0ande+2 es mini 2 a constant, sos +2 is minimal when 2 = 2, or s = V2 (see 6.4). this gives = S52) y- Bo yg BAH 29 except to check that both are positive, it is not necessary to solve explicitly for z and y. Refer also to problem 9, Fall 1982. -206- 17. 18. 19. Solutions—Fall 1984 We need only consider n > 0. Let m= 3k + £ where £= 0, 1, 2 and k>0, 16220, x? = 91%, [%] = 342, prime only for k 2 If2=1, [5] = 3k? 4 2k = k(3k + 2), prime only for k = 1 If 2 oF 1 = (3k + 1)(k + 1), which cannot be prime. Letting » Let P' be the image of P when rotated about a typical point @. Triangle PP'@ is always right and isosceles, so angle GPP! = 45° and PP' = @Py2, Hence we can obtain P' from @ by rotating 45° about P and dilating the resulting figure (performing a honothecy) about P by a factor of YZ, This means that the locus of P' is another square, whose side is ¥2 and whose area is thus 2. For more on honothecy, see references cited in problem 21, Spring 1975. e P let circle 0 be tangent to ZF at x and to BC at Y. By symmetry, point 0 is on diagonal BD. If OD = Ox = OY = r, then OXBY is a square, so 0B = r/Z, ‘Thus OD + OB = v + ¥/3 = 08 = V2, 0 = -2(1 - V2) = 2 - V2, and (pq) = (2-1). 1+ V2 -207- Solutions—Fall 1984 20. 2. 22. Adding, we have 2? + 2ny + y? +a +y = 42, (w+ y)? + (ety) - 42 =0. lett +y=a. Then a? +2 - 42=0, or z= -7,6. Subtracting the original equations, we have x* - y* +2 - y= @+y@-y)+@-yYs@ty+DeE-y=-14, 7 sor-y=Z or w-y= etys-7 atys6 (ey) = (-7/5,-14/3) (ey) = (2.4) Clearly Of = 20, Since 59 = 24, OP = 7, we know OP = 25. Then, if angle POQ = 8, angie TOP = f, we can write angle OQ = (8 + @) : . =O, 24 =, 20 and OX = a= OF cos(8 +B). Now cos d= =2, cosp= 2-2, 24,207, 18 _ 375 _ 3 re teas os mass and OX = 20 z 12 so cos(@ + #) = (see 4.313). If we draw the circle through D and C which is tangent to Bd at B, angle DEC is } arc CD. For any other point E' on BA, angle DE'C is less than 2 are CD, since other points are outside the circle. Hence point E is the point we want. To find BE, let 0 be the center of the circle and of the midpoint of OD. Then ZB, GE|BE, ana or = BM = 2, Then EB = MO, and BB? = MO? = 0p? - mp? = of? - yp? 4-1 Thus BE = ¥3. -208- 22. 23. 24, 25. Solutions—Fall 1984 (Continued) én A BC ° The probability of neither being a spade is % Be HE. wence the probability of getting at least one spade is 1 - # B. We compute the ratios of areas of triangles with equal altitudes by comparing their bases. Using absolute value for area, let lanr| = 4. Then [rex] = 4, |axs| = 34 = [xsy], |yrx| = [rex] = 4 2 (all from ratios along line QS). Also, since [P?s|:|e7s| |prs|: (4 35 rien = eS 4, so |Prs| From ratios along 3 Pa: (pYr| = 3a. Thus |Prsy| = [pry] + [rvx| + |yxs| = sa, and |P¥s| = |prsy| - prs] = sa - 3 $ If P4 = 2 = PB, then PC = 10 - x, and PC? + cA? = PA, or (10 - 2)? ses, 100-202 + 28+ ee 24, 116-2 = 0, v2 HS. Kk x Puo-k a A -209- Solutions—Fal! 1984 26. 2. 28. 29. 30. After the first sock is picked, the chance of the second sock matching is 4/9, rey = 0 if and only if f(z) = 0 while g(z) #0. This happens for 2 and 9. Suppose there are numbers in the set. Clearly, n is more than 2. We will first show that cannot be 3, 4, or 5. For example, if n = 3, the numbers can be represented by a, b, and 3/a, so that bla = ab. Hence |a| = |b| = |b/al = 1. But then at least two of the three numbers must be equal, contrary to the assumptions of the problen. Similar arguments will show that ” cannot be 4 or 5. Next we can show that n cannot he greater than 6. Indeed, if the first two numbers are a and b, then the next five are b/a, 1/a, 1/b, a/b, and a, 50 that the seventh number is equal to the first. Hence n = 6. The Sequence 2,3,3/2,1/2,1/3,2/3 (among others) will satisfy the conditions of the problen. Drop altitude ZF onto FZ. Then AD is the bisector of angle EAB, so BD:DE = AB:AE = 2:73 (see 3.31). This leads to DB = 1/(2 + v3) and De = a8 . Then the area of triangle ACD is (1/2) (0) (AE) 248 = (3 - V3)/4. ¢ E > ‘ 8 Labei any vertex of the 11-gon as point A. Then we can label tie next five vertices going clockwise around the circle as Bl, B2, B3, BA, BS, and the five vertices going counterciockwise as Cl, C2, C3, C4, CS. Let us first find out how many of the triangles described have a vertex at A. Clearly, of the other two vertices of such a triangle, one must be some Bi and another some Cj for 1 <7, J <5. And in fact, it is not hard to see that ¢ + j > 6 if the triangle is to contain the center of the circle. Now it is not hard to count the triangles: each corresponds to the solution of the above inequality for 7 and j natural numbers less than 6. There is one solution if ¢ = 1, two if t= 2, and so on. Hence there are 1 +2 + 3+ 4 +5 = 15 triangles with one vertex at A. Letting each vertex of the 1I-gon play the role of A, we now have 11 + 15 = 165 triangies. But we have then counted each triangle three times (having let each vertex of each triangle play the role of A), so that the number of triangles is actually 165/3 = 55. This argument can be generalized for a regular polygon with an odd number of sides. -210- ADDENDIX A ALGEBRA LL If a:b = e:d, then: Lill (a+ d):b = (e+ Did 112 (a+b):(a-b) + (e+ Ai(e- ad LAS aid = ord = (a +e): + a) Binomial Theorem: tf (7) = wectet » Wor ken), then: (a + 5)” (@) ayrk Tos 1.22 (a + b)* = a? +b? if and only if a = 0 or b = 0 or a+b=0. 1.311 Factor Theorem: For P(x) a polynomial in x and 7 a real or complex number, ¢ - » divides P(x) if and only if P(r) = 0. 1.312 (Corollary) The polynomial P(z) divides the polynomial Q(#) if and only if every (complex) root of P(z) = 0 is also a root of G(x) = 0 (including multiplicities of distinct roots). 8, (a + b)(a? - ab + By 1.322 a® - b* = (a - b)(a? + ab + b?) -211- Lau Sei-#1, p. 34 F82-#16, p. 42 877-#30, p. 15 2 S75-#2, p. 1 $76-#30, p. 9 FI7-#10, p. 16 11 S75-F18, p. 2 F75-#8, p. 4 F75-#10, p. 4 F76-#2, p. 10 F77-#20, p. 17 F78-#18, p. 22 S80-#10, p, 29 S77-#6, p. 13 S77-#8, p. 13 ¥77-#26, p. 17 $79-#21, p. 25 FB1-#4, p. 37 F81-#9, p. 37 F81-#18, p. 38 1,322 S80-F26, p. 30 F81-#4, p. 37 F81-#18, p. 38 3 S77-#25, p. 14 F77-87, p. 16 F78-#8, p. 21 $80-#19, p. 30 F80-#14, p. 32 154 For odd n, at + y= (we yt? - ay + ay? ne? yet. 7 ry ty). te P@) se" - a, tea ee Ca + (-1)"a, = 0 has roots ry, m2, Pay ss.) 7,» thent ay (the sum of the roots) a rrp, (the sum of the roots "taken 2 regen two at a time") a, = 2 rrr, (the sum of the roots "taken TY eee gekent three at a time") a, 2 rmrs. Arithmetic progression: If a1, a2, ... 5 a, are in arithnetic progression with conmon differelice d, then: LS a, = a1 + (n- Dd Fra + @- al = Fa + a,) Geometric progression: If ai, a2, «++, a, are in geometric progression with common ratio », then: -212- 1.34 S7I#10, p. eke) S75-#30, p. FIS-#6, p. 4 F75-#8, p. 4 F75-#9, p. 4 F77-#13, pe F77-#21, F78-#24, p. S79-#17, p. $79-#20, p. 884-#27, p. 1.511 S76-¥7, p. 7 F80-#10, p. 1.512 F75-#11, p. 879-29, p. FI9-#7, p. 2 1,522 24 3 16 p. 17 22 24 25, 51 32 4 26 6 S82-#8, p. 40 S82-#18, p. 1,523 S76-F1, p. 6 S76-#16, Ps S76-#15, P. F76-#19, p. S77-#12, Pp. S78-#24, ps S79-#25, Dp. F81-#10, p. F81-#14, p. F82-#26, Dp. 40 8 1 aL 13 20 28, 37 37 43 n(n + 1/2 De Moivre's Theorem: For any complex number r cis 0 (or cos 6 + ip sin 8), (r cis 6)” = »” cis (n6). Rational Root Theorem: If P(x) = az" +a@, a7! +... + mm + a, is a polynomial with integer coefficients and p/q is a rational root of the equation P(x) = 0 (where p and q are relatively prime), then P must divide a,, while q must divide a,,. If P{«) is a polynomial with real coefficients and P(a + bé) = 0 (where ¢ is the imaginary unit and a,b are real), then P(a - bi) = 0 as well. If P(z) is a polynomial with rational coefficients and P(a + bYé) = 0 (where a, b, and ¢ are rational and ¢ is positive but not a perfect square), then P(a - hve) = 0 NUMBER THEORY, Pythagorean Triples: All positive integer solutions to the equation a® + b* = c* are given by: a= km? - n?) b = k(2nn) @ = k(m? +n?) where m, n, and k are arbitrary positive integers, with m>n, For relatively prime triples (a,b,c), we need k= 1, and and » relatively prime and of opposite parity. Fermat's "Last Theoren" for n = 3: The equation a? +53 = e° has no solutions in integers unless abe = 0. -213- 1.53 S78-¥6, p. S78-#8, Pp. S78-#20, P. S78-#28, P. S78-#23, p- 1.6 F78-#18, p. 22 F78-#30, p. 23 1.7 Fis-#6, p. 4 F75-#10, p. 4 S76-#6, p. 7 S80-#50, p. 31 1.81 S80-#10, p. 29 s80-#20, p. 30 1.82 F77-#28, p. 17 $80-#10, p. 29 2.4 F75-#25, p. 6 S76-#9, p. 7 F76-#30, p. 12 S78-#10, p. 18 78-#9, p. 18 2.3 Fermat's "Little" Theorem: If p is prime and a is relatively prime top, then ro @' ~ 1 is a miltiple of p. FB2-#29, p. 43 GEOMETRY 3.1 The medians of a triangle are concurrent at a point (the centroid) that divides each median in the ratio 2:1. ide ¢ Ua? + py - 1 .2 For median m, to side o, m,* = (a? +b?) - 3 ¢' a b CS € 3.3 If angle bisector t, divides side ¢ into segments of 33 lengths m and , ther F75-#5, p. 4 876-42, p. 6 3.3L aid = mn F78-#12, p. 21 F81-#26, p. 39 $83-12, p. 45 a b FBd-#29, p. 54 "Ww 32 ab- m=? ‘The "Extended Law of Sines”: If R is the circumradius F78-#16, . 21 > P of triangle ABC, wo aR F79-#24, p. 28 sina * in F79-#30, p. 28 F81-f16, p. 38 3.5 If %, Y, and Z are the points of contact of the sides Rese coe of triangle ABC with its inscribed circle and ¢ is the Sos triangle's seniperineter, then: 3.5 — c 577-822, bes tl 878-430, p. 20 BX = BZ =9-b Ni F82-#) 43 AY 2aZ = 0-0 4 Bp. F84-#3, p, 52 If a, b, @ are the sides of triangle ABC, k its area, © its semiperimeter, R its circumradius, and r its inradius, then: 3.61 k= Ve@>a)(@- B)ts~ 2) (Hero's Formula) 3.62 kere 83k = abe/4R 3.64 k = (1/2)ab sino In an equilateral triangle, the sum of the distances from any interior point to the three sides is always equal to the altitude of the triangle. 1. 1£ ax|la¥|lcz, then a8 c Ceva's Theorem: If AP, BQ, CR are concurrent at % (as shown), then: A AR, BP, 00 we Pc Gam} RL\S@ WR 3.822 Jet + Bel B L e Menelaus' Theorem: If "transversal" PR intersects the sides of triangle ABC in P, Q, and F (as shown), then: A PP, BQ, CR P wa mt & B c In parallelogram ABCD, AC? + BD? = 4B* + BC? + CD « DA?, -215- 2 Pe 10 F77-#28, p. 17 S78-#3, p. 18 $80-#18, p. 30 S8l-f21, p. 35 $83-#26, p. 46 3.62 S75-#3, p. 1 S78-#2, p. 18 S80-#18, p. 30 F82-#18, p. 43 3.63, 3.64 578-43, . 18 S79-#2, p. 25 S79-F15, p. 24 S79-F18, p. 24 F79-#4, 'p. 26 F79-#15, p. 27 $80-#12, p. 29 $82-#4, p. 39 $83-#22, p. 45 $83-#30, p. 46 3.7 76-#14, p. 7 81° S77-#14, p. 14 3.83 S77-#1, p. 13 3.84 FR0-#4, p. 31 F80-#12, p. 32 3.91 Ptolemy's Theorem: In any quadrilateral ABCD, AC + BD < AB + CD + AD * BC, with equality holding F75-#9, p. 4 FIS-#15, Pp. 5 if and only if quadrilatera? 48CD can be inscribed F77-#3, p. 15 in a circle. F77-#12, p. 16 $81-#20, p. 35 As $81-#26, p. 36 P. 16 F81-#30, p. 39 F83-#30, p. 49 F84-#15, D. 53 4.1 F76-#12, p. 11 F76-#14, p. 11 S79-#8, p. 25 $79-#21, p. 25 FT9-#5, p. 26 $80-#17, p. 30 F80-#19, p. 32 F81-#13, p. 37 ‘TRIGONOMETRY $83-#10, p. 44 $83-#28, p. 46 F83-#13, p. 47 SB4-F11, p. 50 For a quadrilateral to be cyclic (to possess a circumscribing circle), it is necessary and sufficient that its opposite angles be supplementary. In a circle of radius 7, the length of a chord which subtends an inscribed angle 8 is 2” sin 6. ‘The "Pythagorean" Identities: fa + costa = Att sin’ + costs = 1 S84-#18, D. 50 tant + 1 = sect Cae 12 4.13 cot?e + 1 = esc% F82-#19, p. 43 1 Fe-#14, p. 11 878-413, p. 19 oe 878-#22, p. 19 4.21 sin 2e = 2 sin 2 cos = F78-#30, p. 23 : : - $79-#8, p. 23 4.22 cos 2 = cos’s - sin’e = 2 cose - 1=1- 2 sin’ S79-#18, p. 24 F79-#22, p. 28 F81-#29, p. 39 982-411, p. 40 F82-#30, p. 43 $84-#18, p. 50 Fad-#7, ‘p. 52 2 F78-#30, p. 23 SB1-#8, 'p. 34 F81-#29, p. 39 $82-f11, p. 40 $84-#18, p. 50 Double and Half-Angle Formulas: -216- 4.23 tan = (sin 2x)/(1 + cos 2x) = (1 - cos 2#)/sin 2x 4.24 sinte = 30. Addition Formulas: 4.311 sin (+ y) sin (@ - y) cos (+ y) cos (# - y) sing + sin sin @ ~ sin cos # + cos cos @ - cos sine sin y cos z sin y cos x cos y sin @ cos y tan (z+ y) intes face cos, 2x); sin > a> cos 28); cos $+ [LE SRe sin = cos y + cos @ siny sin # cos y - cos # sin y cos # cos y - sin siny cos 2 cos y + sin # siny = 2 sin (w + y)/2]cos{(e - y)/2] = 2 cos[(e + y)/2lsin{(@ - y)/2] = 2 cosf(e + y)/2]cos{ (a - y)/2] -2 sinl(@ + y)/2Isin[(e - ¥)/21 (1/2) feos (@ - y) = cos (@ + ¥)) (1/2) {sin (@ + y) - sin (@ - y)) (1/2) feos (@ + y) + cos (@ - y)1 (1/2)[sin (@ + y) - sin (@ - y)] tan + tan y T- tan @ tan y -217- 4,28 F76-#14, p. 11 4,24 F76-#14, p. 11 877-#4,'p. 13 F79-#5, p. 26 ¥79-#10, p. 27 4.311 877-723, p. 14 $77-#30, p. 15 F84-#6, p. 52 4.312 577-¥30, p. 15 F82-#30, p. 43 4.313 F76-#25, p. 9 F8d-#6, p. 52 F84-#21, p. 53 F82-824, p. 43 S$83-#28, p. 46 4.321 577-30, p. 15 ET7-#17, p. 16 S79-#4, p. 23 F82-#30, p. 43 F83-#4, p. 46 4.322 582-F27, p. 41 F82-#30, p. 43 4.323 F77-¥9, p. 16 F77-*17, p. 16 F78-#10, p. 21 F83-#4, p. 46 4.331 FrB-Fio, p. 21 4,341 F76-¥3, p. 10 878-#12, p. 19 FI9-#10, p. 27 F79-814, p. 27 F81-#22, p. 38 $82-#24, p. 41 4.342 tan (=) = Een ne tan? vi (A+B 4.4 tan A+ tan B = tan (4 ~ AB 4.51 sin 30 = 3 sing - 4 sin’x 4 coste - 3 cos & cos 30 MISCELLANY For a, b, ¢> 0, a,b, @# 1: 5.11 (log,b) (log,e) = log,e (the "chain rule” for Logarithms) SZ 108) * Tapa If [a] denotes the greatest integer no larger than 2, then: [(@ + m/n] = [({2] + m)/n], for m, n integers, and n> 0. 5.22 For nonintegral « > 0, if [2] = , then [-] INEQUALITIES If 2 > 0, f(s) 22+ Ife attains its minimum value when = = 1, 6.2 The "arithmetic-geometric mean inequality:" for a, b> 0, (a+ 5)/2 > Vab, with equality holding if and only if @=b. In general, for ay, a2». a, > 0» Hear + a2 + 1. + a,) > Vayaenood, , with equality holding if and only if a1 = a2 4, 6.3 If a+b remains constant, the product ab is largest when a = b. -218- F79-#10, p, 27 F79-#11, p. 27 4 S77-#13, p. 13 F81-#17, p. 38 #28, p. 25 5.11 S7R-#7, p. 18 S78-#17, p- 19 F78-#11, p. 21 28, p. 28 $80-#13, p. 29 F82-#14, p. 42 F8S-#17, p. 48 6 876-46, p. 7 F81-#2, p. 36 F82-#27, p. 43 $85-#50, p. 46 6.3 F76-#7, p. 10 $78-#29, p. 20 F80-#21, p. 33 6.4 If ab remains constant, the sum a +b is smallest when 6.4 a=b. FB2-#9, p. 42 F82-#15, p. 42 F82-#21, p. 43 ‘The Inclusion-Exclusion Principle: If we use |4| to F84-#16, p. 53 denote the number of elements in the set A, then: ra , 382-425, p. 41 za ly ay - 5 la asl 12 i=l * lsi

Vous aimerez peut-être aussi